2ちゃんねる ■掲示板に戻る■ 全部 1- 最新50    

■ このスレッドは過去ログ倉庫に格納されています

プログラミングのお題スレ Part9

1 :デフォルトの名無しさん:2016/12/01(木) 16:58:30.97 ID:gTkHDluD.net
プログラミングのお題スレです。

前スレ
プログラミングのお題スレ Part8©2ch.net
http://echo.2ch.net/test/read.cgi/tech/1444216746/

【出題と回答例】
1 名前:デフォルトの名無しさん
  お題:お題本文

2 名前:デフォルトの名無しさん
  >>1 使用言語
  回答本文

【ソースコードが長くなったら】 (オンラインでコードを実行できる)
http://ideone.com/
http://codepad.org/
http://compileonline.com/
http://rextester.com/runcode
http://runnable.com/
http://code.hackerearth.com/
http://melpon.org/wandbox
https://paiza.io/

宿題は宿題スレがあるのでそちらへ。

2 :デフォルトの名無しさん:2016/12/01(木) 19:14:53.02 ID:sow3BhsA.net
即死回避

3 :デフォルトの名無しさん:2016/12/01(木) 21:04:32.24 ID:bG1bMw/g.net
puts 'そ'

4 :デフォルトの名無しさん:2016/12/01(木) 22:56:02.34 ID:sN9AIsZM.net
printfn "れ"

5 :デフォルトの名無しさん:2016/12/01(木) 23:26:28.91 ID:Vh7HCfjU.net
http://pc10.2ch.net/test/read.cgi/tech/1169413998/656
この問題
2つの1バイトのビットパターン b,w (b&w == 0) に対して
r = base[b|w]+b (or base[b|w]+w)
を重複なく定める配列

6668まで縮める配列を見つけたけどもっと縮める或は6561ぴったりにする配列はあるのだろうか

int base[] = {
612,614,1681,1054,4528,3808,6078,2684,3278,3262,6349,336,5719,4410,6249,
700,1587,1584,5960,2284,4738,3352,6230,2412,4496,3514,5924,516,4770,3546,6636,
2876,4371,3096,6497,1952,5977,3880,5696,2668,6153,4268,5708,320,6235,3984,6182,
812,4354,4282,6549,1576,5713,3384,6541,2372,6029,4350,6425,1508,6155,4090,6605,
1760,3620,3496,5182,692,6058,3612,6094,1020,3694,3774,6317,580,4732,3480,6217,
716,6260,3740,5672,1416,6050,3768,6086,2180,6254,3674,6256,1288,6048,3898,6284,
2844,6522,2968,5680,1936,5711,3864,6142,1132,6093,3646,6313,564,6169,3936,6297,
1140,6510,4226,6489,2156,6508,3192,6514,2540,6025,4222,6413,1572,6027,4048,6380,
2716,4396,3266,5559,548,4690,3712,5628,980,5319,3246,5555,368,4540,3214,5567,
908,5380,3258,5800,1544,4876,3320,4938,972,5436,3486,5828,1512,4706,3290,5491,
2308,4693,3250,4755,1980,5833,3792,5791,1912,5285,4204,5281,1956,5769,3198,5265,
1888,4691,4218,5866,1960,5860,3728,5839,2348,5862,3450,5636,256,4428,4026,5201,
1696,4947,3500,5150,568,4969,3456,4961,556,4945,3642,4977,464,4636,3658,4993,
876,4955,3608,5640,1424,4682,3512,4906,2148,5410,3390,5376,1284,5404,3802,5459,
1252,4685,2952,5146,1908,5721,3472,5122,996,4701,4176,4949,448,4620,4188,5130,
732,4699,2944,5792,2064,5735,3200,4866,2468,5374,2940,4687,260,4426,2942,4427,0
};

6 :デフォルトの名無しさん:2016/12/02(金) 00:12:44.35 ID:gTW8SJ/a.net
リンク開けねーし日本語意味不明だし

7 :デフォルトの名無しさん:2016/12/02(金) 00:32:31.53 ID:IjczpQdN.net
二つの2^8の形で表されている実質3^8の状態を一回の256要素の配列の参照と加算で
0から6561になるべく近いユニークな数に変換するその配列を求めるって問題

for (w = 0; w < 256; w++) {
for (b = 0; b < 256; b++) {
if ((b & w) == 0) {
if (++a[base[b | w] + b] > 1)
printf ("err\n");
}
}
}
これをエラーが出ないように通す配列

8 :デフォルトの名無しさん:2016/12/06(火) 09:57:55.18 ID:4tk1iq5o.net
>>1おつ

9 :デフォルトの名無しさん:2016/12/07(水) 14:23:07.62 ID:ltGABuaa.net
景気付けにどうぞ

お題:以下の数列を出力するプログラム
[0, 10, 1110, 110, 2110, 122110, 11222110, 2122110, 1211222110, 11122122110,...]

10 :デフォルトの名無しさん:2016/12/07(水) 18:06:29.71 ID:qM0eFzSC.net
法則性が分からん

11 :デフォルトの名無しさん:2016/12/07(水) 19:07:54.18 ID:ijEXmeus.net
>>9 Java
http://ideone.com/SjEe5N

>>10と同じで法則性考えるのに時間がかかったわ

12 :デフォルトの名無しさん:2016/12/07(水) 19:15:52.19 ID:Gz2S0y7j.net
>>9 Nim
https://ideone.com/WLvRqB

13 :デフォルトの名無しさん:2016/12/08(木) 00:20:55.59 ID:Gvasb8BC.net
>>9
・法則見つけた瞬間までがピーク
・実際書くと何一つスッキリ書けなくて悔しい
https://ideone.com/n0RoNr

[

14 :デフォルトの名無しさん:2016/12/08(木) 00:21:30.57 ID:Gvasb8BC.net
>>9 ruby
・法則見つけた瞬間までがピーク
・実際書くと何一つスッキリ書けなくて悔しい
https://ideone.com/n0RoNr

["0", "10", "1110", "110", "2110", "122110", "11222110", "2122110", "1211222110", "11122122110", "12211222110", "11222122110"]

15 :デフォルトの名無しさん:2016/12/08(木) 00:53:14.68 ID:T7wNCBuS.net
>>9 ruby ちょい整理
https://ideone.com/kyw9bm

16 :デフォルトの名無しさん:2016/12/08(木) 22:20:30.70 ID:eRjONenV.net
・前項の数字の1の位から順に見てって、数字が連続する数を数えていく。
・数字とその連続数を1の位から順に埋める
・3の数字は削除する

17 :デフォルトの名無しさん:2016/12/09(金) 16:43:25.82 ID:pZMdQcgu.net
お題: 文字列に含まれる複数のURLをHTMLのaタグで囲って出力してください。詳細な仕様はないです。
例: http://ideone.com/NY2ooA

18 :デフォルトの名無しさん:2016/12/09(金) 21:12:11.14 ID:Ees/IyTq.net
例に出されたURLも不正だし
>詳細な仕様はないです。
となると判別不能では

19 :デフォルトの名無しさん:2016/12/09(金) 21:52:57.66 ID:BOiFZI5h.net
>>9 Nim
あまりに不細工だったので書き直し
https://ideone.com/9q80v4

正規表現使うバージョン(ただしideoneは古すぎてコンパイルできない)
https://ideone.com/01ktu2

20 :デフォルトの名無しさん:2016/12/10(土) 04:17:33.77 ID:mNmdekz4.net
イデオンって投稿時にURL置換するんだよな。

21 :17:2016/12/10(土) 11:48:18.29 ID:E4ED1H/i.net
ideone.comをフィルターしてるのかと思ったらURL全般だめっぽい

22 :デフォルトの名無しさん:2016/12/11(日) 18:00:53.14 ID:RSXbZT65.net
>>9 Perl
use feature qw{:5.16};
sub {
$_ = join'', @_;
exit if length > 12;
say;
@_ = ();
while (/((.)\2*)$/) {
$len = length($1);
$len = '' if $len > 2;
unshift @_, $len . $2;
$_ = $`
}
goto __SUB__;
}->(0);

無名関数の末尾再帰を使って記述してみました。

23 :片山博文MZ ◆T6xkBnTXz7B0 :2016/12/13(火) 23:58:53.18 ID:AlD2ABgf.net
お題:C言語に関する質問に答えられる人工知能を作れ。

24 :デフォルトの名無しさん:2016/12/14(水) 02:25:35.75 ID:EQpNWDBR.net
自分で作れ

25 :デフォルトの名無しさん:2016/12/17(土) 22:23:37.55 ID:zexuiF2f.net
案1 Amazon Mechanical Turk を使う。
案2 知恵袋に投げて、回答を転記する。
案3 IBM Watson 使う。

26 :デフォルトの名無しさん:2016/12/17(土) 22:35:16.13 ID:R0k7IOIa.net
大穴 自分の脳をささげる。

27 :デフォルトの名無しさん:2016/12/19(月) 21:40:29.23 ID:KFYYsgKs.net
入力音声を2chのスレに書き込む
レスがあったら音声出力する
処理時間→数分〜数日

28 :デフォルトの名無しさん:2016/12/19(月) 22:14:03.89 ID:TQpl8Z1r.net
お題:ペントミノパズルの12種類のピースを表示する

29 :デフォルトの名無しさん:2016/12/20(火) 00:24:55.91 ID:j+0pMEUE.net
>>28 bash
http://ideone.com/51ZjFV

端末でどうぞ

30 :デフォルトの名無しさん:2016/12/20(火) 23:33:43.23 ID:kU3OD1WB.net
お題:ペントミノパズルの全解を列挙する

31 : ◆QZschizo.iSO :2016/12/31(土) 18:31:14.25 ID:nvhfPpJn.net
瞬速年忘れ問題
2017は素数か?素数なら何番目か?

32 :デフォルトの名無しさん:2016/12/31(土) 19:13:55.05 ID:39E0aGGL.net
>>31
C
http://ideone.com/bUbh6M

33 :デフォルトの名無しさん:2016/12/31(土) 19:23:15.93 ID:39E0aGGL.net
たった今一部訂正

34 :デフォルトの名無しさん:2016/12/31(土) 22:12:03.44 ID:FA/ffAJp.net
>>31
同じくC
みなさん今年はお世話になりました
https://paiza.io/projects/Sio8sqlqmX547VHnZkpj_Q

35 :デフォルトの名無しさん:2016/12/31(土) 22:16:03.44 ID:oE9oDlU2.net
ああ、2017ぐらいなら何の工夫もせず力技でごり押ししても大丈夫なのか。
そりゃそうだなw

しかし俺は大みそかに何でこんなスレ覗いてるんだw

36 :デフォルトの名無しさん:2016/12/31(土) 22:56:06.40 ID:07cXWtfS.net
>>31 Emacs Lisp

(require 'cl-lib)

(defun a (x)
(cl-assert (and (integerp x) (> x 1)))
(let ((l (let ((max-lisp-eval-depth most-positive-fixnum)
(max-specpdl-size most-positive-fixnum))
(b (cl-loop for i from 2 to x collect i)))))
(when (= (car (last l)) x) (length l))))

(defun b (l)
(when l
(cons (car l) (b (cl-remove-if (lambda (x) (= (% x (car l)) 0)) (cdr l))))))

(mapcar (lambda (x) (cons x (a x))) '(2 3 4 5 2017))
((2 . 1) (3 . 2) (4) (5 . 3) (2017 . 306))

37 :デフォルトの名無しさん:2016/12/31(土) 23:05:34.75 ID:39E0aGGL.net
>>32で9999991(664579番目の素数)を計算させたら
14.708sかかった
>>34では0.391s
(Athron X2 1.8GHz)

38 :デフォルトの名無しさん:2017/01/01(日) 11:09:07.28 ID:IDYigKuX.net
>>31 Squeak/Pharo Smalltalk

(Integer primesUpTo: 2017+1) indexOf: 2017 "=> 306 "

39 :デフォルトの名無しさん:2017/01/01(日) 12:49:27.41 ID:AcyWguNB.net
>>31 ruby2.0.0
require 'prime'
def f9031(n)
Prime.prime?(n) && Prime.each_with_index.find {|p, _| p == n}
end
p f9031(4), f9031(2), f9031(2017)

false
[2, 0]
[2017, 305]

40 :デフォルトの名無しさん:2017/01/02(月) 13:41:39.67 ID:jOTKTTu2.net
>>31
Bash
seq 2017 | factor | awk '{happy = $1;new=0}NF==2{year++;new=1} END{
print happy,new?year "th":"no","prime"}'

41 :片山博文MZ ◆T6xkBnTXz7B0 :2017/01/02(月) 15:29:42.51 ID:h8C28I9Z.net
お題:与えられたクレジットカードの番号が正しいかどうか判定するプログラム。

42 :片山博文MZ ◆T6xkBnTXz7B0 :2017/01/02(月) 15:34:59.42 ID:h8C28I9Z.net
お題:個人情報(氏名、年齢、住所、電話番号)のダミーデータ(偽物のデータ)を大量に作成するプログラムを作れ。

43 :デフォルトの名無しさん:2017/01/02(月) 15:41:47.73 ID:h8C28I9Z.net
上げまして

44 :デフォルトの名無しさん:2017/01/02(月) 15:47:49.69 ID:jOTKTTu2.net
>>42
wget -r -l 0 URL で適当なサイトから住所のデータを持ってきてシャッフルしダミーデータを生成

45 :デフォルトの名無しさん:2017/01/02(月) 16:37:06.06 ID:4fVv1Ig/.net
正しいかどうかはクレカ会社でないとわからない
Luhnアルゴリズムの計算だけでいいならわりと楽だけど

46 :片山博文MZ ◆T6xkBnTXz7B0 :2017/01/02(月) 17:26:44.05 ID:h8C28I9Z.net
>>45
正論だね。じゃあルーンのアルゴリズムだけにしましょう。

47 :片山博文MZ ◆T6xkBnTXz7B0 :2017/01/02(月) 17:28:27.34 ID:h8C28I9Z.net
>>44
住所と電話番号の関係についてもっとよく考えて下さい。

48 :デフォルトの名無しさん:2017/01/02(月) 17:39:17.52 ID:fYcUjGFM.net
ダミーだから、実在する住所、通話可能な番号は許可しないんだよな?

49 :片山博文MZ ◆T6xkBnTXz7B0 :2017/01/02(月) 18:22:02.26 ID:h8C28I9Z.net
>>48
それはちょっと難しいだろう。実在するかどうやって確認するのかい?

50 :片山博文MZ ◆T6xkBnTXz7B0 :2017/01/02(月) 18:27:21.29 ID:h8C28I9Z.net
住所や電話番号の一部を伏せ字にすれば迷惑が掛からないとは思うが。。。
ダミーデータの用途によるんだよね。データベースのテスト用とか、●●●●用とか。

51 :片山博文MZ ◆T6xkBnTXz7B0 :2017/01/02(月) 18:48:13.86 ID:h8C28I9Z.net
ヒント:住所テキストデータの意味の定義は郵便局が行っている。

52 :デフォルトの名無しさん:2017/01/04(水) 12:00:38.64 ID:y99Ue0/2.net
お題:□に自然数を入れて式を成立させる。^は、べき乗。
□^□+□^□=2017

53 :デフォルトの名無しさん:2017/01/04(水) 12:44:11.80 ID:WgerS0Nf.net
回答無限個あるけどいいの?

54 :デフォルトの名無しさん:2017/01/04(水) 13:15:36.49 ID:UpZyqzsr.net
なにわろてんねん

55 :デフォルトの名無しさん:2017/01/04(水) 18:33:26.02 ID:dwq5ZcuT.net
http://ideone.com/BinLDz
C++??なんかへんな問題だな。

56 :片山博文MZ ◆T6xkBnTXz7B0 :2017/01/04(水) 18:36:48.39 ID:83LRywdJ.net
1^X+2016^1=2017
という形の解が無数に存在する。
これに対する制限がなければ面白くない。

57 :52:2017/01/04(水) 19:17:06.25 ID:y99Ue0/2.net
では□に入れるのを2以上の自然数に変更させてください

58 :デフォルトの名無しさん:2017/01/04(水) 23:01:48.48 ID:tU35eaIW.net
>>57
C++
自然数nがa^b (a>=2,b>=2)で表せるかどうか判定する関数(isFact)の
キャストが糞だから誰かいい書き方教えてください…
http://ideone.com/cB1Ncw

59 :デフォルトの名無しさん:2017/01/04(水) 23:17:21.92 ID:dwq5ZcuT.net
>>57
http://ideone.com/LbWS0f
C++。初期条件変更。
答えがバッサリ減った。

60 :デフォルトの名無しさん:2017/01/04(水) 23:37:14.33 ID:I0E1npQu.net
>>57
C
http://ideone.com/v5btSV

61 :デフォルトの名無しさん:2017/01/04(水) 23:41:45.40 ID:I0E1npQu.net
パフォーマンス改良
http://ideone.com/WDu0Ox

62 :デフォルトの名無しさん:2017/01/04(水) 23:54:08.76 ID:dwq5ZcuT.net
>>57
http://ideone.com/uxu5Bl
C++。頑張って最適化。これ以上はアルゴリズム変えないと大変。

63 :デフォルトの名無しさん:2017/01/05(木) 00:33:24.56 ID:FeYPKi0Y.net
>>57 Java
https://ideone.com/vRiP1c

64 :デフォルトの名無しさん:2017/01/05(木) 01:06:45.10 ID:FeYPKi0Y.net
>>57 Java
http://ideone.com/hNhAMC

>>63の ArrayList<String>[] list = new ArrayList[n]; がすんげぇメモリの無駄に思えたので

65 :デフォルトの名無しさん:2017/01/05(木) 01:50:33.81 ID:Tpy8rHG5.net
>>57
C++
Σ_i=1~m □^□ = nに拡張してみた
http://ideone.com/KKHS6o

66 :デフォルトの名無しさん:2017/01/05(木) 23:04:17.78 ID:Rf7MYzqE.net
>>57
http://ideone.com/9NAHBd

ひまつぶしにやってみた。C言語。

67 :デフォルトの名無しさん:2017/01/06(金) 07:39:18.38 ID:zAIptLYs.net
>>66
12^3 + 17^2 = 2017
17^2 + 12^3 = 2017
AB入れ替わりのこれはあるのに

3^4 + 44^2 = 2017
9^2 + 44^2 = 2017
がないのはなんでなんだぜ?

68 :デフォルトの名無しさん:2017/01/06(金) 08:14:14.51 ID:6Va7LE9a.net
ループ2が43で十分ってことになってるからかな??

69 :デフォルトの名無しさん:2017/01/06(金) 08:20:39.24 ID:FKYJf0Ai.net
全てのパターンを調べろっていうお題じゃないからな

70 :デフォルトの名無しさん:2017/01/06(金) 09:28:08.44 ID:0R4aMgJq.net
>>57 Squeak/Pharo Smalltalk

| ans |
ans := OrderedCollection new.
(2 to: 2017 sqrt) asDigitsToPower: 4 do: [:digits |
 ((digits first: 2) raisedTo: (digits last: 2)) sum = 2017
  ifTrue: [ans add: ('{1}^{3}+{2}^{4}' format: digits)]
].
^ans asStringWithCr

71 :デフォルトの名無しさん:2017/01/06(金) 21:50:55.39 ID:aMqQPymQ.net
>>66
コメントアウトで書いた議論をそのままコードにすればいいのに、そんなにマジックナンバー埋め込んでどうすんだ

72 :デフォルトの名無しさん:2017/01/08(日) 13:22:26.64 ID:XDbKIsfA.net
画面に並んでいるコントロールに左上から右下にタブ順を割り振るアルゴリズムってどう書きますか?
要するに矩形のソート方法です
綺麗に並んでるとは限りませんし重なっているものもあります
目で見た感覚的に正しくなるようにできますかね?

73 :デフォルトの名無しさん:2017/01/08(日) 14:54:28.66 ID:pxQLZoyB.net
>>72
(1) 重心の座標を求めて、
(2) Yでソート後Xでソート

とか。

まあ「感覚的に正しい」なんて要件はこの手の問題じゃ曖昧以外の何でもないし
どうやっても厳密な定義は無理だから、普通に左上座標で妥協してもいいんじゃないの

74 :デフォルトの名無しさん:2017/01/08(日) 15:11:05.77 ID:RSx5of9s.net
ひな形作った
http://jsbin.com/doniwanoqi/edit?html,output

75 :デフォルトの名無しさん:2017/01/08(日) 17:11:20.51 ID:8OGZNgRf.net
完全にスレチです。
JSスレでどうぞ。

76 :デフォルトの名無しさん:2017/01/08(日) 17:17:34.30 ID:5tCF0ZUl.net
>>72
要件が曖昧すぎる
どこを左上として、どこを右下とするのかで結果が変わるだろうし、あなたの感覚とズレが生じることもありえる
円形や一直線に並んでいる場合どこを左上とするのか、
画面右上、左下、右下を結んだ三角形(凾フような形)に並んでいるとき、左上と呼べるようなコントロールが存在しないけど、どうするのか

77 :デフォルトの名無しさん:2017/01/08(日) 17:27:52.04 ID:MJfiP+Ss.net
>>76
いや,逆にあいまいな要件のみ提示されていて,それに感覚的に適合する実装を示せ,というお題だろう
客は自分のしたいことを本当には知っていない

78 :デフォルトの名無しさん:2017/01/08(日) 17:33:29.68 ID:pxQLZoyB.net
>>77
こういう営業さんがいる会社は悲惨なことになるねw
冗談抜きで、IT業界の一部の労働環境が劣悪な理由はこういう安請け合いにある。

79 :デフォルトの名無しさん:2017/01/08(日) 17:43:37.92 ID:MJfiP+Ss.net
>>78
あいまいな要件に対して実装を提案することは問題ないだろう?
問題はその実装をちゃぶ台返しにされないようにすることだろう?
実装の提案ができないようだと仕事にならないのでは?

80 :デフォルトの名無しさん:2017/01/08(日) 17:54:48.18 ID:pxQLZoyB.net
>>79
甘いと思うよ。
それ、製品の完成後にこちらの提案が「やっぱり何か違う」ってちゃぶ台返しされるリスクを
考えてないでしょ。

まあスレ違いなんでこれ以上言わないけどさ。

81 :デフォルトの名無しさん:2017/01/08(日) 18:05:19.61 ID:RSx5of9s.net
これ業務じゃないんだし、自分だったらこう実装するっていうのを書いてけばええやん

82 :デフォルトの名無しさん:2017/01/08(日) 18:21:10.87 ID:XDbKIsfA.net
というかこの問題は順位付けに自明な定義がないところが肝です
AIなどでもそうですけど最近は曖昧かつ感覚に矛盾しない答えを求められる事が増えてきています

83 :デフォルトの名無しさん:2017/01/08(日) 22:47:32.14 ID:Qw43e7Zm.net
で、お題ってどれ?

84 :デフォルトの名無しさん:2017/01/08(日) 23:19:01.41 ID:4zEbWvNh.net
左上と右下を結ぶ線が垂直軸となる座標系に変換して
上から順に拾っていくだけじゃないの?

85 :デフォルトの名無しさん:2017/01/08(日) 23:27:18.98 ID:c38VN46k.net
>>84
それだと(0, 80は(100, 0)より順番が若くなる気がするんだけどw

86 :デフォルトの名無しさん:2017/01/09(月) 00:07:13.57 ID:g5s54bF4.net
>>85
何が言いたいのか分からないのでコメントしにくいけど
何かがおかしいとしたらあなたが考えた座標の変換が間違ってるんだろう

87 :デフォルトの名無しさん:2017/01/09(月) 00:12:11.15 ID:i1HzEaEu.net
>>86
自分で自分の言ってることが分かってないのかw

A = (0, 80)、 B = (100, 0)とすると、お前さんが言ってる座標兼では
AはBの「上」になる。

このぐらいのことそれを思いついた5秒後に分かれよほんと

88 :デフォルトの名無しさん:2017/01/09(月) 03:09:12.91 ID:iIb+NoJa.net
xでソートしてからyでソートだよな?
違う?

89 :デフォルトの名無しさん:2017/01/09(月) 07:01:36.39 ID:s6rymSkb.net
>>88
(10,0)と(5,10000)で後者が先に選択されるのはいやだ


次のような長方形からなる集合Uを定義する:
∀a∈ U に対し,∃b∈ a.center ⊂ b or b.center ⊂ aが成立する

90 :デフォルトの名無しさん:2017/01/09(月) 07:04:25.54 ID:s6rymSkb.net
ミスった

次のような長方形からなる集合Uを定義する:
∀a∈ U に対し,∃b∈Uが存在し a.center ⊂ b or b.center ⊂ aが成立する
要するに連結した長方形を集めるイメージですね
タグの順位付けしたい長方形の集合を連結した長方形の集合に分割してから

91 :デフォルトの名無しさん:2017/01/09(月) 07:08:49.40 ID:s6rymSkb.net
>>90
ほんとごめんなさい
集合Uの頂点を、最も原点(0,0)と集合Uの要素の長方形との距離が最も小さくなる長方形の左上の点と定義して
集合についてソートして、集合の要素についてxでソートするのが個人的に自然な感覚かなぁ

92 :デフォルトの名無しさん:2017/01/09(月) 07:18:31.23 ID:s6rymSkb.net
xでソートしたらこのケースでダメだ
http://i.imgur.com/wpFEt5J.png
集合の頂点を与える長方形を始点として中心点が近い要素順に並べるのが自然か

93 :デフォルトの名無しさん:2017/01/09(月) 08:02:40.56 ID:g5s54bF4.net
>>87
> A = (0, 80)、 B = (100, 0)とすると、お前さんが言ってる座標兼では
> AはBの「上」になる。

AがBの上になるのはあなたがそう変換した(あなたが決めた)からですよ

現在の座標系と左上と右下を結ぶ線が垂直軸となる座標系との関係性には
自由度があります(1秒で気付くと思いますが)
どのような関係にするかはお題の人間の感覚的に正しく見えるような関係を
選ぶべきであって、あなたが選択した関係に従って変換された座標に対して
あなたがおかしいと思ったというだけですね

94 :デフォルトの名無しさん:2017/01/09(月) 10:48:32.68 ID:LhlDvgN3.net
>>93
何が言いたいのかさっぱりわからんな。
数学弱いなら無理しなくていいのに

95 :デフォルトの名無しさん:2017/01/09(月) 10:56:12.03 ID:LhlDvgN3.net
ああ、きっと>>93の世界では平行移動で2点間の相対的な位置関係が
変わるんだろうねw

96 :デフォルトの名無しさん:2017/01/09(月) 17:11:09.51 ID:s0WlKSpH.net
>>57
http://echo.2ch.net/test/read.cgi/tech/1434079972/19
やっとコンパイルが通ったよ,ママ‥

97 :デフォルトの名無しさん:2017/01/09(月) 19:58:25.90 ID:nfkV3o0K.net
>>52 >>57 c
https://ideone.com/KhbiRx

98 :デフォルトの名無しさん:2017/01/10(火) 18:29:09.08 ID:Ilw6o9KC.net
>>94
自己紹介かな
座標軸の張り方次第だっつーのw

99 :デフォルトの名無しさん:2017/01/10(火) 20:42:50.38 ID:ld93epow.net
>>98
じゃあ具体的にどういう座標系ならBがAの上になるのか言ってみろ馬鹿。
重症だな

100 :デフォルトの名無しさん:2017/01/10(火) 21:08:13.05 ID:fM7Pfoze.net
キチガイ降臨age

101 :デフォルトの名無しさん:2017/01/10(火) 21:19:39.42 ID:NXUZz/0q.net
多分アマゾンギフトの人だよ

102 :デフォルトの名無しさん:2017/01/10(火) 21:52:46.22 ID:KKLCbaCP.net
煽って教えてもらうメソッドか
低能って嫌だな

103 :デフォルトの名無しさん:2017/01/10(火) 22:22:58.60 ID:EjxLrxqR.net
どっちかっていうと消えてくれた方がありがたい

104 :デフォルトの名無しさん:2017/01/10(火) 22:34:23.43 ID:U9oYccVU.net
>>102
誰に物をいってるんだ?
身の程をわきまえろ

105 :デフォルトの名無しさん:2017/01/10(火) 22:36:50.29 ID:U9oYccVU.net
>>99
煽って教えてもらうメソッドか
低能って嫌だな

106 :デフォルトの名無しさん:2017/01/11(水) 12:22:04.85 ID:gkYh8YCT.net
教えるも糞も最初から正解は書いてある>>87

107 :デフォルトの名無しさん:2017/01/11(水) 14:49:58.96 ID:iTAjJX4H.net
>>104-105
uyか?Qか?

108 :デフォルトの名無しさん:2017/01/11(水) 19:39:31.27 ID:J0kepQ6p.net
QZ=>>96

109 :デフォルトの名無しさん:2017/01/11(水) 21:08:00.35 ID:0XwQtfyu.net
QZ怒りのマンガでわかる心理学

110 :デフォルトの名無しさん:2017/01/11(水) 21:11:32.29 ID:NdPILmY+.net
>>86
おい!お前のせいでキチガイが増殖してるじゃねえか
責任とって全部持って帰れ

111 :デフォルトの名無しさん:2017/01/11(水) 21:13:18.42 ID:NdPILmY+.net
>>106
いや何を書いたとかじゃなくてさ、
>>99
煽って教えてもらうメソッドか
低能って嫌だな

112 :デフォルトの名無しさん:2017/01/11(水) 21:14:22.15 ID:NdPILmY+.net
心理学怒りのマンガでわかるQZ

113 :デフォルトの名無しさん:2017/01/11(水) 21:18:25.47 ID:NdPILmY+.net
頭の中がQZでいっぱいな心理学おじさんはQ呼称で自分だけは特別な存在アピール

114 :デフォルトの名無しさん:2017/01/11(水) 23:33:02.72 ID:IQVyJq11.net
感覚に頼るような変なお題出すからキチガイが湧くんだよ

115 :デフォルトの名無しさん:2017/01/12(木) 00:14:22.68 ID:OC4R5S2o.net
IDが出るようになったのになぜQはコテを消したのか?
とんだチキンだったのではないだろうか?

116 :デフォルトの名無しさん:2017/01/12(木) 07:39:01.18 ID:sAp1jMS4.net
心理学怒涛の寝ても覚めてもQZ

117 :デフォルトの名無しさん:2017/01/12(木) 13:04:44.00 ID:OdpRjvqI.net
>>16
ようQ

118 :デフォルトの名無しさん:2017/01/12(木) 13:06:09.71 ID:OdpRjvqI.net
×>>16
>>116
似ても焼いても一つも人の役に立てないQ

119 :デフォルトの名無しさん:2017/01/12(木) 21:59:23.34 ID:yV1HOyPB.net
心理学最終手段の居直りQ連呼

120 :デフォルトの名無しさん:2017/01/12(木) 22:00:27.56 ID:yV1HOyPB.net
心理学おじさんとQZを同じ部屋にとじこめたらどうなるの?

121 :デフォルトの名無しさん:2017/01/12(木) 22:38:03.38 ID:KUw+Fq2e.net
ラップでも始めるんじゃね。

122 :デフォルトの名無しさん:2017/01/13(金) 18:15:01.00 ID:hu5eqRYQ.net
QZ=>>96

123 :デフォルトの名無しさん:2017/01/13(金) 21:36:42.18 ID:A97spPPP.net
お題:平面上に四つの点を二点間の距離が二種類になるように配置する

124 :デフォルトの名無しさん:2017/01/13(金) 23:31:20.40 ID:Z31HhTHu.net
正三角形+中心
一辺を共有する二つの正三角形
正方形

他にあるかな?

125 :デフォルトの名無しさん:2017/01/14(土) 03:29:31.67 ID:OgQ0DzZw.net
異なる辺の長さa,bに対して
1)a:5本, b:1本
正三角形(辺の長さa)×2の菱形のみ
2)a:4本, b:2本
2辺と対角線の長さがaの凧型
正方形(辺の長さa)
3)a:3本, b:3本
正三角形と重心

3)はまだまだありそう

126 :デフォルトの名無しさん:2017/01/18(水) 21:37:38.36 ID:6wTe0sJ+.net
sage>>123
 ●

 ●

● ●
ーーー
  ●


●   ●
  ●
ーーー
 ●  ●

●    ●
ーーー
● ●

● ●
ーーー
  ●

  ●

●   ●
ーーー
  ●   ●



●   ●

127 :デフォルトの名無しさん:2017/01/18(水) 22:19:10.68 ID:vpMv+OF/.net
>>126
座標で書いてくれw

128 :デフォルトの名無しさん:2017/01/18(水) 22:50:29.86 ID:nuz5iQOv.net
プログラムで書いてくれ

129 :デフォルトの名無しさん:2017/01/19(木) 12:26:45.56 ID:ZgJp4HEV.net
>>123
平面上に3点P1、P2、P3を配置して三角形を作る時、正三角形を含む二等辺三角形でなければならない
これ以外の配置のやり方をすると辺の長さが3種類となって前提に反する

上記のように配置した3点3点P1、P2、P3に4点目P4を配置するとき、
A) 二等辺三角形P1P2P3と底辺を共有し、長さが等しい辺と同じ長さの2辺をもつ2等辺三角形となるようにP4を配置する
(底辺を共有するように二等辺三角形二つを配置する)
または
B) 三角形P1P2P3が正三角形のとき、外接円の中心と一致するようにP4を配置する

の二通りの配置の仕方がある。逆に言えば、この条件に従うように座標を決定すれば無限の配置の仕方が可能

130 :デフォルトの名無しさん:2017/01/19(木) 13:43:42.27 ID:ff1dOA50.net
>>123 Javascript
http://codepen.io/anon/pen/mRRxXx?editors=1011

観測的手法で 分解能に依存するけど網羅できるはず

131 :デフォルトの名無しさん:2017/01/19(木) 14:02:27.30 ID:ff1dOA50.net
って、>>123よく読んだら>>130じゃダメだ! orz
失敬 撤回しますわ

132 :デフォルトの名無しさん:2017/02/21(火) 19:03:19.21 ID:vHI0u4yb.net
お題:ある数 n が与えられたとして、n の原始根の位数を求めよ
条件:原始根については https://ja.wikisource.org/wiki/%E5%88%9D%E7%AD%89%E6%95%B4%E6%95%B0%E8%AB%96%E8%AC%9B%E7%BE%A9/%E7%AC%AC1%E7%AB%A0/%E5%8E%9F%E5%A7%8B%E6%A0%B9%EF%BC%8C%E6%8C%87%E6%95%B0
を参照せよ、ただし、このテキストでは n が素数であることを前提にしているが、原始根は n が素数でなくても存在する。n が素数でない場合にも対応せよ。
原始根を、小さい数から順次位数を力技で求める方法は認めない。
n の原始根が存在しない場合にアルゴリズムが停止する必要は要請しない。

133 :デフォルトの名無しさん:2017/03/02(木) 16:39:35.38 ID:TV66ybVQ.net
なんでお前偉そうなの
宿題は自分でやりな

134 :デフォルトの名無しさん:2017/03/03(金) 09:55:45.90 ID:SUJ1/PZz.net
C言語スレよりロンダリング

x面体のサイコロy個を振って得られる出目のヒストグラムを出力するプログラムを作ろう
dice.exe 6 3 (6面体のサイコロ3個) の出力例
出目 出現回数 出現率
---- -------- ------
  3    1  0.46
  4    3  1.39
  5    6  2.78
  6    10  4.63
  7    15  6.94
  8    21  9.72
  9    25 11.57
 10    27 12.50
 11    27 12.50
 12    25 11.57
 13    21  9.72
 14    15  6.94
 15    10  4.63
 16    6  2.78
 17    3  1.39
 18    1  0.46

135 :デフォルトの名無しさん:2017/03/03(金) 18:25:27.76 ID:P2edee8O.net
厳密解を出す場合なら動的計画法で素朴にやるとO(x^2y^2)

136 :デフォルトの名無しさん:2017/03/03(金) 20:41:14.23 ID:yCeb1kZ1.net
>>134 Java
https://ideone.com/mVatvl

137 :デフォルトの名無しさん:2017/03/03(金) 21:29:12.11 ID:SEBfdj3K.net
この問題のポイントはサイコロを正N面体に限定してないところだな
それと3面体なんていう指定をされたらどうするとかな

138 :デフォルトの名無しさん:2017/03/04(土) 01:08:57.14 ID:eNSOp3uJ.net
>>136
やるなあ
パスカルの三角形のもう一段階上みたいな感じか

139 :デフォルトの名無しさん:2017/03/04(土) 04:55:45.63 ID:VvbFezkO.net
>>134 C
https://ideone.com/FmyEy7

140 :デフォルトの名無しさん:2017/03/05(日) 03:05:26.21 ID:FnE3e1tn.net
お題: ハート・ダイヤ・クラブ・スペードを各13枚ずつもつ52枚のカードから13枚のカードが配られます.

Q1. 配られたカードの種類がもっとも均質でない(13・0・0・0)確率P1と, もっとも均質である(4・3・3・3)確率P2を求めてください.
Q2. 確率P1よりも低い確率の組み合わせが存在する場合, その組み合わせと確率を求めてください.
Q3. 確率P2よりも高い確率の組み合わせが存在する場合, その組み合わせと確率を求めてください.

141 :デフォルトの名無しさん:2017/03/05(日) 07:48:05.16 ID:KWf32tSz.net
宿題は自分でやろう

142 :デフォルトの名無しさん:2017/03/05(日) 09:20:54.74 ID:FnE3e1tn.net
>>134 Emacs Lisp

(require 'cl-lib)

(defun dice (x y)
(let ((b '(0)))
(dotimes (i y)
(setq b (reduce (lambda (a b) (append a b)) (loop for n in (loop for i from 1 to x collect i) collect (loop for m in b collect (+ n m))))))
(pp (loop for n in (remove-duplicates b) collect (list n (count n b) (/ (float (count n b)) (length b)))))))

(dice 6 3)
((3 1 0.004629629629629629)
(4 3 0.013888888888888888)
(5 6 0.027777777777777776)
(6 10 0.046296296296296294)
(7 15 0.06944444444444445)
(8 21 0.09722222222222222)
(9 25 0.11574074074074074)
(10 27 0.125)
(11 27 0.125)
(12 25 0.11574074074074074)
(13 21 0.09722222222222222)
(14 15 0.06944444444444445)
(15 10 0.046296296296296294)
(16 6 0.027777777777777776)
(17 3 0.013888888888888888)
(18 1 0.004629629629629629))

143 :デフォルトの名無しさん:2017/03/05(日) 11:59:20.94 ID:h/zPOUa4.net
>>134 Squeak/Pharo Smalltalk

| dice |
dice := [:x :y |
 | bag |
 bag := Bag new.
 (1 to: x) asDigitsToPower: y do: [:ary | bag add: ary sum].
 bag sortedElements collect: [:kv | {kv key. kv value. kv value / bag size * 100s2}]
].

dice value: 6 value: 3

"=> {
 {3 . 1 . 0.46s2}.
 {4 . 3 . 1.38s2}.
 {5 . 6 . 2.77s2}.
 {6 . 10 . 4.62s2}.
 {7 . 15 . 6.94s2}.
 {8 . 21 . 9.72s2}.
 {9 . 25 . 11.57s2}.
 {10 27 12.50s2}.
 {11 27 12.50s2}.
 {12 . 25 . 11.57s2}.
 {13 . 21 . 9.72s2}.
 {14 . 15 . 6.94s2}.
 {15 . 10 . 4.62s2}.
 {16 . 6 . 2.77s2}.
 {17 . 3 . 1.38s2}.
 {18 . 1 . 0.46s2}
} "

144 :デフォルトの名無しさん:2017/03/05(日) 15:38:00.05 ID:Z6iNRFEa.net
プログラミングする意味のないお題ばっか

145 :デフォルトの名無しさん:2017/03/05(日) 17:57:56.52 ID:I6Dgph4B.net
>>144
プログラミングする意味のあるお題作ってくれよ

146 :デフォルトの名無しさん:2017/03/05(日) 18:17:30.28 ID:WuF+gNTE.net
>>134 ocaml
https://ideone.com/SWZRyZ

147 :デフォルトの名無しさん:2017/03/05(日) 20:12:40.81 ID:Oe+PHv4r.net
いや>>134は10面体10個を10秒以内で計算するとかの条件付けるとプログラミングの意味あるぞ

>>136はよく出来てるとオモタ

148 :デフォルトの名無しさん:2017/03/07(火) 18:28:24.63 ID:sN5zB8qN.net
>>144
そんなこといわずに >>132 を解いてくれよぅ
原始根の存在定理を素数以外に拡張していいのかどうか、いまいち確信がもてないんだ…

149 :140:2017/03/12(日) 13:40:16.63 ID:VxOAsp1C.net
乱数を使用して10億回試行してみたところ,4・3・3・3の出現回数は105357474 (10.5%), 13・0・0・0は0(12・1・0・0は3)でした.
4・3・3・3よりも出現回数の多い組み合わせは以下の4通りでした.

4・4・3・2 215505541回 (21.6%)
5・3・3・2 155167267回 (15.5%)
5・4・3・1 129327030回 (12.9%)
5・4・2・2 105786216回 (10.6%)

このお題はHAKMEM MIT AI Memo 239 ITEM 46 (Schroeppel)を参考にしました.
http://www.inwap.com/pdp10/hbaker/hakmem/number.html#item46

150 :デフォルトの名無しさん:2017/03/24(金) 16:34:29.50 ID:krVzShuL.net
A+B+C+D+E+F+G+H+I+J+K=170
A-B=−2
A>B>C>D>E>F>G>H>I>J>K
A>0のとき
この式を満たすA〜Kを全て求めよ

151 :デフォルトの名無しさん:2017/03/24(金) 16:44:02.34 ID:IzMGTasH.net
A-B=−2
A>B
この二つを同時に満たせません

152 :デフォルトの名無しさん:2017/03/24(金) 19:36:54.08 ID:krVzShuL.net
済みません間違えてました
A-B=2でした

153 :デフォルトの名無しさん:2017/03/24(金) 19:49:23.03 ID:3IaHiZWx.net
そっちを直すのか

154 :デフォルトの名無しさん:2017/03/24(金) 19:50:50.29 ID:IzMGTasH.net
修正がA>B>C...→A<B<C...じゃなくてA-B=-2→A-B=2だと無数に求められるから全て求めるのは無理じゃないかな?

155 :デフォルトの名無しさん:2017/03/24(金) 19:57:24.58 ID:IzMGTasH.net
例えばこんなん
int A=68,B=66,C=8,D=7,E=6,F=5,G=4,H=3,I=2,J=1,K=0;
while(true) {
A++; B++; K-=2;
}

156 :デフォルトの名無しさん:2017/03/24(金) 22:00:38.98 ID:DBVfNEjn.net
こういうお題を書きたいのなら制約条件を良く考えないと

A+B+C+D+E+F+G+H+I+J+K=170
A>B>C>D>E>F>G>H>I>J>K
A〜Kは全て自然数とする

こういうのとかな

157 :デフォルトの名無しさん:2017/03/25(土) 07:43:46.30 ID:LJUF46o1.net
>>150
http://ideone.com/ZXck7T
C++。ちょっと自信ないけど、一応できた。
頑張って最適化した。

158 :デフォルトの名無しさん:2017/03/25(土) 07:59:35.07 ID:LJUF46o1.net
あれ?条件おかしいな。俺ってせっかちさん!

159 :デフォルトの名無しさん:2017/03/25(土) 08:00:35.83 ID:LJUF46o1.net
と思ったら専ブラの安価化けてた。

160 :デフォルトの名無しさん:2017/03/25(土) 13:31:35.87 ID:wrPCcs4m.net
https://www.hackerrank.com/sinapusu2002-1
ハッカーランクという競技プログラミングサイトで出題中の問題。
オリジナル問題6問。
ハッカーランクに登録して解いてくれ。
スレ違いだろうか?
2つの三角形の共有面積はかなりマゾいのでお勧め。
一応競技プログラミング界の重鎮で東大で数学してる人にも解いてもらったりはしたけど。
挑戦者数が増えないのが悩みの種。
ただいま次回コンテストに向けて3/6問作成済み、3問作成予定。

161 :デフォルトの名無しさん:2017/03/25(土) 13:58:45.28 ID:LJUF46o1.net
レジストするのめんどいのと頭悪いので無理〜〜。

162 :デフォルトの名無しさん:2017/03/25(土) 14:01:32.89 ID:P+Eymz64.net
挑戦者数が増えないのは問題が悪いからだろ。
良問をだしてれば自然と挑戦者は増えるはず。
宣伝してズルした時点でお前の人間性の低さがわかるんだよ

163 :デフォルトの名無しさん:2017/03/25(土) 14:02:58.32 ID:LJUF46o1.net
んんん?どうした?

164 :デフォルトの名無しさん:2017/03/25(土) 14:13:46.85 ID:LJUF46o1.net
頭悪いって俺のこと行ってるんだが。すまんな!

165 :デフォルトの名無しさん:2017/03/25(土) 15:30:57.12 ID:P+Eymz64.net
>>160
東大生に解いてもらっても屑な問題はクズだ。
そんなもので東大のブランド志向に思ってる奴を釣ろうとしている
お前も同じ穴の貉だよ。
俺はそいういうやつは軽蔑するな。

166 :デフォルトの名無しさん:2017/03/25(土) 18:47:23.24 ID:OqJqFalA.net
まあ完全にスレチだから無視すれば?

167 :デフォルトの名無しさん:2017/03/28(火) 09:57:58.96 ID:084k/T9f.net
お題:格子点を1個だけ含む面積4の凸多角形を求める

168 :デフォルトの名無しさん:2017/03/28(火) 13:00:05.06 ID:qGSQyMpt.net
>>167
斜めってる正方形とか、無限にできるんでないの?

169 :デフォルトの名無しさん:2017/03/29(水) 00:52:34.65 ID:IYsCUDjz.net
>>167
xが0.9未満なら
(0,0)(x,0)(0.1,4/x)(0.1+x,4/x)
の平行四辺形でいいよね

170 :デフォルトの名無しさん:2017/04/01(土) 19:39:48.39 ID:bpxrLk3k.net
>>167
(0,0)-(0.8,0)-(9.8,10)

171 : ◆QZaw55cn4c :2017/04/08(土) 18:54:06.32 ID:O01wSTFq.net
[1] 授業単元:線形代数

[2] 問題文
与えられたn字正方行列の逆行列を求めよ。
求めた行列は、与えられた行列を掛けて n 次単位行列となることを確認せよ。
以下のCプログラムを元に実装されていない部分を補完せよ。C 以外の言語で記述する場合は、この C プラグラムの対応する部分も記述すること。
https://ideone.com/k996I1
逆行列が存在しない場合は適切に処理してよい。

[3] 環境
 [3.1] OS: 問わない
 [3.2] コンパイラ名とバージョン: 問わない
 [3.3] 言語: C または任意の言語

[4] 期限: ([2017年4月15日23:59まで]
[5] その他の制限:お題です。

172 :デフォルトの名無しさん:2017/04/08(土) 21:30:20.69 ID:n0eUgEex.net
QZのひとか。ここは宿題スレじゃないよ。

173 :デフォルトの名無しさん:2017/04/08(土) 21:43:02.82 ID:gcfXcmks.net
お題ですぅ‥

174 :デフォルトの名無しさん:2017/04/09(日) 20:50:45.75 ID:uOnZBcD2.net
お題:
辺の長さが100より小さい既約のピタゴラス三角形をすべて求めう。
出力の順序は問わない。

175 :デフォルトの名無しさん:2017/04/09(日) 23:51:51.14 ID:eG+aks/D.net
>>171
これ、宿題の解答にならないようなプログラムにするのは結構難問かも。

176 :デフォルトの名無しさん:2017/04/10(月) 23:26:39.47 ID:9Aj0xnIY.net
>>174 Java
https://ideone.com/g3K6Z8

177 :デフォルトの名無しさん:2017/04/11(火) 01:24:52.99 ID:gtlUVgn4.net
>>174
Ruby
N=100
(1..N).step(2){|a|(a+2..N/a).step(2){|b|c=b*b-a*a>>1;a.gcd(b)<2&&c<N&&p([a*b,c,c+a*a])}}

178 :デフォルトの名無しさん:2017/04/11(火) 21:44:43.48 ID:MO2JwS29.net
お題:フランス語で1から100まで数える

179 :デフォルトの名無しさん:2017/04/11(火) 21:50:23.26 ID:LjaFq8Gx.net
1(フランス語 ・・・ 100(フランス語

180 :デフォルトの名無しさん:2017/04/11(火) 22:35:16.36 ID:gMY8o6LW.net
お題:
トランプの札のリストが文字列で与えられるので
同じマークor同じ数字のペアが最大何組できるか求めよ

D:ダイヤ、H:ハート、S:スペード、C:クラブ、
A:エース、2〜9:数字の2〜9、T:10、J:ジャック、Q:クイーン、K:キング

例:
"DAD2HAH3" -> 2
"DAD3D8D9DJH5H8HKSASKCAC5C8CK" -> 7
"SAS2S3S4S5S6S7S8S9C2C3C4C5C6C7C8C9CT" -> 9

181 :デフォルトの名無しさん:2017/04/13(木) 22:19:39.85 ID:S/dW7TnL.net
>>180
知らんけどNP完全とかそういうやつ?
違ったらスマソ

182 :デフォルトの名無しさん:2017/04/14(金) 01:20:45.82 ID:xslPbFLu.net
>>180
http://ideone.com/UgHgtz
C++。多分間違ってる。数字も合わないしな。
i7-6700でリリースビルドで40秒くらいかかる。

見てる感じ塩基配列だよなこれ。

183 :デフォルトの名無しさん:2017/04/14(金) 02:38:04.01 ID:/pg6QsRy.net
>>180
c++ http://ideone.com/WK9LQi

問題文には明記なかったが、ユニークな52枚(4*13)1セットだけだとして解いた。

計算量は 1問 14^4*6 くらいになるのだろう。
(最初は14^5*6でやったが、ひとつ計算量落とした。
この数値じゃ、意味なかったけど)

184 :デフォルトの名無しさん:2017/04/14(金) 06:35:41.46 ID:qDe/EUun.net
動的計画法はよくわからん。
書けもしない。Orz

185 : ◆QZaw55cn4c :2017/04/14(金) 18:11:30.73 ID:UkhykSdt.net
>>171 人気がないね…ループの知識だけあればOKでループの構成を考える腕力が多少あればそれなりに楽しめるお題と思ったんだけれども
新規性がないのが弱点なのかな?
次のお題を考えることにしようか

186 :デフォルトの名無しさん:2017/04/14(金) 22:18:00.95 ID:qDe/EUun.net
http://ideone.com/Et54St
意味ないけど最適化したら現実的な時間で終わるようになった。
今暇なので、いい暇つぶしになったなぁ。意味ないけど。

187 :デフォルトの名無しさん:2017/04/15(土) 10:54:03.85 ID:tThpfxds.net
>>186
>DAD2HAH3:4,2
>DAD3D8D9DJH5H8HKSASKCAC5C8CK:394,38
>SAS2S3S4S5S6S7S8S9C2C3C4C5C6C7C8C9CT:1972800,16

どういうこと???
組み合わせの数がカードの枚数すら超えてるの???

188 :デフォルトの名無しさん:2017/04/15(土) 11:05:02.69 ID:eiwfOO/l.net
そいつのコード見てないから分からんが多分、最初の例なら
1)DAHA
2)DAD2
3)HAH3
4)DAD2,HAH3
みたいにカウントされちゃってるんじゃないか?

189 :デフォルトの名無しさん:2017/04/15(土) 17:04:06.01 ID:gqnlgfq4.net
>>180
最後のは9じゃなくて8組だろ

同じマーク S:4組、C:4組、合計8組
同じ数字 2,3,4,5,6,7,8,9の各ペアで合計8組
最大組数は8組

190 :デフォルトの名無しさん:2017/04/15(土) 17:17:26.54 ID:gqnlgfq4.net
>>180 C
http://ideone.com/vD1JHy

191 :デフォルトの名無しさん:2017/04/15(土) 17:40:29.91 ID:gqnlgfq4.net
俺も間違ってたので修正した
http://ideone.com/vD1JHy

答えは2組,6組,8組じゃないの?

192 :デフォルトの名無しさん:2017/04/15(土) 19:29:06.55 ID:BbAH7YT3.net
>>189
反例

数字で(S2=C2)で1ペア、
残り8枚ずつで同スートのペアを8つくる
合計 9ペア

数字で(S2=C2, S3=C3, S4=C4) 3ペア
残りは6枚ずつになるので、6ペアできる
合計9ペア

その他 数字のペアを奇数個つくれれば、9を作れる

193 :デフォルトの名無しさん:2017/04/15(土) 20:24:00.89 ID:gqnlgfq4.net
>>192
意味がわかった

194 :デフォルトの名無しさん:2017/04/15(土) 21:12:03.91 ID:v99qvGb2.net
8bitの整数を一つ引数として与え、
上位4bitと下位4bit を符号なし整数として掛け算した結果の8bitを返す関数を
論理演算子のみで実装したもののうち、
もっとも実行時間が短いものを挙げよ。
論理演算は全て1命令1サイクルとする。

195 :デフォルトの名無しさん:2017/04/15(土) 21:23:09.15 ID:v99qvGb2.net
表引きで1クロックとかは無しね。

196 :デフォルトの名無しさん:2017/04/15(土) 22:10:36.10 ID:fG6QrlNX.net
最も実行時間が短いかどうかの判定ってNP完全っぽそう

197 :デフォルトの名無しさん:2017/04/15(土) 23:11:44.46 ID:v99qvGb2.net
8bitなら力技で解けないかな?

198 :デフォルトの名無しさん:2017/04/16(日) 03:52:44.93 ID:T2fHNlZE.net
http://ideone.com/QNbwWa
C++。えーん終わらないよ〜。
デバッグ覗いたら早い段階で答え出てるんだけど打ち切り方がわからない。

>>186などは題意を把握してなかった。

199 :デフォルトの名無しさん:2017/04/16(日) 05:47:25.03 ID:T2fHNlZE.net
ふえぇーん。1時間半回してもおわらないよー。
動的計画法すごい。

200 :デフォルトの名無しさん:2017/04/16(日) 08:35:26.85 ID:0couv4LD.net
>>180 Java
http://ideone.com/ABj7QB

201 :デフォルトの名無しさん:2017/04/16(日) 14:11:29.20 ID:oOWZ5kyJ.net
お題
これ
http://www.mext.go.jp/programin/
に相当するアプリケーションを
非Web環境でも使えるような仕様で
単一の言語でコーディングしろ

202 :デフォルトの名無しさん:2017/04/16(日) 14:13:03.29 ID:aobdGNjh.net
>>180 C
http://ideone.com/CH5bY4

203 :デフォルトの名無しさん:2017/04/16(日) 16:14:29.58 ID:z1TgtbB7.net
>>201 Squeak Smalltalk で
https://scratch.mit.edu/scratch_1.4/

204 :デフォルトの名無しさん:2017/04/20(木) 11:21:43.08 ID:QJAwY2Mb.net
>>201 Pharo Smalltalk のもあった
http://www.phratch.com/

205 :デフォルトの名無しさん:2017/04/22(土) 15:45:58.46 ID:Btp73K0S.net
こういうのに瞬時に回答できるようになるまでどのくらいの時間がかかるものなの?

206 :デフォルトの名無しさん:2017/04/22(土) 21:31:15.12 ID:DL7Mzdas.net
お題: くまモンをかく

207 :デフォルトの名無しさん:2017/04/23(日) 10:45:10.81 ID:33igmMUW.net
くまモン

208 :デフォルトの名無しさん:2017/04/23(日) 20:01:13.92 ID:fsvPYIZO.net
>>205
一瞬

209 :デフォルトの名無しさん:2017/04/23(日) 20:24:02.53 ID:nc0/ghBL.net
とんちかよw

210 :デフォルトの名無しさん:2017/04/23(日) 20:37:32.86 ID:7bZTt8wr.net
ああ、やっと意味が分かったw

211 :デフォルトの名無しさん:2017/04/24(月) 05:27:50.71 ID:+7Sa1g5s.net
>>206
http://light.dotup.org/uploda/light.dotup.org437888.png
http://ideone.com/eJ2xul
C++。あー大変だった。
以前ラウム螺旋作った時の流用したけど、タートルグラフィックはGUI無いとキツイなぁ。
似てないとか苦情は受け付けません。あしからず。

212 :sage:2017/04/24(月) 05:36:47.92 ID:4BlFLqQl.net
>>206 glsl
http://glslsandbox.com/e#40107.0

似てないなぁ……

>>211 GJ! 先越されたー

213 :デフォルトの名無しさん:2017/04/24(月) 05:43:14.19 ID:+7Sa1g5s.net
>>212
時代は3Dか・・・。
ロートルにはできん芸当だ。GJ。

214 :デフォルトの名無しさん:2017/04/24(月) 11:57:42.03 ID:SbFm/aQk.net
>>212
へーこんなところあるんだ知らんかった

215 :デフォルトの名無しさん:2017/04/24(月) 21:55:43.67 ID:WHcUZH3G.net
すみません。何言ってるかわからないだろうけど、
12桁の数字が並ぶ羅列は何を表しているのでしょうか???
/021/000/000/000
とか

216 :デフォルトの名無しさん:2017/04/24(月) 22:16:03.57 ID:9NN+cmWY.net
最初の3つが市外局番 次が区番号 残りが個人識別子だよ

217 :デフォルトの名無しさん:2017/04/25(火) 05:30:26.30 ID:8YHxJHtd.net
12桁くらいの数字を使う規格ってそれなりにありそうだけどな。

218 :デフォルトの名無しさん:2017/04/25(火) 23:23:31.83 ID:bgy6YsrD.net
お題:自然数nのヤング図形をもとめる

219 :デフォルトの名無しさん:2017/04/26(水) 07:01:54.95 ID:ezAhxtxU.net
>>218
ギブアップ。うぃきぺ見ても意味わかんない。頭爆発しそう。

220 :デフォルトの名無しさん:2017/04/26(水) 07:24:25.57 ID:5DpYjYxD.net
数字の入ったヤング盤じゃなくて、図形だけでいいの?
nのヤング図形を全部求めるってこと?

221 :デフォルトの名無しさん:2017/04/26(水) 12:59:54.54 ID:BGeLfl3x.net
>>218 Nim
https://wandbox.org/permlink/cNfdZ0908tcbQhod

222 :221:2017/04/26(水) 16:03:33.34 ID:BGeLfl3x.net
>>221は1行目が「× Yaund」「○ Young」という恥ずかしい間違いから始まり
N <= 0の時のチェック漏れやら
長方形のパターン(N=10の時の[5,5]や[2,2,2,2,2])が抜けてるなど
ミスしまくってました orz てことで、修正します

>>218 Nim
https://wandbox.org/permlink/RUf5B0SVg0MozD25

223 :デフォルトの名無しさん:2017/04/26(水) 16:07:13.33 ID:ezAhxtxU.net
なーんだそれでいいのか。
それならなんとか組めるかも。

224 :デフォルトの名無しさん:2017/04/26(水) 19:19:14.42 ID:ezAhxtxU.net
>>218
http://ideone.com/6PRt5z
C++。こういうこと?やっぱりよくわからん。

225 :デフォルトの名無しさん:2017/04/27(木) 10:01:02.40 ID:LnKeLTZP.net
問題の想定解と合ってるのかな?
出題者のコメントが欲しいところ

226 :デフォルトの名無しさん:2017/05/05(金) 08:45:57.67 ID:f8yfM1E5.net
素人の言語処理100本ノック:まとめ
http://qiita.com/segavvy/items/fb50ba8097d59475f760

227 :デフォルトの名無しさん:2017/05/24(水) 21:48:00.75 ID:7oyfe1wq.net
お題:以下を出力
857142
714285
571428
428571
285714
142857

228 :デフォルトの名無しさん:2017/05/24(水) 22:07:09.20 ID:L9mU+g8W.net
>>227
とりあえずPythonでやってみた
https://ideone.com/UCpMVm

229 :デフォルトの名無しさん:2017/05/24(水) 22:19:15.30 ID:lEJcnLZT.net
>>227
法則性がわからん
文字列じゃなくあくまで数値で?

230 :デフォルトの名無しさん:2017/05/24(水) 23:21:53.31 ID:FC0+qCJl.net
>>229
一番下の数字に1∼6を掛ければ

231 :デフォルトの名無しさん:2017/05/24(水) 23:33:01.85 ID:0ssPtL9D.net
>>227
C
http://ideone.com/eRLqLi

232 :デフォルトの名無しさん:2017/05/24(水) 23:48:35.88 ID:99+UTRWu.net
各々n/7の循環部だね

233 :デフォルトの名無しさん:2017/05/24(水) 23:51:10.59 ID:99+UTRWu.net
>>227
Ruby
6.downto(1){|i|p i*1000000/7}

234 :デフォルトの名無しさん:2017/05/25(木) 01:00:30.19 ID:vvyNjvHe.net
>>227 Perl

use 5.016;
use integer;
say 857142 / $_ for 1..6;

235 :デフォルトの名無しさん:2017/05/25(木) 01:16:57.10 ID:vvyNjvHe.net
>>234 間違えてたスマソ
Perl

use 5.016;
say 142857 * (7 - $_) for 1..6;

$ perl 9_227.pl
857142
714285
571428
428571
285714
142857

236 :デフォルトの名無しさん:2017/05/25(木) 13:01:14.56 ID:+8TeyhTL.net
>>227 Squeak/Pharo Smalltalk

(142857 * (6 to: 1 by: -1)) asStringWithCr

=> '857142
714285
571428
428571
285714
142857'

237 :デフォルトの名無しさん:2017/05/25(木) 13:21:29.71 ID:+8TeyhTL.net
>>227 GNU Smalltalk

6 to: 1 by: -1 do: [:i | (142857 * i) displayNl]

http://ideone.com/yNgDeB

238 :デフォルトの名無しさん:2017/05/25(木) 18:49:32.83 ID:F0MQej0l.net
>>227 common lisp
(do ((i 6 (- i 1))) ((equal i 0)) (print (floor (/ (* 1000000 i) 7))))

239 :デフォルトの名無しさん:2017/05/25(木) 19:28:46.12 ID:MmhezNib.net
お題が情報不足過ぎて何を使っていいのか悪いのかがわかんない

240 :デフォルトの名無しさん:2017/05/25(木) 21:35:24.87 ID:Z+pf95Ly.net
>>227
@Mathematica

In[1]:=142857//
  IntegerDigits//
  NestList[RotateLeft,#,Length[#]-1]&//
  Map[FromDigits,#]&//
  Sort//
  Reverse//
  Map[{#}&,#]&//
  Grid

Out[1] = 857142
     714285
     571428
     428571
     285714
     142857

241 :デフォルトの名無しさん:2017/05/25(木) 21:56:10.72 ID:ShXkIc2G.net
>>227 Io
a := 999999 / 7
for(i, 6 * a, a, -a, i println)

242 :デフォルトの名無しさん:2017/05/25(木) 22:10:21.35 ID:73MQsPzv.net
>>227 Nim
for i in countdown(6,1): stdout.writeline i*142857

243 :デフォルトの名無しさん:2017/05/25(木) 23:50:37.58 ID:SwCyV6dp.net
>>227 ruby
puts (1..6).map {|i| i * 142857}.reverse

>>227 rust
fn main() {
//println!("{}", (1..7).rev().map(|i| (142857 * i).to_string()).collect::<Vec<_>>().join("\n"));
for i in (1..7).rev() {println!("{}", 142857 * i)}
}

244 :デフォルトの名無しさん:2017/05/27(土) 20:11:30.48 ID:x9+XjHY5.net
227 R
write((6:1)*999999/7.",1)

245 :デフォルトの名無しさん:2017/05/28(日) 14:54:52.19 ID:/dM5Kslk.net
>>227 SWI-Prolog
main(N, X) :- X1 is X * N, write(X1), nl, N > 1, N1 is N - 1, main(N1, X).
:- main(6, 142857).

246 :プログラミングのお題:2017/06/01(木) 12:06:57.65 ID:o30QeXgp.net
トランプ氏の謎の単語

covfefe

の謎を解明する。

247 :デフォルトの名無しさん:2017/06/01(木) 13:12:24.25 ID:/fo1vNun.net

c,o,v,f,e,f,eの7文字をランダムに出力し続け、covfefeが完成するまでの文字数をカウント

248 :デフォルトの名無しさん:2017/06/01(木) 13:47:08.78 ID:sszEh+oH.net
>>247 Squeak/Pharo Smalltalk

| count covfefe buffer |
count := 0.
covfefe := 'covfefe' asOrderedCollection.
buffer := covfefe copy atAllPut: $*.
[ count := count + 1.
 Transcript nextPut: (buffer removeFirst; add: covfefe atRandom).
 (count isDivisibleBy: 1000) ifTrue: [Transcript endEntry].
 buffer = covfefe
] whileFalse.
Transcript endEntry.
^count "= 34301 "

249 :デフォルトの名無しさん:2017/06/01(木) 23:30:50.24 ID:MQJNOblg.net
>>247 効率悪いけれど…
Perl

$w='covfefe';
@l=split '',$w;
do {
$_ .= $l[int(rand(7))];
} while (!/$w/);
print index $_, $w . "\n";

250 :デフォルトの名無しさん:2017/06/01(木) 23:44:01.17 ID:MQJNOblg.net
>>249
ゴメン、これだと最初に見つかったときに0になっちゃうんで
「完成するまでの文字数」を出すためには
× print index $_, $w . "\n";

○ print index($_, $w) + 7 . "\n";
に修正させてください。

251 :デフォルトの名無しさん:2017/06/02(金) 02:47:24.66 ID:znhUDH9D.net
>>247 Nim

import random
const word = "covfefe"
var count, match = 0
randomize()
while match != word.len():
    let ch = word[random word.len()]
    stdout.write ch
    match = if ch == word[match]: match + 1 else: 0
    inc count
echo "\n", count

252 :デフォルトの名無しさん:2017/06/02(金) 07:41:45.64 ID:r1TSATN8.net
それだとccovfefeのときとか抜け落ちるんじゃね?

253 :デフォルトの名無しさん:2017/06/02(金) 14:13:15.27 ID:znhUDH9D.net
>>252
その通りですね
お題のcovfefeなら安直な修正でいいかな

-    match = if ch == word[match]: match + 1 else: 0
+    match = if ch == word[match]: match + 1 elif ch == word[0]: 1 else: 0

254 :デフォルトの名無しさん:2017/06/02(金) 20:36:39.00 ID:GZbUsfWf.net
>>247 ruby
covf, buff = 'covfefe', ''
buff += covf.chars.sample until buff.end_with? covf
p buff.size

255 :片山博文MZ :2017/06/02(金) 21:34:10.62 ID:ECCI41T/.net
お題: 2つの別々の牧場ウィンドウがあり、その中に複数の牛がうごめいている。複数の牛を選択して、別の牧場ウィンドウにドラッグ&ドロップできるようにしなさい。

256 :デフォルトの名無しさん:2017/06/02(金) 21:43:02.00 ID:ZGhpg2JU.net
GUIの課題ってコードも長くなりがちたから答えにくくない?

257 :片山博文MZ :2017/06/02(金) 21:46:38.91 ID:ECCI41T/.net
賞金は早い者勝ちで三千円。スクリプト野郎どもにはできないだろう。へっ。

258 :片山博文MZ :2017/06/02(金) 22:03:27.70 ID:ECCI41T/.net
>>256
githubがあるじゃないか。

259 :デフォルトの名無しさん:2017/06/02(金) 22:03:48.67 ID:doJoDkLD.net
賞金とか誰かと思ったら片山博文MZか。

260 :デフォルトの名無しさん:2017/06/02(金) 23:44:04.74 ID:cFhdiKGB.net
三千円じゃ安い

261 :デフォルトの名無しさん:2017/06/03(土) 01:24:32.19 ID:4TZnG99D.net
>>255
動的言語のscriptでもQt,Wxwidget,Tkinterなど色々のGUI fwが使えるから書けるよ。
でも、エッセンスがなく、会コードが無駄に長くなるお題は、作成に時間がかかるし獣よな技術はないし
趣旨を考えで出題しろよ。
すくなくとも自分で作る気になれる題を出せ

262 :デフォルトの名無しさん:2017/06/03(土) 01:26:10.05 ID:4TZnG99D.net
>>261
会コードが無駄に長くなるお題は、作成に時間がかかるし獣よな技術はないし

解コードが無駄に長くなるお題は、作成に時間がかかるし技術はないし

263 :デフォルトの名無しさん:2017/06/03(土) 01:50:43.99 ID:4TZnG99D.net
でも、まぁ地獄の沙汰も金次第というじゃありませんか。
お見積もり30万円以上でしたらpython+tkinterで書いてお納めすtることも
検討させていただきますよ。ハイ
更にハイグレードに300万円だったらPerl+Ptkもお付けいたいます。
いかがですか?だんな
もみ手

264 :デフォルトの名無しさん:2017/06/03(土) 19:38:09.78 ID:bt+/AaG5.net
【問題】
アルバートとバーナードは、シェリルと友達になったばかりです。
シェリルの誕生日を2人は聞きましたが、彼女は10個の日にちを候補としてあげました。

・5月15日、5月16日、5月19日
・6月17日、6月18日
・7月14日、7月16日
・8月14日、8月15日、8月17日

それからシェリルは、アルバートに「月」だけを、バーナードに「日付」だけをそれぞれ教えました。
アルバート「僕はシェリルの誕生日を知らないけど、バーナードも知らないよ」
バーナード「僕はシェリルの誕生日を知らなかったけど、今は知ってるよ」
アルバート「それなら僕もいつだか知っているよ」
シェリルの誕生日はいつでしょうか?

プログラムを書いてプログラムに解かせること。

265 :デフォルトの名無しさん:2017/06/03(土) 20:24:51.42 ID:LavjhbKR.net
Console.WriteLine("知らんがな");

266 :デフォルトの名無しさん:2017/06/03(土) 20:43:26.68 ID:GyX0IIiI.net
(begin (display "知らんがな")(newline))

267 :デフォルトの名無しさん:2017/06/03(土) 21:39:03.81 ID:3br47TQ3.net
print("知らんがな")

268 :デフォルトの名無しさん:2017/06/03(土) 21:39:54.96 ID:+ZiDT+Cr.net
世界で初めて原爆実験が行われた日を
わざわざ答えに選んだのは何か意図があってのこと?

269 :デフォルトの名無しさん:2017/06/04(日) 02:59:00.78 ID:vYNPJugT.net
2年前のログ見てみたけどそのときはここに持ちこむ奴いなかったんだな
Prologおじさんとかが嬉々としてやりそうだけど

270 :デフォルトの名無しさん:2017/06/04(日) 03:31:57.97 ID:JSJPiIxT.net
7月16日

271 :デフォルトの名無しさん:2017/06/04(日) 11:23:47.25 ID:ICo3ogub.net
>>264 f#
http://ideone.com/cBGHxs

272 :デフォルトの名無しさん:2017/06/04(日) 12:17:11.76 ID:/fL6DBjJ.net
>>264 Perl

@md = ([5, 15], [5, 16], [5, 19],
[6, 17], [6, 18],
[7, 14], [7, 16],
[8, 14], [8, 15], [8, 17]);
push @{$c{$$_[1]}}, $$_[0] for @md;
push @{$d{$$_[0]}}, $$_[1] for grep{1 < @{$c{$$_[1]}}} @md;
while (($m, $v) = each %d) {
print "$m/$$v[0]\n" if 1== @$v;
}

実行結果

$ perl 9_264.pl
6/17

273 :デフォルトの名無しさん:2017/06/04(日) 12:22:24.53 ID:/fL6DBjJ.net
>>272
7月16日が正解なら 解き方間を違えているのかも知れん

274 :デフォルトの名無しさん:2017/06/04(日) 14:27:12.46 ID:ArM8onCc.net
アルバート「僕はシェリルの誕生日を知らないけど、バーナードも知らないよ」
5,6月を排除

バーナード「僕はシェリルの誕生日を知らなかったけど、今は知ってるよ」
14日を排除

アルバート「それなら僕もいつだか知っているよ」
残り候補が一つの月 -> 7月16日

275 :デフォルトの名無しさん:2017/06/04(日) 15:02:32.31 ID:/fL6DBjJ.net
>>272 の解き方で考えたこと

アルバート「僕は(「月」だけしか教えてもらっていないので)シェリルの誕生日を知らないけど、
      (「日付」だけを教えてもらった)バーナードも知らないよ」
⇒「日付」だけ聞けば誕生日だと判明する、即ち日の登場回数が一回だけの月日、
 具体的には5月19日、6月18日は対象外とみなし除去

バーナード「僕は「日付」だけを教えてもらっても)シェリルの誕生日を知らなかったけど、
アルバートが「僕はシェリルの誕生日を知らないけど、バーナードも知らないよ」と言うのを聞いて
今は知ってるよ」
⇒日の登場回数が一回だけの19日、6月18日を除去したあと、
 登場回数が一回だけの日が バーナードの聞いた「日付」に当たり、
 誕生日だと考えられる。

⇒6/17

この考え方が違ったんだろうな…

276 :デフォルトの名無しさん:2017/06/04(日) 15:42:35.42 ID:ArM8onCc.net
>>275
アルバートは月を知ってるが、バーナードも知らない事を確信できるのは、
18,19日を含まない7,8月のどちらかという事になる -> 5,6月は全削除

それを聞いてバーナードは誕生日がわかるので、7,8月両方に含まれる14日ではなく、
15,16,17日のどれかになる

それを聞いてアルバートがわかるので、候補が一つしか残ってない7月16日という事になる

277 :デフォルトの名無しさん:2017/06/04(日) 16:11:15.57 ID:Thsr1gL6.net
6/17の方ぽいね

278 :デフォルトの名無しさん:2017/06/04(日) 16:23:55.65 ID:8topuOK/.net
5,6月は全排除でしょ

279 :デフォルトの名無しさん:2017/06/04(日) 16:27:46.47 ID:/fL6DBjJ.net
>>278
そこがオレにはよく理解できていなくてさ。
まぁ言葉にあいまいな面があるかもしれんから解釈に差が出たのかな

280 :デフォルトの名無しさん:2017/06/04(日) 16:46:10.26 ID:3NGxsH/O.net
>>279
解釈の差だけが問題じゃないだろ

> ⇒日の登場回数が一回だけの19日、6月18日を除去したあと、
>  登場回数が一回だけの日が バーナードの聞いた「日付」に当たり、
>  誕生日だと考えられる。

18日、19日は日の登場回数が一回だけであるということは
他の日は複数回登場するということだからその論理は破綻してる

281 :デフォルトの名無しさん:2017/06/04(日) 16:54:14.37 ID:/fL6DBjJ.net
>>280
それは誤解というか解読不足。
5月19日、6月18日が除去されることによって、
元々複数回登場していた他の日のうち6月17日が単一の日となり
17日という日付さえ知らされれば、誕生日は6月17日と判明できる。

282 :デフォルトの名無しさん:2017/06/04(日) 17:03:26.79 ID:ArM8onCc.net
>>281
客観的に見て、アルバートがバーナードも知らない事を確信できる為には、
アルバート自身が知っている月には18,19日が含まれていない必要がある
従って、アルバートが知っている月は5,6月ではないという事

283 :デフォルトの名無しさん:2017/06/04(日) 17:07:47.61 ID:/fL6DBjJ.net
>>282
なるほど考え方は理解できた。
でも5月6月には他の日もあるからバーナードが聞かされた日がそれらで無いとはっきりしていないうちに
月ごと排除して大丈夫?

284 :デフォルトの名無しさん:2017/06/04(日) 17:11:33.29 ID:/fL6DBjJ.net
>>281
17日は8月17日もあるから、
6月が17日だけになったからといって、
6月17日が誕生日だとするのは
アルバート、バーナードの台詞を根拠に基づく論理に
無理がないか検証不十分だという気が自分でもしてきた

285 :デフォルトの名無しさん:2017/06/04(日) 17:12:55.48 ID:ArM8onCc.net
>>283
逆に最初の時点でアルバートはバーナードが知らないとは確信できない
例えばアルバートは6月と聞かされた場合、6月18日の可能性もあるので、
それだとバーナードは18日と聞かされているから知ってるかもしれない

286 :デフォルトの名無しさん:2017/06/04(日) 17:26:33.24 ID:/fL6DBjJ.net
>>285
大体分かった。ありがとう
単一な日をまったく含まない月を教えられたからこそ、
アルバートは最初の台詞
「僕はシェリルの誕生日を知らないけど、バーナードも知らないよ」
になったという考え方だね。

287 :デフォルトの名無しさん:2017/06/04(日) 17:30:30.29 ID:ArM8onCc.net
>>286
イエスイエス

288 :デフォルトの名無しさん:2017/06/04(日) 22:32:17.70 ID:BGwDpUyE.net
ディスコプログラミングコンテスト 2017 7/8
https://www.disco.co.jp/procon/
の練習問題
https://www.disco.co.jp/procon/#example

解答バレにならないように1語だけ書くけど
1問目の

TOUSHITSUWOTOTTE

だけが、わけわからん

289 :デフォルトの名無しさん:2017/06/05(月) 00:31:08.14 ID:cnQQABDP.net
>>288
Q1といてみたけど
「糖質を摂って」じゃない?

290 :デフォルトの名無しさん:2017/06/05(月) 08:51:14.13 ID:h9zULQkR.net
糖質を摂って
だよねぇ。

謎のメッセージだな。

291 :デフォルトの名無しさん:2017/06/05(月) 09:47:23.18 ID:Na336mAM.net
>>290
いやそう謎でもない。
解の文を全部通しで読むと
この前に食べものの話がある。

292 :デフォルトの名無しさん:2017/06/05(月) 10:35:52.00 ID:S7KIxJHR.net
糖質が頭の働きを良くするという通説と逆に頭を鈍らせるという説があるけど
この会社が前者を支持することを明言する意味がある

293 :デフォルトの名無しさん:2017/06/05(月) 13:18:37.12 ID:nmQdV7hA.net
例題に挑戦して下さりありがとうございます!全問正解した参加者にはディスコ限定どら焼きをプレゼント!
大会当日に受付でお渡ししします。糖質を摂って優勝目指して頑張って下さい!

294 :デフォルトの名無しさん:2017/06/05(月) 13:45:37.09 ID:hFu+7Z6c.net
誰が統失だって?

295 :デフォルトの名無しさん:2017/06/05(月) 17:48:32.89 ID:2PleAf1D.net
ダメじゃん。全解答を書いちゃって。

でも簡単すぎる問題だしどうでもいいか。

296 :デフォルトの名無しさん:2017/06/26(月) 21:09:32.46 ID:92/cX5j1.net
前にあったやつ。



回転寿司にやってきた私は、コンベア上の寿司をすべて食べて帰ることにしている。
コンベアは毎秒1皿分の速度で流れ、目の前の皿を取るか取らないかを選ぶことができる。
皿取ると同時に食べ始め、食べている間は次の皿を取ることができない。
私が取る以外、皿は追加されたり無くなったりしない。
コンベアの状態が次のような文字列で与えられる。 
"31_2"
数字はその皿を食べ終えるのにかかる秒数を表し、_は皿がないことを表す。1文字目が目の前にあり毎秒、左へ回転する。
例えば、"31_2"で最初の皿を食べたとき食べ終わった時の状態は、"2_1_"となる。

すべての寿司を食べ終えるまで最短何秒かかるか求めよ。
"12_3" > 6秒
"313__" > 8秒
"4_35_1264_23_434" > 60秒
"123456789123456789" > 98秒
"88967472612377988186" > 149秒
"19898693316679441672" > 170秒
"93769682716711132249893" > ?

297 :デフォルトの名無しさん:2017/06/26(月) 22:59:10.29 ID:GM19K0OY.net
計算オーダーの条件は?
無いなら二進木で

298 :デフォルトの名無しさん:2017/06/26(月) 23:01:34.06 ID:GM19K0OY.net
皿がもうちょっと多いと難しくなるけど、>>296なら力業でも

299 :デフォルトの名無しさん:2017/06/26(月) 23:40:33.73 ID:JhsaOf6q.net
>>296 Perl
ttp://ideone.com/iUAYUy

実行結果は

$ perl 9_296.pl
12_3: 6
313__: 10 (合わない…orz)
4_35_1264_23_434: 62 (合わない…orz)
123456789123456789: 98
88967472612377988186: 151 (合わない…orz)
19898693316679441672: 170
93769682716711132249893: 176

となり、半分が合わない。
そのうち 313__ を手で研鑽すると 10 になるのだが、
313__ は本当に8になるの?

300 :デフォルトの名無しさん:2017/06/26(月) 23:41:06.22 ID:JhsaOf6q.net
>>299
研鑽じゃねぇや手で起算な。

301 :デフォルトの名無しさん:2017/06/26(月) 23:45:40.94 ID:JhsaOf6q.net
>>300
起算でもねえ、手で計算な…orz

302 :デフォルトの名無しさん:2017/06/26(月) 23:58:59.38 ID:92/cX5j1.net
313__ はこれでは?

まず一皿ながして
1を食う、2秒時点の状態  3__3_
3を食う、5秒時点の状態  3____
3を食う、8秒で食べ終わり

303 :デフォルトの名無しさん:2017/06/27(火) 00:02:53.75 ID:5fkiI7k4.net
>>302
そっか、最初の3を食べちゃったら最短時間にならないな
>>299は最初の皿からダボハゼみたいに食いつくので必ずしも最短にはならないな
きっと腹が減りすぎていたんだろう…orz

304 :デフォルトの名無しさん:2017/06/27(火) 00:12:08.45 ID:PK1LDhK1.net
>>296 は、
目の前にあるやつを食べ続けるだけで最短になっちゃうのもあるってことか。

305 :デフォルトの名無しさん:2017/06/27(火) 00:46:54.34 ID:v9AhJc3r.net
>>296
http://ideone.com/PGKCb4

計算量は2^n*n (n:コンベアの長さ) n=24はほぼ限界
n!をbitDPで計算量落とす。

(空皿処理で昔より手を抜いている)

306 :デフォルトの名無しさん:2017/06/27(火) 08:18:25.38 ID:bJ//gE7J.net
考えてみたけと計算オーダーを減らすのはむずかしいね
枝刈りは色々と出来るけど

307 :デフォルトの名無しさん:2017/06/28(水) 00:38:04.57 ID:SkQPDtDj.net
>>218 Perl
ttp://ideone.com/ylFIEa

ソースコード4行目の
my $n = 8; # 分割する自然数を設定
の8を書き換えると他の整数についてもヤング図形を出力できます。

308 :デフォルトの名無しさん:2017/06/28(水) 10:38:30.75 ID:+O8L6XqQ.net
366 :nobodyさん 2017/05/29(月) 16:07:39.16 ID:6v4UcGhE
今回の民法改正、ソフトウェア受託開発の場合、(検収後ではなく)バグ発見後1年瑕疵担保責任があるということで、地獄かよ、と思ったが、
元々問題が起きがちな受託案件がビジネス的に成立しなくなることで強制的に業界再編につながるなら良いことかもと思うようになった。
一部で地獄を見ても。
https://twitter.com/yukihiro_matz/status/869061879389343744

367 :nobodyさん 2017/05/29(月) 16:28:06.55 ID:6v4UcGhE
ニュース - 改正民法が成立、「瑕疵担保責任」などシステム開発契約に影響大:ITpro
http://b.hatena.ne.jp/entry/itpro.nikkeibp.co.jp/atcl/news/17/052601508/

372 :nobodyさん2017/05/29(月) 19:10:37.12 ID:???
Railsでシステム作って納品する

Railsはマイナー、メジャーのアップデートが半年以内に必ずある

客がアップデートする。アップデートによるエラーやバグ、動作の不具合に気づく

気づいてから1年以内に通知すれば、5年間無料保証ゲット

つまりRailsがアップデートするたびに、無償の修正作業を発生するということかな

376 :nobodyさん2017/05/30(火) 09:20:20.09 ID:L5po86sS
>>378>>379>>375
客が瑕疵担保責任法の法改正を知ってくると思うから、今後5年無償保証をお願いされるだろう
営業がそれでも仕事を取ってこれるか?たぶん無理だろう。無限の直していたら赤字になる。
こういう保守に弱い言語、ころころ仕様が変わる言語は仕事として発生しなくなってくる。
これは変わり目だ。お前らも早く逃げたほうがいいぞ。RubyやPHPなど動的言語は確実に廃れる。
保守に強い言語のみ生き残れる。

309 :デフォルトの名無しさん:2017/06/28(水) 10:38:43.66 ID:+O8L6XqQ.net
瑕疵担保責任(かしたんぽせきにん)

瑕疵担保責任のポイント

民法改正で事実上期限が「無制限」になった
バグや設計のミスなどは、瑕疵担保責任
納品物に不具合があれば損害賠償を請求される可能性もある
不具合を指摘されたらすぐに行動をとるべし
軽微なミスでも先延ばししない

http://www.atmarkit.co.jp/ait/articles/1706/26/news014.html
http://itpro.nikkeibp.co.jp/atcl/news/17/052601508/?rt=nocnt

改正法では欠陥に気付いてから1年以内にITベンダーに通知すれば、
通知後5年以内は修正や報酬の減額などを求められるとしている

全ベンダーが泣いた民法改正案を解説しよう その1
http://www.atmarkit.co.jp/ait/articles/1609/14/news009.html
http://www.atmarkit.co.jp/ait/articles/1609/14/news009_2.html
http://www.atmarkit.co.jp/ait/articles/1609/14/news009_3.html

ポイント1:修補や損害賠償、契約解除の期限がなくなる

従来あった「瑕疵担保期間は引き渡しから1年」という考えはなくなる。
条文にある通り、注文者は成果物が契約の目的に適合しないことを発見したら、
その「発見したときから1年以内」ならさまざまな請求ができる。発見が10年後なら、
11年後まで請求可能なのだ。

もっとも、現実のユーザーとベンダーの関係でも、たとえ契約書に「瑕疵担保責任期間は納品から1年と」明記されていても、
「2年目以降は不具合の修正に対応しない」と主張するベンダーはまれだ。多くの場合は、納品から何年たっても、
バグが見つかればユーザーのところに飛んで行き、無償で改修するだろう。

310 :デフォルトの名無しさん:2017/06/28(水) 10:56:55.66 ID:fzKvr8sM.net
コピペマン参上!!まで読んだ

311 :デフォルトの名無しさん:2017/06/28(水) 22:13:35.80 ID:CE32dHls.net
>>305
修正 http://ideone.com/PGKCb4

経路情報復元(ベストは複数あるかもしれない中から一つ選んで)。
ついでにその経路での途中経過を表示してみた。
インデックス、待機秒数、開始時間、食事秒数、終了時間、

312 :デフォルトの名無しさん:2017/06/28(水) 23:25:36.64 ID:KJ5QOfYg.net
>>218 >>307 Perl
http://ideone.com/GC2JTj

再帰を使わず、リスト処理とloopで書き直したら
もう少しコンパクトですっきりしたものになりました…

313 :デフォルトの名無しさん:2017/07/01(土) 18:47:40.02 ID:tnFwdv+3.net
前に書いたけどコード紛失した。

314 :デフォルトの名無しさん:2017/07/01(土) 21:49:46.76 ID:WoDBh/Wa.net
お題: パスカルの三角形

315 :デフォルトの名無しさん:2017/07/01(土) 22:05:06.08 ID:1Cx1myAa.net
>>314
良いお題だね

316 :デフォルトの名無しさん:2017/07/02(日) 13:43:30.38 ID:bHJ33QxN.net
>>314 Perl5
http://ideone.com/YCw1OC

桁の多い数値の幅を反映して数値間の空白の数を決めれば
数値の位置がもう少し見やすくなるとおも…

317 :デフォルトの名無しさん:2017/07/03(月) 08:12:51.85 ID:EltE6GHS.net
お題:完全なヤング図ソルバー。
http://ideone.com/hkUkFM
書いてみたけど、不完全なのがやっとだった。
あってるかもわからん。

図の効率がいいほど評価が上がります。

318 :デフォルトの名無しさん:2017/07/04(火) 21:28:12.76 ID:QK6Kginy.net
>>296 >>299 Perl5
http://ideone.com/0yJ5U9

リスト処理ではなく、先ずは正規表現と文字列処理を使って書いてみた。

31…の3のように、食べているうちに後続の数値皿が通り過ぎてしまうような、
取りこぼしを起こし得る皿では、その数値を食べるか、あるいはスルーするか、
再帰的に両方に分岐し、木構造で計算しているが、
逆に食べている間に飛び越しを起こさないところでは、分岐が不要なので
来た順に直ちに食べることによって、枝分かれの過剰な細分化を抑制した。

それでも全探査すると、サンプルデータの三つ目まではすぐ解けるが、
四つめ以降は時間がかかりいつ終わるか分からない。

そこで、検索された食事秒数の最小値の更新状況を記録し、
同じ最小値が一定回数以上連続して繰り返し検出されるようになったら
最短値に収束したと見なし、探索を打ち切ることによって短時間で
解を出力できるようにした。打ち切り上限は10をハードコードしてあるが
今回のサンプルデータについては4か5で十分そうだ。

なお、23_ のような、2を食べることによって飛び越しを起こすポイントの
一番最後のものは,食べずにスルーして先に2を食べた方が、
次の周で早く食べ終わることは明らかだ。
これを演繹的に繰り返して、遡ってゆけば、上記のように木構造に
わたって動的に計算して探索しなくても、静的に求解できそうな気がしたが
難しそうなので、見送った。

319 :デフォルトの名無しさん:2017/07/04(火) 21:31:48.78 ID:QK6Kginy.net
>>318
書き忘れたけど、食事秒数を探索中に、それまでに見つかっている最小病数を超えたら
打ち切るという、簡単な枝刈りを取り入れてあります。

連投スマソ

320 :デフォルトの名無しさん:2017/07/04(火) 23:51:29.56 ID:sQGcZTdy.net
>>318
枝刈りで最短を刈り取ってしまったら駄目じゃないか
例えば "3324" -> 15秒 にならないな

321 :318:2017/07/06(木) 00:31:45.46 ID:iCfNzc8Y.net
>>320
誤解です。
枝刈りは、ある探索中の枝において始点から既に経過した秒数が
それまでの別の枝における探索で最後まで食べた最小秒数を超過したら、
現在の枝の探索はもうこれ以上進んでも秒数が増える一方なので打ち切って
別の枝の探索に移るというものなので大丈夫です。
"3324" の最短秒数を探索すると 15秒になります。

322 :デフォルトの名無しさん:2017/07/06(木) 00:52:46.56 ID:ywrsmrRJ.net
>>321
あれ、変だな
>>318のリンク先のコードで"3324"を計算すると 16 になるんだけどこっちの環境が変なのかな?
同様に"3328"、"3364"は最短19秒だけど>>318だと20になった

323 :318:2017/07/06(木) 01:20:52.00 ID:iCfNzc8Y.net
>>322
同じコードをideoneに張りなおして3324を入力して実行してみました。
http://ideone.com/vXrTp8

ソースを一箇所編集しています。

31 die if $hit >= 20; # 一定以上同じ最小値が繰り返し計算されたら収束と判定し脱出

の繰り返し回数上限判定地を10から20に増やしています。

3324は15になりますが、15が登場するのは11回目以降でそれまで16が出続けます。
3364も20が10回繰り返した後19が出て続きます。

お手数おかけしますが
一定以上同じ最小値が繰り返し計算されたかの判定値を10より多くして
評価してください。

324 :318:2017/07/06(木) 01:35:51.32 ID:iCfNzc8Y.net
>>323
3324と3364の解を見ていて気が付いた点があります。

一定以上同じ最小値が繰り返し計算されたかの判定値を20にしていますが、
3324の15や3364の19は20ではなくて13回しか現れず、これが最小値のため
解として表示されています。
これは、3324の15や3364が4桁しかないので、
最小値が20回現れる前に全探査が完了し、その中で見つかった最小値を
解として表示していることによります。

>>318の一定回数繰り返したら収束とみなすという判定方法は、
ニュートン法のような数値計算では有効ですが、
>>296の問題の解の判定方法としては適切とは言えないかもしれませんね…orz

325 :デフォルトの名無しさん:2017/07/06(木) 01:53:08.89 ID:bBo7q2K6.net
3324を拡張した887654329は閾値どれくらい増やせば対応できるんですかね

326 :318:2017/07/06(木) 02:06:40.59 ID:iCfNzc8Y.net
>>325
延々探索を続けないと解に至らないかもしれない入力については
定数で打ち切りを決めるこの解法じゃ解に至りにくいかもしれない。
887654329がそういったカテゴリーに属する入力かというと
チョット分からない。
なので適切な閾値はこれだと断言しにくいです。
さーせん

327 :デフォルトの名無しさん:2017/07/06(木) 21:08:21.84 ID:ywrsmrRJ.net
>>326
結局>>321は大嘘だったし、閾値20の>>323にしたところで
例えば"14432"は最短にならないし
閾値が決められないならその解法はやはり駄目だな

328 :318:2017/07/06(木) 22:03:39.91 ID:0agEc1HZ.net
>>327
閾値20で打ち切ると最小に至らない入力もあるのはそうだけど、
計算しても最小を更新しない枝に降りずに切り上げてくる>>321は嘘ではないよ。

329 :318:2017/07/06(木) 22:08:34.07 ID:0agEc1HZ.net
見込みの無い枝をもっと早めに切り上げらる方法がありそうだと気が付いた。
それによって20で打ち切るようなやり方を改善できればいいんだけれども…
それでも計算量が増えていくと、真の解に至るまでにかかる時間が増大して
とけなくなる

330 :デフォルトの名無しさん:2017/07/06(木) 23:01:53.78 ID:ywrsmrRJ.net
>>328
閾値20で打ち切るのは枝切りじゃないという主張のようだけど
打ち切るという動作は枝切り以外の何物でもない

>>318は”3324”の最短に到達しないから>>321
> "3324" の最短秒数を探索すると 15秒になります。
というのも嘘

331 :318:2017/07/06(木) 23:19:13.87 ID:0agEc1HZ.net
>>330
絡むね。そんな暇あったらコードでも書けばいいのにw

閾値20でその入力については解の探査を止めて
別の枝に移らず次の入力データに移るのはどちらかといえば中断で、
枝かりではないでしょ。

>319
> >>318
> 書き忘れたけど、食事秒数を探索中に、それまでに見つかっている最小病数を超えたら
> 打ち切るという、簡単な枝刈りを取り入れてあります。
にかいてあるでしょうに。


>>318は”3324”の最短に到達しないから>>321
> "3324" の最短秒数を探索すると 15秒になります。
>というのも嘘

これは10回の打ち切りの緩和を書きもらしたんだよ。

何が狙いで、こだわって絡んでくるやらねぇ。

332 :318:2017/07/06(木) 23:37:37.26 ID:0agEc1HZ.net
「打ち切る」という言葉を

>318
>…
>同じ最小値が一定回数以上連続して繰り返し検出されるようになったら
>最短値に収束したと見なし、探索を打ち切ることによって短時間で
>解を出力できるようにした。打ち切り上限は10をハードコードしてあるが

では「その入力に対する求解を中断する」ところで使い、

>319
> >>318
> 書き忘れたけど、食事秒数を探索中に、それまでに見つかっている最小病数を超えたら
> 打ち切るという、簡単な枝刈りを取り入れてあります。

では「その枝の下の方への探索をせず、別の枝の探索に移る」枝刈りの
ところで使ったのが誤解を招いてしまったのかな…

333 :デフォルトの名無しさん:2017/07/07(金) 04:22:27.25 ID:pbX9YCbr.net
3次の動的計画法ってどんだけメモリ食うんや?

334 :デフォルトの名無しさん:2017/07/08(土) 03:20:24.48 ID:hDxZO8qP.net
お題: 自然数Nの平方根を整数部含めて(1000*N)桁求めたとき、出現する0の個数を数える
たとえば、N = 4の時ルート4を4000桁(整数部1桁+小数部3999桁)求めたとき、出現する0の個数は3999個

N = 3 => ?
N = 5 => ?
N = 7 => ?

335 :デフォルトの名無しさん:2017/07/08(土) 03:22:50.68 ID:5gcIwgbE.net
ブロックチェインの新手のコイン発掘か?

336 :デフォルトの名無しさん:2017/07/08(土) 03:59:02.35 ID:kzKE4jeR.net
>>334 Ruby
require 'bigdecimal'
[3, 4, 5, 7].each{|i|
n = 1000*i - 1
puts "N = %i => %i"%[i, ("%.#{n}f"%BigDecimal(i).sqrt(n)).count(?0)]
}


N = 3 => 2956
N = 4 => 3999
N = 5 => 4956
N = 7 => 6954

337 :デフォルトの名無しさん:2017/07/08(土) 04:25:25.94 ID:kzKE4jeR.net
>>336はミス。0がこんなに多いわけがない

require 'bigdecimal'
[3, 5, 7].each{|i|
n = 1000*i - 1
puts "N = %i => %i"%[i, BigDecimal(i).sqrt(n).floor(n).to_s(?F).count(?0)]
}

N = 3 => 309
N = 5 => 492
N = 7 => 738

338 :デフォルトの名無しさん:2017/07/08(土) 07:13:48.73 ID:hDxZO8qP.net
>>337
N = 5の場合が間違ってると思う
多分、丸めモードの関係か、精度が足りてないと思われる

339 :デフォルトの名無しさん:2017/07/08(土) 09:51:11.17 ID:3gkxwDpM.net
>>334 C++

#include <iostream>
#include <string>
#include "gmpxx.h"
int main () {
  int sq_me;
  while( std::cin >> sq_me ){
    int prec = 1000*sq_me, cnt = 0;
    mpf_class sq_out = sqrt( mpf_class(sq_me, prec*4) );
    mp_exp_t exp;
    auto str = sq_out.get_str( exp,10,prec );
    for( auto it=str.begin(); it!=str.end(); it++ ) if( *it=='0' ) ++cnt;
    std::cout << "N = " << sq_me << " => " << cnt+prec-str.length() << '\n';
  }
}

N = 3 => 309
N = 5 => 493
N = 7 => 738
N = 11 => 1079
N = 13 => 1305
N = 17 => 1664
N = 19 => 1875
N = 23 => 2265
N = 29 => 2911
N = 31 => 3113
N = 37 => 3795
N = 41 => 4095
N = 43 => 4312
N = 47 => 4798
N = 53 => 5340

340 :デフォルトの名無しさん:2017/07/08(土) 11:54:07.74 ID:H5pSyGdF.net
>>334 Squeak/Pharo Smalltalk

| sqrt |
sqrt := [:n :m |
 "ref. https://xar.sh/post/67066374255/ "
 | a b |
 a := 5 * n. b := 5.
 [:exit | [
  a >= b ifTrue: [a := a - b. b := b + 10] ifFalse: [
   b log > m ifTrue: [exit value] ifFalse: [
    a := a * 100. b := b // 10 * 100 + (b \\ 10)
   ]
  ]
 ] repeat] valueWithExit.
 b
].

#(3 5 7) collect: [:i | i -> (((sqrt value: i value: i*1000) asString first: i*1000) occurrencesOf: $0)]

"=> {3->309 . 5->493 . 7->738}"

341 :デフォルトの名無しさん:2017/07/08(土) 12:18:57.24 ID:hDxZO8qP.net
>>339
N = 29とN=41の場合が間違ってる可能性? それ以外は正しい模様
N = 29 => 2912、N = 41 => 4094 じゃなかろうか

>>340
合ってる

342 :デフォルトの名無しさん:2017/07/08(土) 12:48:13.59 ID:1hnJaOYb.net
>>334 Perl5
http://ideone.com/fAyh2l

343 :デフォルトの名無しさん:2017/07/08(土) 13:10:39.20 ID:1hnJaOYb.net
>>334 Perl5
http://ideone.com/cMBD8o

>>342 をもう少しすっきり書けたので差し替え。

344 :デフォルトの名無しさん:2017/07/08(土) 13:31:31.09 ID:3gkxwDpM.net
>>341
> N = 29 => 2912、N = 41 => 4094 じゃなかろうか

それが正しいようです
GNU MPだとどうしても最後の桁は四捨五入?されるようで
任意のNに対して正確な答えを出すのは面倒なので修正は断念

345 :デフォルトの名無しさん:2017/07/09(日) 10:26:36.71 ID:aJSGzdPS.net
結局バイナリーツリーになっちゃったなぁ。むずかし。

346 :デフォルトの名無しさん:2017/07/09(日) 10:55:01.51 ID:xLkjNLhf.net
>>344
考え直したら面倒じゃなかった

>>334 C++
http://codepad.org/k0Sq8Fqo


N=10000くらいまでなら現実的な時間で計算出来そうだ

347 :デフォルトの名無しさん:2017/07/09(日) 11:25:12.20 ID:nhQrw0mT.net
N=100000, 1億桁のくらいなら現実的な時間で出来る

丸めは切り捨て?四捨五入?

348 :デフォルトの名無しさん:2017/07/09(日) 11:33:40.13 ID:nhQrw0mT.net
ルートの計算は速い
整数のルートは特に速い

349 :346:2017/07/09(日) 12:21:20.78 ID:4Kodr3MO.net
>>347
GNU MPだとget_str() とか gmp_sprintf() では四捨五入されるようなので
floor() であらかじめ切り捨ててから get_str() した

350 :デフォルトの名無しさん:2017/07/09(日) 12:57:37.64 ID:DBjzEn12.net
ルートの問題で初めてきたが、これってゼロの個数に上限があるのか? 簡単に求まるのか?
連続するゼロの個数の最大だろ?
無理数は規則なく無限に続くから、ゼロの個数ももし1000個連続が見つかれば、1001個もいつかでるとおもうんだが。

351 :デフォルトの名無しさん:2017/07/09(日) 13:14:46.15 ID:df6kAKcY.net
最大って何の話しとるんや

352 :デフォルトの名無しさん:2017/07/09(日) 13:28:51.97 ID:DBjzEn12.net
もとめる桁数のほうに上限があったのか、それを見逃してた。

353 :デフォルトの名無しさん:2017/07/09(日) 13:45:03.68 ID:NvRZfELm.net
連続する個数でもないぞw

354 :デフォルトの名無しさん:2017/07/09(日) 13:51:59.10 ID:DBjzEn12.net
どっちも間違えたな、ゼロの総数だったか。

355 :デフォルトの名無しさん:2017/07/09(日) 19:14:37.92 ID:6MYOcrZ9.net
>>349
floorを行った後の結果に誤差は無い
という検証は出来てるの?
何もしてないなら、それはたまたま偶然当たったっていうだけだぞ

ていうか、君には聞いてない
出題者の意図を聞いてる

356 :デフォルトの名無しさん:2017/07/11(火) 15:16:56.21 ID:1hL73PK3.net
√2でなるべく長い0の連続をみつけるは?

357 :デフォルトの名無しさん:2017/07/11(火) 15:49:47.43 ID:QxseLuPf.net
>>355
君には向いて無いよ

358 : ◆QZaw55cn4c :2017/07/11(火) 16:29:49.99 ID:ZfeFayuI.net
>>355
>floorを行った後の結果に誤差は無い
>という検証は出来てるの?

ぱっとみ当然だと思うんだが

>>356
何桁求めるか指定しないと意味がないのでは?

359 : ◆QZaw55cn4c :2017/07/11(火) 16:35:18.57 ID:ZfeFayuI.net
>>358
ん、考え直した
10進に変換した結果にて 99999 とかが末尾にあるようでは、余分の計算はしないといけないね

360 :デフォルトの名無しさん:2017/07/11(火) 18:55:08.87 ID:dSS1j36W.net
[][Tebla][]

}

000-"Yob*RtStrike"[%Kil\]MO,fla>%$9999VLTS

001-GYORLith"0\R"/"ESUBA"%$%

HADO-"EM","L","O","NU"###END

361 :デフォルトの名無しさん:2017/07/14(金) 06:57:35.22 ID:PYQ8V1MO.net
>>296
http://ideone.com/VzYVY9
C++。解けた気がする。
状態をメモ化してみた。
何で動いてるのか自分でもよくわからない。
暇だったので解いてみた。

362 :デフォルトの名無しさん:2017/07/14(金) 07:42:51.88 ID:PYQ8V1MO.net
あー多倍長精度演算ほしー。もちろん標準で。

363 : ◆QZaw55cn4c :2017/07/14(金) 07:55:58.80 ID:TDGI45F0.net
>>362
私も欲しかったので作ってしまった、今 >>334 を奮闘中

364 :デフォルトの名無しさん:2017/07/14(金) 07:56:52.78 ID:PYQ8V1MO.net
>>363
それはすごいな。
後々破棄するようなものを作るモチベーションが出てこないよ。

365 :デフォルトの名無しさん:2017/07/14(金) 07:57:44.37 ID:TDGI45F0.net
>>364
書き捨てに慣れてしまったんだ‥

366 :デフォルトの名無しさん:2017/07/14(金) 07:59:21.83 ID:PYQ8V1MO.net
>>365
あはは・・・。
コード書き捨てるのは良いけど、道具書き捨てるのは俺には向いてないわ。
なので、標準待ち。

367 :デフォルトの名無しさん:2017/07/14(金) 09:33:33.04 ID:gEZu1299.net
boostという任意倍長の計算Libraryがあります。
C++では使えるそうです。

368 :デフォルトの名無しさん:2017/07/14(金) 09:38:45.73 ID:PYQ8V1MO.net
>>367
Boostも良いんだけどね。残念なことにあれは実験環境で準標準って扱いなんだよなぁ。
あれから取り入れられるライブラリも多いんだけど、標準じゃないからね。
残念なことに。

369 :デフォルトの名無しさん:2017/07/14(金) 12:49:16.12 ID:gnKUWanp.net
まあ標準ライブラリしか使わない縛りをしたければ好きにすればいいんじゃない?

370 :デフォルトの名無しさん:2017/07/14(金) 14:27:07.19 ID:JyiCltLg.net
車輪の再発明

371 :デフォルトの名無しさん:2017/07/14(金) 14:31:43.15 ID:DwybRUfK.net
競プロみたいな相手方の環境使う物だと標準と準標準の差はでかい
自分の環境なら導入すればいいだけだが

372 : ◆QZaw55cn4c :2017/07/14(金) 18:52:31.62 ID:TDGI45F0.net
>>370
個体発生は系統発生を繰り返す

373 :デフォルトの名無しさん:2017/07/15(土) 12:42:06.98 ID:odVkuNfb.net
>>361
厳密解を出しているのなら、チャレンジ
(わかって近似値解狙いなら気にしないで)

"14432" と "887654329"

両方とも既出の"貪欲つぶし"(?)数列

"14432"は 20秒 (ゼロインデック順で02341)
"887654329"は 80秒(同123456708)でいける。

374 :デフォルトの名無しさん:2017/07/15(土) 14:59:21.20 ID:OEoVgGO0.net
>>373
http://ideone.com/cBzPSj
C++。それ解くとほかの問題が解けなくなる。
厳密解のつもりだったが、ちょっと自分の領分超えてるなぁ。
うまくいかないものだ。
真実が奥の方にあると貪欲法は弱いな。Orz

375 :デフォルトの名無しさん:2017/07/16(日) 16:33:07.01 ID:8ZBD9z9c.net
お題:
自分用多倍長整数演算関数

…って思ったけど、処理系の標準ではないとか、仕事でGNU MP使っては駄目とかの
制約で、簡易的なもの(乗算くらいまでとか)を書いた事ある人は少なくないと見た。

376 :デフォルトの名無しさん:2017/07/16(日) 18:30:49.57 ID:8+Akms5T.net
多倍長整数演算がサポートされている言語を使う

終わり

377 : ◆QZaw55cn4c :2017/07/16(日) 18:54:09.85 ID:eA1jggM5.net
>>375
C++98 http://codepad.org/hUObVCsR
オートボクシング等はなく便利にはできていない.

378 :デフォルトの名無しさん:2017/07/16(日) 20:34:05.63 ID:yctBkD01.net
掛け算の実装がキモだろう。
ここがボトルネックになるはず。
ここができると円周率とか、ルート計算も高速化できるはず。

379 : ◆QZaw55cn4c :2017/07/16(日) 20:36:24.09 ID:eA1jggM5.net
>>378
うん,FFTを使うそうだが‥いまいちよくわからない

380 :375:2017/07/16(日) 20:41:46.81 ID:8ZBD9z9c.net
>>376
仕事で言語を選べる立場になってみたいものだわ。
この言語でやってってののは多々あるけど…orz

>>377
Karatsuba-Ofman法を目指してごーごー

381 :デフォルトの名無しさん:2017/07/17(月) 22:48:25.95 ID:5edeqhg+.net
>>296
手計算で計算出来るレベルにまで計算量を減らせた
もちろん数学的な裏付け付きで
ある条件を見たせば一瞬で求まる

"123456789123456789" > 98秒
残念ながら、これだけはその条件を満たしてない

382 :デフォルトの名無しさん:2017/07/18(火) 06:37:17.84 ID:nFCFlf58.net
>>381
22とか2323もその条件を満たしてない感じ?

383 :デフォルトの名無しさん:2017/07/18(火) 07:37:05.09 ID:Ew0RSScO.net
22 は微妙
2323 は大丈夫

384 :デフォルトの名無しさん:2017/07/18(火) 07:41:44.22 ID:Ew0RSScO.net
まだコードになってないんで、
コードになったらアップします

寿司を食べる時間 < レーンの回転周期
という前提をつけちゃおうと思ったけど、
つけない方が良さそうですね

寿司を食べる時間がレーンの回転周期の整数倍の寿司は
ちょっと特別な処理が必要

385 :デフォルトの名無しさん:2017/07/18(火) 08:01:32.43 ID:Ew0RSScO.net
整数倍の寿司が無いもので
条件に当てはまらない最小は
2222
かな

386 :デフォルトの名無しさん:2017/07/18(火) 08:49:51.93 ID:YLlwVFMJ.net
>>334 SageMath
ttps://sagecell.sagemath.org/?q=brdclf

普通に(?)多桁のisqrt()なので何の捻りも無し。

387 :386:2017/07/18(火) 09:39:56.20 ID:YLlwVFMJ.net
>>339
つ mpz_sqrt()

388 :デフォルトの名無しさん:2017/07/19(水) 18:12:29.37 ID:Np9hKHT2.net
>>296
>>373
http://ideone.com/B9vl8l
C++。結局、i7-6700のmem2G使って7分で解けた。
どうしようもない位遅いな。
でも一応題意には添えたと思う。
もう見たくない・・・。Orz

高速化するにはインラインアセンブリ使うか、スレッド分割できるようなアルゴリズムかんがえるか。
よくわからんけど、数学で頑張ってる人に期待だ。

389 :386:2017/07/20(木) 01:57:16.81 ID:Q7XnESC/.net
100のべき乗に変更
ttp://sagecell.sagemath.org/?q=mciykc

390 :デフォルトの名無しさん:2017/07/21(金) 15:21:18.13.net
>>296
http://ideone.com/mXPglY
C++。試しに再起化してみたら処理速度倍になった。
自分の環境では3分ちょいで解ける。
相変わらずメモリ馬鹿食いするけど。
もう俺には無理。

俺の中では終了でーす。Orz

391 :デフォルトの名無しさん:2017/07/22(土) 08:54:36.28 ID:OQXA8cUK.net
>>388
数学で頑張ってる人だけど、
もうちょっとまって

>>296の問題だけなら簡単だけど、
まだ全体を解明できてない

というか、忙しくて>>381から進んでない

392 :デフォルトの名無しさん:2017/07/22(土) 08:55:28.19 ID:OQXA8cUK.net
このスレが無くならないうちに解明します

393 :デフォルトの名無しさん:2017/07/22(土) 10:43:30.03 ID:apsnR2Z0.net
>>391
wktkデス!
コード見るのが好きなのでぜひ完走していただけたらと思います。

394 :デフォルトの名無しさん:2017/07/23(日) 11:26:56.94 ID:ipiEUPYV.net
>>375
のほかの実装はでてこないねぇ‥

395 :デフォルトの名無しさん:2017/07/23(日) 12:53:55.68 ID:7fREas1L.net
>>394
使えるコードにするためには、規模がでかくなりすぎるから

396 :デフォルトの名無しさん:2017/07/23(日) 14:15:20.13 ID:ipiEUPYV.net
C/C++ で最長1000行ぐらいとみて、2日ぐらいあれば、とりあえず動く
土日で仕上がってくるんじゃないかと期待してたんだが

397 :デフォルトの名無しさん:2017/07/23(日) 14:18:11.58 ID:7fREas1L.net
速度が考えられてないコードなんて実用にはならないよ

398 :デフォルトの名無しさん:2017/07/23(日) 14:21:31.79 ID:7fREas1L.net
ていうか、
コードに対する条件とか
サポートする機能とか
条件が無さすぎる

399 :デフォルトの名無しさん:2017/07/23(日) 14:24:57.75 ID:ipiEUPYV.net
速度‥か‥

どうしてもローテートとかキャリーフラグとかを使いたいから、これはアセンブラの領域になるね
よくみかけるアセンブラ中毒者が今頃爪を研いでいるのだろうか?

400 :デフォルトの名無しさん:2017/07/23(日) 14:25:42.29 ID:ipiEUPYV.net
>>398
そこは「自分用」だから自由に決めていいんでないかい?

401 :デフォルトの名無しさん:2017/07/23(日) 14:49:34.77 ID:TcY6qE9r.net
>>394

当日に >>305>>390 より10倍以上早いのがでているだろう。
しかも 計算量まで書いてある

402 :デフォルトの名無しさん:2017/07/23(日) 14:53:19.94 ID:ipiEUPYV.net
>>401
お題が違うのでは?

403 :デフォルトの名無しさん:2017/07/23(日) 15:05:28.62 ID:TcY6qE9r.net
>>402 >>394
確かに違った、すいません。

c++多倍長なら、karatsubaにも対応して300行くらいの以下をパクって使うのも一案

http://sites.google.com/site/indy256/algo_cpp/bigint

404 :デフォルトの名無しさん:2017/07/23(日) 15:09:21.91 ID:ipiEUPYV.net
>>403
base 10進ならば、表示(operator<<) が楽でいいね、なるほど、それは思いつかなかった

405 :デフォルトの名無しさん:2017/07/23(日) 16:51:44.88 ID:7fREas1L.net
>>399
通常、
処理時間のほとんどが乗算
乗算のほとんどがFFT
アセンブラの出番は当分先

406 :デフォルトの名無しさん:2017/07/23(日) 16:52:43.21 ID:7fREas1L.net
FFTのライブラリをどこからか持ってくるのでもいいけど、
それなら素直に多倍長ライブラリを持ってくれば
ってことになる

407 :デフォルトの名無しさん:2017/07/23(日) 16:54:07.19 ID:7fREas1L.net
今は浮動小数点演算が速いので、
カラツバの出番はあまりない

408 :デフォルトの名無しさん:2017/07/23(日) 18:50:17.74 ID:5CSy1R8t.net
基数を10のべき乗にするとか(printf()的なものが簡単だから)、乗算はunsigned shortやintとの
乗算に限るとか、除算無しとかいうのは…

プログラムの本体に組み込まれてしまって、再利用可能なライブラリの形で括りだされてる事の
方が少ないかw

409 :デフォルトの名無しさん:2017/07/24(月) 00:48:44.80 ID:XgJeE+LA.net
>>6-285 SageMath
ttp://sagecell.sagemath.org/?q=veftjc

410 :デフォルトの名無しさん:2017/07/24(月) 18:24:00.06 ID:UuUAOyUA.net
>>408
裁定ラインとしては、乗算は Bigint×Bigint、および除算の実装ですかね、でも足し算の回数での乗算や引き算の回数での除算は嫌ですね

411 :デフォルトの名無しさん:2017/07/24(月) 18:58:12.05 ID:5ve8i6tz.net
お題:お題スレ3の>>170をファレイ数列を使って解く。
http://peace.2ch.net/test/read.cgi/tech/1390525149/170

412 :デフォルトの名無しさん:2017/07/24(月) 19:10:35.21 ID:nJVItCRy.net
>>411 Ruby
def farey_sequence(n)
(1..n-1).map{|i| 1r*i/n}
end

def ans_411(m)
(2..m).map{|i| farey_sequence(i)}.flatten.uniq.sort
end

ans_411 3 #=> [(1/3), (1/2), (2/3)]
ans_411 5 #=> [(1/5), (1/4), (1/3), (2/5), (1/2), (3/5), (2/3), (3/4), (4/5)]

413 :デフォルトの名無しさん:2017/07/24(月) 19:11:05.16 ID:7nQ6Z7f9.net
>>296
超高速版が出来ました!
http://ideone.com/FrRkof

一皿9秒が上限であれば、計算オーダーはn
関数自体は何秒でも大丈夫です

コードだけじゃ意味がわからないでしょうけど、とりあえずコードだけ
あまりテストしてないので、バグってたらごめんなさい

414 :デフォルトの名無しさん:2017/07/24(月) 23:00:47.69 ID:fjGi9Yh0.net
オーダーnは凄いな

415 :デフォルトの名無しさん:2017/07/25(火) 05:40:32.49 ID:ubbfnjuS.net
>>413
うーんわからん。
俺の思考とは別系統かな。
ホントに0秒で解けてるし、素晴らしい。
素直に賞賛。

416 :デフォルトの名無しさん:2017/07/25(火) 11:52:16.57 ID:bLUUDw7G.net
回転寿しの問題は、部分的な最短経路が全体の最短経路にならないんだよな
だが最短時間はレーン長の2倍程度の再帰回数で出る
そのあと数十億回再帰して総当たりしてもそれより短くならない
最後の皿から逆方向に探索してもおそらく同じ状況

417 :デフォルトの名無しさん:2017/07/25(火) 11:56:47.84 ID:bLUUDw7G.net
例えば、”122” は最短時間6だが、1周目で2番目の要素”2”をパスしないとそうならない

418 :411:2017/07/26(水) 19:54:35.63 ID:6H34MdHA.net
>>412
ファレイ数列の中間数(mediant)を再帰的に生成すると、uniqもsortも要らないのだけど、
mが3や5だと大差無いかw

419 :デフォルトの名無しさん:2017/07/26(水) 20:50:49.45 ID:s8dUUqTb.net
>>411
リンク先が見えません
問題文をもう一回書いてください

420 :デフォルトの名無しさん:2017/07/26(水) 20:52:34.29 ID:s8dUUqTb.net
と思ったら見れました

ファレイ数列を使って何かを解くわけじゃなくて、
ファレイ数列を求める問題?

421 :411:2017/07/26(水) 23:20:07.89 ID:6H34MdHA.net
>>420
元の問題はそういうもの(=ファレイ数列の両端(0/1と1/1)無し版を求める問題)と
解釈してますです。

422 :デフォルトの名無しさん:2017/07/26(水) 23:26:01.52 ID:lPM9zwS7.net
#include <list>
#include <iostream>
const int N_MAX = 10;
struct RATIONAL {
int num;
int den;
};
int main() {
std::list < RATIONAL > farey;
RATIONAL zero = {0, 1};
RATIONAL one = {1, 1};
farey.push_back(zero);
farey.push_back(one);
for (int n = 1; n <= N_MAX; n++){
for (std::list < RATIONAL > ::iterator i1 = farey.begin(), i0 = i1++; i1 != farey.end(); i0 = i1, i1++) {
if (i0->den + i1->den <= n) {
RATIONAL m = {i0->num + i1->num, i0->den + i1->den};
farey.insert(i1, m);
}
}
std::cout << n << " : ";
for (std::list < RATIONAL > ::iterator i = farey.begin(); i != farey.end(); i++) {
std::cout << i->num << "/" << i->den << " ";
}
std::cout << "\n";
}
return 0;
}

423 :デフォルトの名無しさん:2017/07/26(水) 23:29:22.49 ID:lPM9zwS7.net
これから0と1を除けば良いって問題であれば、
表示のループに以下を加えれば
if (i->den != 1)

424 :デフォルトの名無しさん:2017/07/26(水) 23:31:57.11 ID:lPM9zwS7.net
問題の意味も意図も良くわからん

出題者が「そういうものと解釈しています」とか
出題者が >>418 みたいな回答をバカにする発言とか

なんか非常に感じが悪い

425 :412:2017/07/27(木) 00:12:38.86 ID:qteH6K3e.net
そもそも>>412のfarey_sequenceは定義が間違ってたわ
んでもって再帰にすると>>412より遅くなるという
Ruby

class Farey
def self.[](m)
if m == 1
[0r, 1r]
else
succ(m - 1)
end
end

def self.succ(m)
self[m].each_cons(2).inject([0r]){|s, (a, b)|
x = a.denominator + b.denominator
s << 1r*(a.numerator + b.numerator)/x if x == m + 1
s << b
}
end
end

Farey[3] # => [(0/1), (1/3), (1/2), (2/3), (1/1)]
Farey[5] # => [(0/1), (1/5), (1/4), (1/3), (2/5), (1/2), (3/5), (2/3), (3/4), (4/5), (1/1)]

426 :デフォルトの名無しさん:2017/07/27(木) 01:59:46.61 ID:GuEy9AL1.net
>>411 Java
https://ideone.com/w0q7cN

427 :デフォルトの名無しさん:2017/07/28(金) 18:51:16.80 ID:XBSdfIgC.net
>>375
のほかの実装はでてこないねぇ‥

428 :デフォルトの名無しさん:2017/07/28(金) 19:19:26.77 ID:mqZJq6H+.net
昔brainf**kで実装したのあるけどちょっとなぁ

429 :デフォルトの名無しさん:2017/07/28(金) 19:24:55.65 ID:WViVOgsq.net
また懐かしい言語を

430 :デフォルトの名無しさん:2017/07/28(金) 19:26:36.86 ID:WViVOgsq.net
どうせならチューリングマシンで作ってよ

431 :デフォルトの名無しさん:2017/07/30(日) 10:59:37.37 ID:A7gIx2b1.net
お題:MathematicaのFareySequence[n,k](引数2つ)に相当するものの実装
ttp://reference.wolfram.com/language/ref/FareySequence.html

432 :デフォルトの名無しさん:2017/07/30(日) 11:53:03.31 ID:EQKnHSgY.net
>>431
http://ideone.com/m7BnJN
C++。一瞬計算が合わなくてビビったけど、空目だった。
インデックス概念がベーシックなんだな。

433 :デフォルトの名無しさん:2017/07/30(日) 12:00:36.94 ID:EQKnHSgY.net
っていうか、この関数インデックスに0与えたら何が出力されるんだろう・・・。
早速バグってる気がする。

434 :デフォルトの名無しさん:2017/07/30(日) 12:15:40.85 ID:EQKnHSgY.net
>>432
バグってた。のでエディトしてFIXした。
所持する数の概念勘違いしてた。

435 :デフォルトの名無しさん:2017/07/30(日) 12:25:59.30 ID:B3p9Yl5S.net
>>422の微妙な変更でいいよね

436 :デフォルトの名無しさん:2017/07/30(日) 12:26:41.34 ID:B3p9Yl5S.net
1個だけ求めるなら、もっといい方法がある?

437 :デフォルトの名無しさん:2017/07/30(日) 12:27:22.69 ID:B3p9Yl5S.net
ていうか、いい加減Fareyはもういいでしょ
他の課題の方が

438 :デフォルトの名無しさん:2017/07/30(日) 12:30:15.15 ID:EQKnHSgY.net
フィボナッチって素数何だっけ?

439 :デフォルトの名無しさん:2017/07/30(日) 12:44:33.08 ID:B3p9Yl5S.net
1, 1, 2, 3, 5, 8, ...
違うよね

440 :デフォルトの名無しさん:2017/07/30(日) 12:47:15.18 ID:EQKnHSgY.net
だよねー。>>422ってフィボナッチ使ってない?
あんまり深く考えてないだけど。Orz

441 :デフォルトの名無しさん:2017/07/30(日) 12:47:30.85 ID:B3p9Yl5S.net
じゃあ、任意の二個の数からはじまるフィボナッチ数列で、はじめから連続する素数の数が多い物を探す
って課題で

442 :デフォルトの名無しさん:2017/07/30(日) 12:48:36.68 ID:B3p9Yl5S.net
>>440
フィボナッチではない
wikipediaにのってるレベルの知識で作った

443 :デフォルトの名無しさん:2017/07/30(日) 12:49:05.29 ID:EQKnHSgY.net
あれ?俺とんちんかんなこと言ってるか?
>>422が数列としてあってるのかよくわからない。Orz
どう考えればいいんだろう。

444 :デフォルトの名無しさん:2017/07/30(日) 12:53:04.00 ID:EQKnHSgY.net
まぁ、俺のもあってるかどうかはしらんけど。><;

445 :デフォルトの名無しさん:2017/07/30(日) 12:56:43.60 ID:EQKnHSgY.net
頭悪くてゴメン。爆発しそう。。。

446 :デフォルトの名無しさん:2017/07/30(日) 13:01:05.96 ID:EQKnHSgY.net
引っ込む。すまんかった。

447 :デフォルトの名無しさん:2017/07/30(日) 14:17:22.65 ID:t+CfDp82.net
>>431 Java
http://ideone.com/9AXdRV

448 :デフォルトの名無しさん:2017/07/30(日) 19:16:10.29 ID:LizATlBz.net
>>431 Ruby

def farey(n, k)
return [0r, 1r][k] if n == 1
farey(n - 1, 0..-1).each_cons(2).inject([0r]){|s, (a, b)|
x = a.denominator + b.denominator
s << 1r*(a.numerator + b.numerator)/x if x == n
s << b
}[k]
end

449 :デフォルトの名無しさん:2017/08/03(木) 07:36:01.80 ID:cLWzUq7C.net
お題:ミンコフスキーの疑問符関数の実装
ttps://en.wikipedia.org/wiki/Minkowski%27s_question_mark_function
ttp://reference.wolfram.com/language/ref/MinkowskiQuestionMark.html

450 :デフォルトの名無しさん:2017/08/03(木) 10:39:36.15 ID:ONmyLPuf.net
WIKIぺにコード乗ってますが。

451 :デフォルトの名無しさん:2017/08/03(木) 10:48:34.50 ID:ONmyLPuf.net
>>449のWIKIより。
/* Minkowski's question mark function */
double minkowski(double x) {
long p=x; if ((double)p>x) --p; /* p=floor(x) */
long q=1, r=p+1, s=1, m, n;
double d=1, y=p;
if (x<(double)p||(p<0)^(r<=0)) return x; /* out of range ?(x) =~ x */
for (;;) /* invariants: q*r-p*s==1 && (double)p/q <= x && x < (double)r/s */
{
d/=2; if (y+d==y) break; /* reached max possible precision */
m=p+r; if ((m<0)^(p<0)) break; /* sum overflowed */
n=q+s; if (n<0) break; /* sum overflowed */

if (x<(double)m/n) r=m, s=n;
else y+=d, p=m, q=n;
}
return y+d; /* final round-off */
}

452 :デフォルトの名無しさん:2017/08/05(土) 17:44:11.83 ID:40G0sflG.net
>>375
のほかの実装はでてこないねぇ‥

453 :デフォルトの名無しさん:2017/08/12(土) 18:46:00.57 ID:953va2dM.net
寿司のオーダーNのやつを理解しようとしたけどまだやってない。
その仕組みと、ほんとに正解してるのかとか。いたら誰が解説して。

454 :デフォルトの名無しさん:2017/08/12(土) 19:04:22.18 ID:Bi4KH0eW.net
>>413です

もちろん合っているつもりのコードです
作者が言っても何の説得力もありませんが

455 :デフォルトの名無しさん:2017/08/12(土) 19:07:04.34 ID:4r/z/Qd5.net
会社に帰ってこない巡回セールスマンだよね
寿司の乗った皿がノード、計算量はO(n!)

456 :デフォルトの名無しさん:2017/08/12(土) 19:10:18.10 ID:Bi4KH0eW.net
それぞれの寿司を食べている期間をレーン上の線分で表します

この線の重なり具合をpileで表しました

効率良く食べられた場合はレーンがpile_max周するまでの間に食べきることが出来ます

170行目の判定がそれで、trueの場合は効率良く食べられない場合です

457 :デフォルトの名無しさん:2017/08/12(土) 19:12:06.32 ID:4r/z/Qd5.net
>>456
もしそれで最適解が得られるなら巡回セールスマンも可能じゃないかな?

458 :デフォルトの名無しさん:2017/08/12(土) 19:17:11.73 ID:6XNTCj+p.net
巡回セールスマン問題とけたら色々応用範囲アルヨ。
マジでどっかに売り込んでもいいくらい。
天才か。

459 :デフォルトの名無しさん:2017/08/12(土) 19:18:34.85 ID:6XNTCj+p.net
社会的に言うと交通統制とかもそれじゃないかな?
信号の待ち時間問題。よくしらんけど。

460 :デフォルトの名無しさん:2017/08/12(土) 19:19:17.76 ID:Bi4KH0eW.net
効率良く食べられない方が簡単なのでその場合から

お寿司を以下のグループに分けます
----
各グループのお寿司は、レーンの特定の位置から食べ始めた場合、pile[グループ]周以内で食べ終わることが出来る
このとき、pile_max = Σ pile[グループ]
となる
---
このようなグループの分け方の最小の物が存在します

461 :デフォルトの名無しさん:2017/08/12(土) 19:22:56.16 ID:Bi4KH0eW.net
同じグループのお寿司は連続して食べます
開始時と、各グループのお寿司を食べ終わった後、最初に来るお寿司から食べはじめ、pile[グループ]以内で食べられる食べ方でそのグループを食べ終える
ということを繰り返せば最小の時間で食べ終えることが出来ます

462 :デフォルトの名無しさん:2017/08/12(土) 19:26:29.79 ID:Bi4KH0eW.net
グループ分けした時に1個のグループになった場合は、
効率良く食べられることになります
つまり、pile_max周以下で食べ終えることが出来ます

この時は、コード上にあるダミーのお寿司を追加してから最小時間を求め、ダミーのお寿司を食べてる時間を引けば求められます

463 :デフォルトの名無しさん:2017/08/12(土) 19:28:18.79 ID:4r/z/Qd5.net
うーん、よくわからん
セールスマンの巡回先を一次元にマッピングできれば同じことできそうな
無理か

464 :デフォルトの名無しさん:2017/08/12(土) 19:30:27.01 ID:Bi4KH0eW.net
グループの分け方は少し難しいです

レーンの各整数位置に対して、
お寿司の線の両端にあたる点同士
線の重なりがpile_max未満である区間の点(両端を含む)
を同じグループの点とし、
これらを続けることで最小のグループ分けが出来ます
線の両端の点のグループが、そのお寿司のグループになります

465 :デフォルトの名無しさん:2017/08/12(土) 19:31:42.19 ID:Bi4KH0eW.net
それぞれ、証明は出来ているつもりです

466 :デフォルトの名無しさん:2017/08/12(土) 19:32:49.51 ID:Bi4KH0eW.net
もちろん、一般の巡回問題はこの方法では無理です

467 :デフォルトの名無しさん:2017/08/12(土) 19:37:29.23 ID:4r/z/Qd5.net
全ノードを巡回する最短時間の問題だから、できそうな気がするけどね

468 :デフォルトの名無しさん:2017/08/12(土) 19:39:44.61 ID:2Yw2XYfL.net
372仕様書無しさん2017/08/11(金) 10:31:43.41
フリーランスで検索すると引っかかる零細ITがやっているフリーランスのサイトはだめだ。
高額に見せているけど実際は50万前後
JIET加入した方がいいよ。案件は毎日千件以上末端価格は60万円 平凡な稼働時間の80万円の案件もある。
ユー子も求人をだしてる。名刺も渡せる。ユー子に名刺が渡せるんだぞ。夢のようだ

それらの案件まさぐってHPで転売していたのが零細ITがやるフリーランスサイト

473非決定性名無しさん2017/08/03(木) 15:21:30.71

JIETに加入すれば誰でも3次60万からスタートだ。フリーランスのサイトをやってる
自称エージェントもそこから案件情報を取得しきてる。サイトで60万で釣って40万から55万の
間でやらしている。

446非決定性名無しさん2017/08/02(水) 22:12:48.95

JIETに毎月5千円払えば3次から入場できるだろ?
高額をうたうフリーランスのサイトはだいたい5次から45万円
JIETで閲覧応募できる末端価格からさらに搾取するのが高額をみせつけるフリーランスサイトでした
高額案件をみせつけるフリーランスサイトも案件の取得はJIETでした

自称エージェントはJIETから流れてくる案件を転売してるだけだった。
JIETに加入すれば誰でも案件に応募することができた。収入が40万50万台にならなくて済む

469 :デフォルトの名無しさん:2017/08/12(土) 19:40:20.63 ID:Bi4KH0eW.net
pile_maxとその位置から下限が得られますが、
>>296 の例では98秒の物以外はすべてその下限になっています
一個その下限になるような例を見つければ答えがわかるのですが、
自力で検索してみればわかると思いますがそのような例はあっさり見つかります

98秒の例は効率良く食べられない場合になります

効率良く食べられる側のなかでも、pileから得られる下限値より大きくなる場合もあります

470 :デフォルトの名無しさん:2017/08/12(土) 19:43:06.99 ID:Bi4KH0eW.net
いずれの場合も、PCを使わなくても手計算で十分可能です

471 :デフォルトの名無しさん:2017/09/15(金) 10:14:33.05 ID:lRMsxOf0.net
お題:
N次元で1辺のマス目がM個の魔法陣を作る
N>3(任意)、M>=3(任意)の超立方体

472 :デフォルトの名無しさん:2017/09/15(金) 10:20:33.77 ID:lRMsxOf0.net
参考
http://magcube.la.coocan.jp/magcube/jp/
http://nadamath2012.web.fc2.com/bushi/2004_mas2.pdf

473 :デフォルトの名無しさん:2017/09/17(日) 16:38:38.03 ID:DSKC3zx4.net
魔方陣は1個作ればいいの?

Mが奇数か4の倍数は簡単
4で割って2余るのは検索するしかないのかな?

474 :デフォルトの名無しさん:2017/09/17(日) 16:55:04.37 ID:fthJj6jv.net
バックトラックで組もうかと思ったけど、重そうだったからやめた。
数独より重そう。
それに一列合計をどの数字にするのかちょっとわからなかった。

475 :デフォルトの名無しさん:2017/09/17(日) 23:20:36.13 ID:DSKC3zx4.net
一列合計は、M*[数字の平均]
になる

つまり
M*(M^N+1)/2

476 :片山博文MZ :2017/09/18(月) 21:53:54.06 ID:iMidYxoH.net
お題: URLから適当なサムネイルを生成するWebプログラム。

477 :デフォルトの名無しさん:2017/09/18(月) 23:06:01.10 ID:FC5+Wne9.net
お題
0以上90未満の整数nを入力として
タンジェントn°の値が有理数ならば真
そうでなければ偽を返す

478 :デフォルトの名無しさん:2017/09/18(月) 23:29:46.49 ID:45aelXxs.net
bool f(int n){return n==0 || n == 45;}

479 :デフォルトの名無しさん:2017/09/18(月) 23:33:52.62 ID:ILsR+BHw.net
sed -r -e "s/^(0|45)\$/True/" -e "s/[1-8][0-9]*/False/"

480 :デフォルトの名無しさん:2017/09/19(火) 01:13:30.78 ID:zMNLdsjY.net
tanの計算しないのはどうかと

481 :デフォルトの名無しさん:2017/09/19(火) 01:57:23.24 ID:Ten4kOds.net
計算で有理数かどうか確認?
それは非常に難しいな

by 東大数学科卒

482 :デフォルトの名無しさん:2017/09/19(火) 02:28:36.10 ID:SyuGyzWY.net
>>480
そう思うなら他者を批判するより行動で示せばいいと思うよ

483 :デフォルトの名無しさん:2017/09/19(火) 03:58:59.82 ID:KVkpgN/c.net
tan1°が無理数であることの証明すら面倒くせえのに一体どんな回答を求めているんだ

484 :デフォルトの名無しさん:2017/09/19(火) 06:37:25.45 ID:Ten4kOds.net
>>483
面倒くさい?
てことは出来るの?
やってみて

485 :デフォルトの名無しさん:2017/09/19(火) 07:41:26.64 ID:KVkpgN/c.net
>>484
確か京大の過去問にあったでしょう
説明めんどいからは解法は自分で調べて

486 :デフォルトの名無しさん:2017/09/19(火) 08:13:59.26 ID:Ten4kOds.net
いや、おれは出来るよ

>>483の実力で出来るのか?と疑問に思っただけ
実力じゃなくてカンニングね

487 :デフォルトの名無しさん:2017/09/19(火) 08:21:42.19 ID:KVkpgN/c.net
何言ってんだこいつ

488 :デフォルトの名無しさん:2017/09/19(火) 08:49:35.38 ID:q1kL6yRz.net
問題が悪いな

与えられた有理数rに対し、
tan(πr)が有理数かどうか判別するプログラムを書け

ならテーブルは使えない

489 :デフォルトの名無しさん:2017/09/19(火) 09:06:24.19 ID:emxMAzY1.net
また、多倍長精度演算のないC++にはきつい問題を・・・。

490 :デフォルトの名無しさん:2017/09/19(火) 11:09:01.37 ID:q1kL6yRz.net
>>488
すいません
結果を知っていたらこれでも簡単でした

491 :デフォルトの名無しさん:2017/09/19(火) 11:10:19.22 ID:q1kL6yRz.net
>>489
多倍長を使っても解決しないでしょ

492 :デフォルトの名無しさん:2017/09/19(火) 12:53:02.28 ID:RSOddfRB.net
そもそも出題者はどういう回答を期待してるんだ?
数学の知識無しでは作れないし、数学の知識を使えば>>478になる

493 :デフォルトの名無しさん:2017/09/19(火) 14:38:36.12 ID:LvSRuVZD.net
tan()の加法定理
 tan(α+β)=(tanα+tanβ)/(1-tanαtanβ)
により

もしtan(α)が有理数なら
tan(nα) (n = 1,2,3,4・・・)
も全て有理数

このため
整数nにより
tan(n)が無理数なら
nの約数全てによるtan()が無理数

ここで
tan(60)=√3
が無理数なのは簡単に証明されるため、
tan(1)
も無理数

証明終わり

494 :デフォルトの名無しさん:2017/09/19(火) 14:54:56.80 ID:RSOddfRB.net
>>476を解くにはあとtan(18度)が無理数であることを証明しないと

495 :デフォルトの名無しさん:2017/09/19(火) 14:55:28.25 ID:RSOddfRB.net
>>476じゃなくて>>477

496 :デフォルトの名無しさん:2017/09/19(火) 16:11:41.85 ID:HSXd4/vW.net
>>493
なるほど面白いねw

497 :デフォルトの名無しさん:2017/09/19(火) 19:41:00.46 ID:Ten4kOds.net
>>493
tan(π/4)は有理数だけど
tan(π/2)は有理数じゃない

498 :デフォルトの名無しさん:2017/09/19(火) 20:13:34.35 ID:KVkpgN/c.net
tan1(rad)が超越数であることを証明せよ

499 :デフォルトの名無しさん:2017/09/19(火) 22:25:08.25 ID:FbLYus+p.net
>>492
WolframAlphaが
is tan(pi * 1 / 180) a rational number?
→ not a rational number
と返す仕組みを知りたかった

500 :デフォルトの名無しさん:2017/09/19(火) 22:57:34.84 ID:Ten4kOds.net
xが有理数、tan(πx)が有理数 ====> xは1/4の倍数

って覚えてるだけかと

501 :デフォルトの名無しさん:2017/09/20(水) 14:48:00.57 ID:jgmli1ek.net
>>493
は加法定理で(1-tanαtanβ)が0になってはまずいので
0度以上90未満の範囲内に限定しないといけないな。


tan()の加法定理
 tan(α+β)=(tanα+tanβ)/(1-tanαtanβ)
により

もしtan(α)が有理数で、かつ 0 <= nα < 90なら
tan(nα) (n = 1,2,3,4・・・)
も全て有理数

このため
整数 n ( 0 <= n < 90 ) により
tan(n)が無理数なら
nの約数全てによるtan()が無理数

ここで
tan(60)=√3
が無理数なのは簡単に証明されるため、
tan(1)
も無理数

502 :デフォルトの名無しさん:2017/09/20(水) 14:51:06.46 ID:jgmli1ek.net
tan(1)だけじゃなくて
>>477 >>478 も証明できるかな???

つまり整数 n ( 0 <= n < 90 ) において
tan(n)が有理数になるのはn=0,45に限ることの証明

tan(90-n) = 1/tan(n) なので
n ( 0 <= n < 45 ) の範囲で証明されればOK

またtan(45)が有理数で加法定理で減算し
 tan(45-n):有理数 ⇔ tan(n):有理数 ( 0 <= n < 45 )
も成立

503 :デフォルトの名無しさん:2017/09/20(水) 14:51:42.84 ID:jgmli1ek.net
60の約数 はtan(n)無理数
1,2,3,4,5,6,10,12,15,20,30

これの45-n もtan(n)無理数
44,43,42,41,40,39,35,33,25,15

この約数で、まだ含まれていないもの
11,22,21,8,13,7

45-nにより
34,23,24,37,32,38

この約数で、まだ含まれていないもの
17,16,19

45-nにより
28,29,26

この約数で、まだ含まれていないもの
14

45-nにより
31

ここまでの数を並べると
01,02,03,04,05,06,07,08,**,10,
11,12,13,14,15,16,17,**,19,20,
21,22,23,24,25,26,**,28,29,30,
31,32,33,34,35,**,37,38,39,40,
41,42,43,44

9度の倍数の証明のみが残された

504 :デフォルトの名無しさん:2017/09/20(水) 16:48:28.32 ID:UU/UGcdT.net
だから>>494と書いたんだけど

505 :デフォルトの名無しさん:2017/09/20(水) 21:20:48.49 ID:8kWE0pQL.net
tan(1 rad)が超越数であることは誰も証明できないの

506 :デフォルトの名無しさん:2017/09/20(水) 21:27:13.08 ID:UU/UGcdT.net
プログラムに証明させる問題?

507 :デフォルトの名無しさん:2017/09/20(水) 22:23:37.78 ID:vEoThqNS.net
なぜラジアン?
話の流れ的にはtan(1度)だろ

508 :デフォルトの名無しさん:2017/09/20(水) 22:25:37.42 ID:vEoThqNS.net
と思ったけど、簡単すぎた

509 :デフォルトの名無しさん:2017/09/21(木) 16:21:06.33 ID:na02B6ss.net
[1] 授業単元名:FizzBuzzクイズ
[2] 問題文(含コード&リンク):
[3] 環境
 [3.1] OS: (Windows/Linux/等々)特に問わない
 [3.2] コンパイラ名とバージョン: (gcc 3.4 VC 6.0等)特に問わない
 [3.3] 言語: (C/C++/どちらでも可 のいずれか)特に問わない

http://kohada.2ch.net/test/read.cgi/prog/1209467166/401
FizzBuzzクイズ

1.fizz.buzz #=> 1
3.fizz.buzz #=> "Fizz"
5.fizz.buzz #=> "Buzz"
15.fizz.buzz #=> "FizzBuzz"
999.fizz.buzz #=> 999

となるようなメソッドfizz、buzzは定義可能か?
可能である場合、同様にgizzを追加定義し、
7.fizz.buzz.gizz #=> "Gizz"
21.fizz.buzz.gizz #=> "FizzGizz"
35.fizz.buzz.gizz #=> "BuzzGizz"
105.fizz.buzz.gizz #=> "FizzBuzzGizz"
105.fizz.gizz.buzz #=> "FizzGizzBuzz" と拡張・応用ができるか?

メソッドのコールに()が必須の言語では 3.fizz().buzz() 形式でも構わない。
オープンクラス機構やメソッドのない言語では関数(buzz(fizz(3)) #=> "Fizz" など)で。

510 :デフォルトの名無しさん:2017/09/21(木) 19:58:51.31 ID:+ykHPOb/.net
まともに仕様を書けない出題者

511 :デフォルトの名無しさん:2017/09/22(金) 07:02:49.79 ID:aD9oWCn2.net
これ普通の発想では無理

512 :デフォルトの名無しさん:2017/09/22(金) 07:14:29.33 ID:eEMHecr4.net
>>509
修正
×999.fizz.buzz #=> 999
○997.fizz.buzz #=> 997

513 :デフォルトの名無しさん:2017/09/22(金) 07:55:33.84 ID:FtjqsiSd.net
>>509
C++版
https://ideone.com/JdTeeD

514 :デフォルトの名無しさん:2017/09/22(金) 08:00:39.43 ID:pX6TouLp.net
仕様が謎

515 :デフォルトの名無しさん:2017/09/22(金) 08:13:32.67 ID:FtjqsiSd.net
>>509
C言語
https://ideone.com/eWR8Pl

516 :デフォルトの名無しさん:2017/09/22(金) 09:43:07.90 ID:eeRMTLx0.net
外部出力を伴う関数(あるいはメソッド)なら簡単
たぶん関数(あるいはメソッド)の返値がそうなるようにって意味かと
(じゃないと普通に書けてクイズにならない)
たしか数理学的にはこういう関数は書けないことになっていたはず

517 :デフォルトの名無しさん:2017/09/22(金) 12:43:18.77 ID:qmG6L9xB.net
>>509>>516 みたいなのとは絶対に一緒に仕事をしたくない

518 :デフォルトの名無しさん:2017/09/22(金) 12:43:56.86 ID:qmG6L9xB.net
別に戻り値にしても大して変わらんけど

519 :デフォルトの名無しさん:2017/09/22(金) 12:51:02.25 ID:qmG6L9xB.net
C言語だとトリッキーな技を使わないと出来ない
同じ関数名で複数関数を作れないから
2段や3段重ねて、intを受けて文字列を返すのは普通には無理

C++だと簡単
大きく分けて2つの方法がある

C++でも数値によって戻り値の型を変えるのは無理
数値がconstexprで良いなら出来るだろうけど

520 :デフォルトの名無しさん:2017/09/22(金) 14:57:30.68 ID:eEMHecr4.net
>>513
>int l_ = 3
これ、なんとかならないか?

521 :デフォルトの名無しさん:2017/09/22(金) 16:30:21.56 ID:W1Y66+yK.net
>>520
関数インターフェース的にはその値を与えることが出来ない

別の関数か何かで変えるかインターフェースを変えるしか

522 :デフォルトの名無しさん:2017/09/22(金) 16:41:43.04 ID:W1Y66+yK.net
>>516
戻り値を文字列にする方法

方法1
段階によって引数と戻り値の型を変える
S1 fizz(int n);
S2 fizz(S1 s);
std::string fizz(S2 s);
※テンプレートを使うと楽

方法2
戻り値をstd::string固定にしてなんとかする

方法2-1
戻り値は常に結果の文字列にし、パラメーター以外で情報を渡す
方法2-2
文字列に情報をエンコードして入れる
最終型段だけ結果を返すようにする

方法3
戻り値を結果文字列そのままではなく、文字列情報を含む情報とする
(これは反則かな?)

523 :デフォルトの名無しさん:2017/09/22(金) 16:44:03.80 ID:W1Y66+yK.net
方法2-1であれば >>520の問題は解決する
ただし、そのままだとスレッドセーフじゃなくて気持ち悪い

524 :デフォルトの名無しさん:2017/09/22(金) 16:48:22.63 ID:eEMHecr4.net
>>509
#=>
これって ruby の記法だったかな?評価値を変えたいようだ

525 :デフォルトの名無しさん:2017/09/22(金) 18:17:47.82 ID:eeRMTLx0.net
例えばRubyだと文字列を含め組み込み型にインスタンス変数を仕込めるので
たぶんそれで次のメソッドに情報を渡せる

526 :デフォルトの名無しさん:2017/09/22(金) 19:04:29.86 ID:FtjqsiSd.net
>>522 の方法2-1
C++版
https://ideone.com/ZavKGg

外部情報は「n」のみ
複数スレッドや割り込みハンドラからコールする時はこのnが問題になるんで
なんとかしてstd::stringに埋め込めれば良いんだけど

>>525
問題を変えちゃダメだよね
> [3.3] 言語: (C/C++/どちらでも可 のいずれか)特に問わない

527 :デフォルトの名無しさん:2017/09/22(金) 19:08:43.22 ID:FtjqsiSd.net
8行目、なんとなく文字列から判別してみたけど、
素直にnと同じように外部にフラグを持てば条件が減る
(文字列の最後が数字にならないとか文字コードが連続してるとか)

528 :デフォルトの名無しさん:2017/09/22(金) 19:09:11.03 ID:pX6TouLp.net
「(C/C++/どちらでも可 のいずれか)特に問わない」って日本語がまず謎
有限個の具体例しか与えられていないので仕様も謎

529 :デフォルトの名無しさん:2017/09/22(金) 19:26:24.34 ID:FtjqsiSd.net
出題者の選択枝が [C/C++/どちらでも可] の3個あって、出題者がその「いずれか」を選ぶ
というフォーマットを使った出題
出題者は回答者に対し『その3個のどれでも良いよ』という意味で「特に問わない」と

と私は解釈した
つまり、回答者の選択枝はCかC++のどちらかだと

出力する文字列のルールはリンク先を見れば大体わかる
gizzが7の倍数かどうかは実際には不明で、実は14で割ると7余る数かもしれないが...

リンク先に「プリントする」とあるので
printfなどで標準出力に出せば良いのかと思ったが、
>>516の解釈は違うらしい

数値の場合だけ""でくくってないので、
文字列の場合は""をくっつける必要があるのか、
型を変えろと言っているのかはよくわからん
いずれにしろ、CやC++では値によって戻り値の型を変えるのは不可能

530 :デフォルトの名無しさん:2017/09/22(金) 19:32:11.08 ID:FtjqsiSd.net
いずれにしろ、
>>510 >>514 >>517

531 :デフォルトの名無しさん:2017/09/22(金) 19:33:47.07 ID:eEMHecr4.net
>>529
>回答者の選択枝はCかC++のどちらかだと
いや、そうじゃなくて、本当に「特に問わない」、どういったらいいかな、このテンプレはC宿題スレのものを、そのまま頂戴しただけじゃない?

532 :デフォルトの名無しさん:2017/09/22(金) 19:36:43.23 ID:FtjqsiSd.net
>>531
君は出題者なのか?違うのか?
立場をはっきりと

533 :デフォルトの名無しさん:2017/09/22(金) 19:39:18.64 ID:FtjqsiSd.net
出題者なら0点

534 :デフォルトの名無しさん:2017/09/22(金) 19:43:00.43 ID:pX6TouLp.net
>>529
なるほど
.gizzは、与えられた数字に対してfizzbuzzが数字になるなら"Gizz"、
それ以外の場合は所定の位置(説明省くけど)に"Gizz"を挿入するものなのかと思ってたわ

535 :デフォルトの名無しさん:2017/09/22(金) 19:51:22.72 ID:FtjqsiSd.net
確かに
7の倍数じゃなくて1の倍数でも良いよな
たまたま >>509 の例がすべて7の倍数になってただけで

この場合、上にあげた3個のコードいずれも
7 を 1 に変えてくださいな

536 :デフォルトの名無しさん:2017/09/22(金) 19:54:43.95 ID:FtjqsiSd.net
そろそろ
出題者の模範解答
よろしくね

537 :デフォルトの名無しさん:2017/09/22(金) 20:00:08.36 ID:eEMHecr4.net
え゛
0点の出題だしー模範解答の質も推して知るべし、なんじゃないでしょうか……:−)

538 :デフォルトの名無しさん:2017/09/22(金) 20:35:12.69 ID:aD9oWCn2.net
>>526
↓この但し書きがあるってことは、問題作成者(≠出題者)としてはC/C++限定とは考えてはいないだろう

> メソッドのコールに()が必須の言語では 3.fizz().buzz() 形式でも構わない。
> オープンクラス機構やメソッドのない言語では関数(buzz(fizz(3)) #=> "Fizz" など)で。

そもそもここで出題する時点で [3] の縛りは意味をなさないよ

539 :デフォルトの名無しさん:2017/09/22(金) 20:43:54.80 ID:W1Y66+yK.net
>>509に書いてある以上は、それに従うのが基本

540 :デフォルトの名無しさん:2017/09/22(金) 20:47:11.96 ID:W1Y66+yK.net
と思って私は回答しましたが、
他の人が他の解釈で回答することまで否定はしません

541 :デフォルトの名無しさん:2017/09/22(金) 20:47:48.19 ID:W1Y66+yK.net
ということで、
>>538 よろしく!

542 :デフォルトの名無しさん:2017/09/23(土) 03:26:09.79 ID:nBwtcNcI.net
>>509 Ruby >>525の方針で
https://ideone.com/B6E8ig

543 :デフォルトの名無しさん:2017/09/23(土) 05:00:15.00 ID:FxaWa0db.net
>>509 F#
https://ideone.com/svYTTM

544 :デフォルトの名無しさん:2017/09/23(土) 09:40:19.02 ID:9eQI4Qct.net
>>509
@Mathematica

https://ideone.com/oC0nu7

入力値(n)をリストにして次の関数に渡さないとダメポ

545 :デフォルトの名無しさん:2017/09/23(土) 11:10:41.17 ID:koNmB6po.net
PerlやPythonは?

546 :デフォルトの名無しさん:2017/09/23(土) 11:25:56.13 ID:7PRDVMsP.net
ネタバレになるけど
このクイズはグローバル変数を使えばそれで済んでしまうシンプルな話なんだけど、それをあえて
- 各言語の機能を熟知・駆使して、面白くしたりひと工夫したりする(たとえばスレッドセーフとか)
- 前者のしくみと、7の倍数のgizzの拡張に必要な追加を最小限にすることを両立させる
というポイントが楽しみどころなんじゃないかな

547 :デフォルトの名無しさん:2017/09/23(土) 11:29:36.36 ID:w6RxEhSu.net
>>509
https://ideone.com/4p67HE
C++。題意は満たしてないけど、自分が書くとこんな感じだな。
末尾判定難しい。

548 :デフォルトの名無しさん:2017/09/23(土) 17:45:33.07 ID:tyQvRaHd.net
>>509
>>536

>>524>>547 の方法を想定
Java: https://ideone.com/i8wMea

549 :デフォルトの名無しさん:2017/09/23(土) 17:47:47.85 ID:tyQvRaHd.net
× >>524 >>547
>>524 >>542

550 :デフォルトの名無しさん:2017/09/23(土) 18:26:18.44 ID:80k6Tqnu.net
関数の入出力の型が同一である必要がある
Cならintをchar*と解釈するわけにいかないから構造体だろう

551 :デフォルトの名無しさん:2017/09/23(土) 22:30:47.29 ID:PEiEI8OX.net
スレッドローカル変数で
書いてみている

552 :デフォルトの名無しさん:2017/09/23(土) 22:51:39.70 ID:PEiEI8OX.net
>>509 Squeak Smalltalk だけどなんとか >>547 っぽい方法で

| FizzBuzzQuiz |
FizzBuzzQuiz := Trait named: #FizzBuzzQuiz uses: #() category: 'FizzBuzz-Quiz'.
FizzBuzzQuiz compile: 'isDivisibleBy: m
  ^(Processor activeProcess environmentAt: #fbValue) isDivisibleBy: m'.
FizzBuzzQuiz compile: ', str Processor activeProcess environmentAt: #fbValue put: self. ^str'.

FizzBuzzQuiz compile: 'fizz ^(self isDivisibleBy: 3) ifTrue: [self, ''Fizz''] ifFalse: [self]'.
FizzBuzzQuiz compile: 'buzz ^(self isDivisibleBy: 5) ifTrue: [self, ''Buzz''] ifFalse: [self]'.

{Number. String} do: [:each | each uses: FizzBuzzQuiz].

1 fizz buzz. "=> 1 "
3 fizz buzz. "=> 'Fizz' "
5 fizz buzz. "=> 'Buzz' "
15 fizz buzz. "=> 'FizzBuzz' "
14 fizz buzz. "=> 14 "

FizzBuzzQuiz compile: 'gizz ^(self isDivisibleBy: 7) ifTrue: [self, ''Gizz''] ifFalse: [self]'.

7 fizz buzz gizz. "=> 'Gizz' "
21 fizz buzz gizz. "=> 'FizzGizz' "
35 fizz buzz gizz. "=> 'BuzzGizz' "
105 fizz buzz gizz. "=> 'FizzBuzzGizz' "
105 fizz gizz buzz. "=> 'FizzGizzBuzz' "

553 :デフォルトの名無しさん:2017/09/24(日) 08:25:43.37 ID:wOaJDXIV.net
>>552
×>>547 っぽい方法で → ○>>542 っぽい方法で

554 :デフォルトの名無しさん:2017/09/24(日) 08:49:01.13 ID:wOaJDXIV.net
>>509 Ruby >>542>>552 と同様の手法でリファイン
https://ideone.com/2EsoeT

555 :デフォルトの名無しさん:2017/09/25(月) 19:17:51.82 ID:WU5gUeBt.net
>>509 c
https://ideone.com/oAsiG2
・構造体つこうた
・gizzの「追加定義」については簡易解釈

556 :デフォルトの名無しさん:2017/10/15(日) 20:26:14.16 ID:12RNBD+4.net
過去問を眺めていたが、もっとお気楽な問題が多かったようですね
肩慣らし問題を一つ

問題
循環小数を有理数に直せ。
循環節は括弧をつかって表現する。


0.[555] = 5/9
0.3[33] = 1/3
12.[345] = 4111/333
1.2[34] = 611/495

557 :デフォルトの名無しさん:2017/10/15(日) 21:45:12.25 ID:IPnwHMWa.net
連分数を使うのかね

558 :デフォルトの名無しさん:2017/10/15(日) 23:12:59.97 ID:Y75uJW9z.net
>>556 Java
https://ideone.com/QkUwMu

559 :デフォルトの名無しさん:2017/10/17(火) 15:01:04.33 ID:qd6dTZ1I.net
循環小数は有理数な訳だが

560 :デフォルトの名無しさん:2017/10/17(火) 16:22:17.58 ID:kN20YVKE.net
0.[555] = 0.[5] = 5/9
0.3[33] = 0.[3] = 3/9
12.[345] = 12+345/999
1.2[34] = 1.2+34/990

561 :デフォルトの名無しさん:2017/10/17(火) 18:47:49.98 ID:ELG/Hivs.net
てすと

562 :デフォルトの名無しさん:2017/10/18(水) 11:15:59.21 ID:xwRaz5Kx.net
>>560
おお。そういう法則で行けるのか。
きっと数学では大昔に証明されてるんだろうけど知らなかった。(または忘れたのかなあ?)

563 :デフォルトの名無しさん:2017/10/18(水) 11:35:57.38 ID:bG8m3FQp.net
ああ。なんとなくわかった。10の桁数乗の値で割るとそっくりそのまま小数点以下になるが
1足りないから循環するのか。ああ、しかし、数学的にどう表現したらいいかわからない。w

564 :デフォルトの名無しさん:2017/10/18(水) 15:51:59.37 ID:+Osy4cjh.net
お題:顔文字(^o^)があります。この(^o^)を左右に動かしながら出力します。(^o^)は左から右へ一文字ずつ動き、端に到達した瞬間だけ(^o^)から(>_<)に変化し、また(^o^)に戻って左端へ行き、同じように繰り返します。
端から端までは最初80文字分の幅がありますが、(^o^)が端に達した回数だけ1文字ずつ狭くなっていき、最終的に(^o^)の端まで狭くなり、(^o^)が動けなくなります。(^o^)が動けなくなったらプログラムを終了してください。

565 :デフォルトの名無しさん:2017/10/18(水) 17:36:32.86 ID:jSYDae9q.net
>>564 Bash (builtins)
https://ideone.com/KFko4s

幅80文字だと出力が長くなりすぎて途中で切られるので50文字にしました。

566 :デフォルトの名無しさん:2017/10/18(水) 19:53:42.77 ID:4F2aMcKp.net
ウインドウズでエスケープシーケンス扱うのにおまじないいるからメンドクセー。

567 :デフォルトの名無しさん:2017/10/18(水) 21:05:28.73 ID:xwRaz5Kx.net
>>564
改行せずにカーソルを先頭に戻すのは CR (13) の出力で良いのか?
それとも curses ライブラリを使うべきか?

568 :デフォルトの名無しさん:2017/10/19(木) 01:56:08.11 ID:Lj1i7npR.net
>>567
好きな方をどうぞ
curses使うのはいいですね
こちらからは見れませんが

569 :デフォルトの名無しさん:2017/10/19(木) 03:35:28.31 ID:CNJYIyj0.net
じゃ、とりあえず CR 出力版。Perl プログラム。
但し、待ち時間入れないと速すぎて見えないので適当に usleep を入れた。
テストした環境は Linux で端末は Windows の TeraTerm。
TERM=xterm の状態。

但し、プログラムを貼り付けたサイト(paiza.io)での出力はおかしくなる。
何故なら端末として動いてないから。
試したい場合はプログラムをコピーして自分の環境のエディタ等にペーストして保存後に実行して。

https://paiza.io/projects/rQDCQizcsrydlcrkxylR1w

570 :デフォルトの名無しさん:2017/10/19(木) 05:11:39.88 ID:sgSfn4oM.net
>>564
https://ideone.com/dakfBb
C++。改行でやってみた。ちょっと汚い。

571 :名無しさん@そうだ選挙に行こう! Go to vote!:2017/10/22(日) 11:49:54.00 ID:/Umsqxkx.net
お題:A〜Z、1〜9で出来たランダムな文字列がある(文字列はプログラム開始時に自動的に決めてよい)
キーを2つ決めて(←→キーが自然かも)例えば→キーを押すと、文字列のうち2〜9があるか
どうかを調べ、あればそのうち一つをランダムに選び、数字を一つ減らし(9なら8へ)、左右ランダムに
1を置く
つまり2以上の数字文字があればそこがゴムのような役目をして文字列が伸びる
全部の数字文字が1になったら何もしない

逆に例えば←キーを押すと、文字列のうち1〜8があるかどうかを調べ、そのうちランダムに一つを
選び、その左右どちらかに数字文字がないかを調べ、足した合計が9を超えないようなら足し合わせて
数字文字をその合計値にし、文字列を1つ縮める
足し合わせた合計が9を超えるようなら他の数字文字もランダムに同様に一つ選び、足し合わせて
9を超えない数字文字の部分が見つかったらそれを一つだけ足し合わせて縮める

全部の数字文字が9になるか、9に満たないが足し合わせると9を超えるようになったら何もしない

572 :デフォルトの名無しさん:2017/10/23(月) 04:21:48.29 ID:sHZ1Pe4U.net
>>571
https://ideone.com/pjZbOc
C++。デバッグ難しくてやる気がしないのでバグってるかも。
カーソルキー取るの面倒だからASキーにしておいた。
こんな感じでいいのか?

何のシミュレータかしらんけどめんどくせーな。

573 :デフォルトの名無しさん:2017/10/23(月) 04:35:24.25 ID:sHZ1Pe4U.net
あ、それと、文字列で計算するのめんどくさかったから、数字でやった。
そっち事情なんか知ったこっちゃない。

574 :デフォルトの名無しさん:2017/10/23(月) 05:46:30.69 ID:iFI38Dlw.net
%%%%1000%%%%

000-[HUM%58*73.1\%]/2I/3NM/61.3SNMK%?%3%51.22222222222221%
001-[[[%6/4$17.6135412α3]]]]+DOM+SIL+7%
002-UML7%[61.2[31.5[!%32∂LM17.36%!16.3!%<<<%!HSTOL7%!Q!S!=3m=<2TOL<3Q9A<2.1GHz%,DOK,HAOARA,
003-[[[HEMLOT47[<\41.2%Q,===>[MLS<DPNO<\2.3>#ESOLA!5%!3MLA!>LTOSA>7TONSA>%>%end

575 :デフォルトの名無しさん:2017/10/23(月) 17:28:50.67 ID:f/2PkHQ/.net
>>574
全く読めねぇw

576 :デフォルトの名無しさん:2017/10/23(月) 20:53:32.54 ID:burVCZw1.net
ランダムの分布は指定なし?

577 :デフォルトの名無しさん:2017/10/23(月) 23:01:59.68 ID:sHZ1Pe4U.net
いろんなところでたまに見かけるけど、>>547 ってPGなの?

578 :デフォルトの名無しさん:2017/10/24(火) 06:49:36.20 ID:kt50Dt6N.net
PGとは?

579 :デフォルトの名無しさん:2017/10/24(火) 07:14:59.91 ID:Ohc+APnW.net
プロパンガス

580 :デフォルトの名無しさん:2017/10/24(火) 08:55:25.49 ID:TzjXrYm3.net
パーフェクトグレイド

581 :デフォルトの名無しさん:2017/10/24(火) 11:04:02.33 ID:2qWQgTrR.net
>>575>>577
>>574はvbsウィルスの一部だよ
つまりワクチンソフトに引っかかるとこのログが検疫されるので注意

582 :デフォルトの名無しさん:2017/10/24(火) 21:14:43.61 ID:6ceRFBNE.net
>>581
さんくす。

583 :デフォルトの名無しさん:2017/10/25(水) 20:13:14.36 ID:gieh1Z5o.net
お題
())())のように括弧のみからなる文字列が与えられるので
すべての括弧が正しく対応付けされるためには
最低で何箇所の括弧を逆向きに変更すればよいか求めよ
例えば上の例では2文字目か3文字目を変更すればよいので1を出力せよ
何文字変更しても正しく対応付けできない場合は-1を出力せよ

) -> -1
())()) -> 1
)()()( -> 2
)))((( -> 4
(())())((())(()( -> 3
())((())()))()(((()))()((((((((()()(())) -> ?

584 :デフォルトの名無しさん:2017/10/25(水) 21:21:26.00 ID:/TQ9iqwZ.net
>>583 Java
https://ideone.com/Oca2HR

585 :デフォルトの名無しさん:2017/10/26(木) 00:42:38.67 ID:0Gn/TXrF.net
>>583
Ruby
https://ideone.com/wQA5TN

586 :デフォルトの名無しさん:2017/10/26(木) 07:09:40.17 ID:vEkFybta.net
>>583
https://ideone.com/OkCwgQ
C++。効率とかショートコーディングとかそういうものをかなぐり捨ててべた書き。
あってるかな?

587 :デフォルトの名無しさん:2017/10/26(木) 08:33:36.44 ID:8oLfrbud.net
おむ

588 :デフォルトの名無しさん:2017/10/26(木) 08:35:57.56 ID:8oLfrbud.net
誤爆
>>583 Ruby

def calc str
return -1 if str.size.odd?
ary = optimise str.scan(/(?=.)(\(*)(\)*)/).map{|a, b| a.size - b.size}
(-ary[0] + ary[1].to_i).abs/2 + ary[0]%2
end

def optimise ary
a = ary.reject(&:zero?).chunk(&:positive?).to_a.transpose[1].map(&:sum)
return a if a.size < 3
a.unshift(0) if a[0] < 0
optimise a.each_slice(2).map(&:sum)
end

STR = %w{
)
())())
)()()(
)))(((
(())())((())(()(
())((())()))()(((()))()((((((((()()(()))
}

STR.each{|s| puts "%s -> %d"%[s, calc(s)]} #=>
) -> -1
())()) -> 1
)()()( -> 2
)))((( -> 4
(())())((())(()( -> 3
())((())()))()(((()))()((((((((()()(())) -> 5

589 :デフォルトの名無しさん:2017/10/26(木) 19:12:02.71 ID:MqWL4ZqL.net
>>583 ruby
https://ideone.com/jGTfL3

590 :デフォルトの名無しさん:2017/10/27(金) 22:31:59.57 ID:sfTuRN3o.net
>>583
@Mathematica

https://ideone.com/19n4r2

591 :デフォルトの名無しさん:2017/11/04(土) 00:05:20.36 ID:4+O3ouw4.net
Quineの派生ということで、コードそれ自身を反転させたものを出力せよ
反転とは文字列"abc\ndef"を"fed\ncba"にすること

592 :デフォルトの名無しさん:2017/11/04(土) 01:52:41.37 ID:P09Vk2Mx.net
数列 6,66,666,6666,66666.....
これをダミアン数列と呼ぶことにしましょう

nを自然数としたときn^n(^はべき乗)の桁数(10進数で)が
ダミアン数列のどれかになることはあるか?
初歩的な計算で7^7=823543が6桁になることがわかります

問 このような不吉な数は
    7のみである
    有限個存在する
    無限に存在する

ここまで書いてみたけどこの問題だとプログラミングのお題じゃないね
数論で解けるのかなあ?

改めてお題
ダミアン数列の最初の10項につながる不吉な自然数はあるか、あるとすれば
その数はいくつか

力技では時間が掛かりすぎると思うので工夫してみてください

593 :デフォルトの名無しさん:2017/11/04(土) 03:18:31.96 ID:RXqoYVvx.net
意味不明

594 :デフォルトの名無しさん:2017/11/04(土) 03:39:51.41 ID:pxF/c+yt.net
>>591
https://ideone.com/1bQnxO
C++。VCオンリー。
ウニコード対応しようと思って色々やってたけど、なんかうまくいかねー。
一応VCではうまくいってるっぽいけど、GCCと共通のコードはまだ規格的にきつそうだ。
うへー。大変だったわ。やっぱ、文字列は鬼門。

595 :デフォルトの名無しさん:2017/11/04(土) 04:34:35.38 ID:tUO6oLmA.net
>>592 Java
https://ideone.com/furdsD

596 :デフォルトの名無しさん:2017/11/04(土) 08:04:59.76 ID:bqd73Ayh.net
>>591 Squeak/Pharo Smalltalk

thisContext method getSource reversed allButLast: 8

"=> '8 :tsaLtuBlla desrever ecruoSteg dohtem txetnoCsiht' "

597 :デフォルトの名無しさん:2017/11/04(土) 10:09:48.41 ID:VZ1zDZPp.net
>>591 ruby
https://ideone.com/k9nGSp

598 :デフォルトの名無しさん:2017/11/05(日) 14:54:08.02 ID:rWIlHQ+T.net
>>597 ruby
d="]esrever.d,d[%'d=p%;s%' stup";puts '%s;%p=d'%[d,d.reverse]
#=> ]esrever.d,d[%'d=p%;s%' stup;"puts '%s;%p=d'%[d,d.reverse]"=d

599 :デフォルトの名無しさん:2017/11/05(日) 14:54:35.06 ID:rWIlHQ+T.net
安価ミスったorz

600 :デフォルトの名無しさん:2017/11/05(日) 22:47:04.05 ID:Pt23fyK7.net
>>595
2で割らずにシフトしてたり芸が細かいですな

601 :片山博文MZ :2017/11/07(火) 23:38:24.35 ID:BS6pey7a.net
お題。ツイッターのフォロワーを使ってお金を稼ぐ具体的な方法を思い付く限り列挙せよ。

602 :デフォルトの名無しさん:2017/11/07(火) 23:48:06.18 ID:aP9yM4om.net
やだ。

603 :デフォルトの名無しさん:2017/11/08(水) 00:39:27.61 ID:z/y1zyUv.net
プログラミングに関係ないお題は却下

604 :デフォルトの名無しさん:2017/11/08(水) 20:48:53.00 ID:XbOytUUT.net
片山博文MZってコジキなのか?

605 :デフォルトの名無しさん:2017/11/13(月) 00:48:07.30 ID:PHmyYrtX.net
コジキっていうか、頭の弱い子

606 :デフォルトの名無しさん:2017/11/14(火) 21:49:31.73 ID:yEmE0LhS.net
コード中でa-zA-Z0-9の文字を一切使わずに
Hello World!!
と出力せよ。
"!"の後ろの改行の有無は問わない。

607 :デフォルトの名無しさん:2017/11/14(火) 22:13:39.71 ID:1d5ZohBo.net
>>606 whitespace

608 :デフォルトの名無しさん:2017/11/15(水) 00:12:32.03 ID:ckRbh5hb.net
前にも見たなぁ。

609 :デフォルトの名無しさん:2017/11/15(水) 00:21:09.60 ID:edbITJRa.net
>>606
それじゃプログラム組めないと思うんだが、記号だけの言語みたいなの使えってこと?

610 :デフォルトの名無しさん:2017/11/15(水) 00:30:14.08 ID:au/IFdC5.net
>>606 bhnjdsbkjdsb
_

611 :デフォルトの名無しさん:2017/11/15(水) 00:31:44.60 ID:ckRbh5hb.net
iostreamがすでにアウト。

612 :デフォルトの名無しさん:2017/11/15(水) 01:21:11.75 ID:f03ykBDy.net
>>606

613 :デフォルトの名無しさん:2017/11/15(水) 01:21:50.26 ID:f03ykBDy.net
>>606 Ruby
https://ideone.com/z4vm4F
perlとjavascriptでも殆ど同じことができる

614 :デフォルトの名無しさん:2017/11/15(水) 01:23:33.97 ID:0AqsUHvD.net
今日は七五三ということで
7,5,3,+,-,×,÷,(),^2を使った式(ただし7,5,3は一個しか使えない)で1から連続でいくつまで数を作れるか

1=3+5-7
2=5-3
3=3
4=(5-3)^2
5=5
6=(7-5)×3


615 :デフォルトの名無しさん:2017/11/15(水) 06:25:11.32 ID:21MTGrxx.net
^2 もありですか

616 :デフォルトの名無しさん:2017/11/15(水) 11:19:06.80 ID:+wQkBp8E.net
>>613
回答見てもわからない
どういうこと?

617 :デフォルトの名無しさん:2017/11/15(水) 11:36:08.43 ID:cnBoJhFE.net
>>606
Ruby で、

unpacked = "Hello World!!".unpack("c*")
p unpacked
#=> [72, 101, 108, 108, 111, 32, 87, 111, 114, 108, 100, 33, 33]

packed = unpacked.pack('c*')
puts packed
#=> Hello World!!

618 :デフォルトの名無しさん:2017/11/15(水) 11:46:46.76 ID:f03ykBDy.net
>>616
特殊変数$$から1を作ってそれをもとに2, 4, 8, 16などを作る
"%c"を繰り返したものをあらかじめ作っておき
そこに上記の数字で作った"Hello World!!"の文字コードをsprintフォーマットする
標準出力を表す特殊変数$>に<<メソッドでできた文字列を出力する

あとは 「"" << 文字コード」で「文字コード.chr」と同様の結果が得られるので適宜利用すると便利

619 :デフォルトの名無しさん:2017/11/15(水) 12:14:35.50 ID:GYwcr8MQ.net
>>617の間抜けさ加減に草

620 :デフォルトの名無しさん:2017/11/15(水) 13:25:57.72 ID:YypYHZ3m.net
>>614
^2 は
int sqr(int n){return n*n;}
みたいな関数が使えるって意味だよね

つまり、
x^2^2 とかは (x^2)^2 の意味で使うなら可能ってことだよね

621 :デフォルトの名無しさん:2017/11/15(水) 13:32:50.33 ID:+wQkBp8E.net
>>618
記号で1を作って、数値、文字コード、文字列としてくのか
いろんな省略記法も知らないとできないな
解説ありがとう

622 :デフォルトの名無しさん:2017/11/15(水) 19:03:12.96 ID:vzgZy9E8.net
>>614
5と3を繋げて53にするようなこともしていいの?

623 :デフォルトの名無しさん:2017/11/16(木) 00:18:43.09 ID:/xRbPsNU.net
計算部は書いたけど、元の表記で何算してるか表記するのが面倒だ。
あと、遅い・・・。

624 :デフォルトの名無しさん:2017/11/16(木) 00:24:47.24 ID:IIofg8Am.net
73/5=14とかは駄目だよね?

625 :デフォルトの名無しさん:2017/11/16(木) 01:59:04.45 ID:/xRbPsNU.net
>>614
https://ideone.com/k7hAtB
C++。
それとなく書いてみたけど、誤差でまくり。割り算鬼門すぎる。
多分バグってる。あと、題意理解してない可能性が微レ存。

626 :デフォルトの名無しさん:2017/11/16(木) 02:00:39.12 ID:/xRbPsNU.net
あ、そうだ。
カッコの処理がバグバグだったからカッコ使わなかった。

627 :デフォルトの名無しさん:2017/11/16(木) 02:11:42.94 ID:/xRbPsNU.net
うーむ・・・。なんていうか。。。
ギブアップだ。Orz

628 :デフォルトの名無しさん:2017/11/16(木) 11:38:12.54 ID:clS3oGAP.net
>>625
「題意理解してない可能性が微レ存」どころじゃねえだろこれww

629 :デフォルトの名無しさん:2017/11/16(木) 14:11:34.74 ID:9+S8V57k.net
整数の範囲でも有理数の範囲でも答えが変わらないからつまらん

一旦非整数を経由しないと作れないのがないとやっぱり...

630 :デフォルトの名無しさん:2017/11/16(木) 15:03:27.99 ID:/xRbPsNU.net
>>628
可笑しいところ教えて!

631 :デフォルトの名無しさん:2017/11/16(木) 15:46:11.61 ID:9+S8V57k.net
(3^2)^2 = 3^4
((3^2)^2)^2 = 3^8
だから、3^(2*3) とかやっちゃダメだろ

あと、
3×5÷7 = 15÷7 ≠ 2

632 :デフォルトの名無しさん:2017/11/16(木) 15:59:11.96 ID:/xRbPsNU.net
>>631
あー、俺がタコでした。
まぁ、前段は表示系の問題だと思うお。
後者は割り算が全部悪い。
浮動小数で比較したくないんだよなぁ。悩ましい。

633 :デフォルトの名無しさん:2017/11/16(木) 16:39:35.92 ID:9+S8V57k.net
有理数クラスを作るのだ

634 :デフォルトの名無しさん:2017/11/16(木) 17:43:36.30 ID:/xRbPsNU.net
有理数の法則がよくわかってないし、デカイ。
ままならんなー。

635 :デフォルトの名無しさん:2017/11/16(木) 18:03:35.37 ID:9+S8V57k.net
(a/b) + (c/d) = (ad + bc) / bd
(a/b) - (c/d) = (ad - bc) / bd
(a/b) * (c/d) = ac / bd
(a/b) / (c/d) = ad / bc
(a/b) = (c/d) <===> ad = bc

分子 : 整数
分母 : 0以外の整数

636 :デフォルトの名無しさん:2017/11/16(木) 21:25:39.77 ID:/xRbPsNU.net
数学ムズイ。。。
PGも算数で解いてるからな。

637 :デフォルトの名無しさん:2017/11/17(金) 00:16:40.63 ID:5DUWZGJy.net
紙とペンで考えてみたところ
0以外の任意の整数なら3,5,7で表わせるから問題として不適なのでは?

638 :デフォルトの名無しさん:2017/11/17(金) 00:20:03.76 ID:M2EFWWXH.net
17

639 :デフォルトの名無しさん:2017/11/17(金) 10:03:52.61 ID:oe8UBfUe.net
>>637
3,5,7は1回までって回数制限があるから
表せる数は限られるよ。

640 :デフォルトの名無しさん:2017/11/17(金) 10:44:12.02 ID:a6b9gyRQ.net
17が出来ない

641 :デフォルトの名無しさん:2017/11/17(金) 19:35:00.15 ID:5DUWZGJy.net
ああ、本当だ。17はどうやっても作れないね
しかしこれをどうやってコードで計算すんだろう
^2があるから全探査はできないし
自分は「+または-」をいくつ使うかで場合分けして一個一個可能性を消していったんだけれども

642 :デフォルトの名無しさん:2017/11/17(金) 20:04:40.56 ID:M2EFWWXH.net
コードはアップしないけど出来たよ

643 :デフォルトの名無しさん:2017/11/17(金) 20:07:16.14 ID:M2EFWWXH.net
独自有理数クラス

演算回数を1回ずつ増やしていって、
出来た値に対応するフラグをセット

644 :デフォルトの名無しさん:2017/11/17(金) 20:09:06.68 ID:M2EFWWXH.net
数値をstd::multisetで保持
演算n回目のmultisetをstd::setで保持

645 :デフォルトの名無しさん:2017/11/18(土) 17:51:34.79 ID:6foiYhRZ.net
ABC4D
-E3FG
-----
77777

A〜G は、1〜9 の異なる数字。
ただし、3, 4 ではない

646 :デフォルトの名無しさん:2017/11/18(土) 17:56:36.72 ID:R4dFDjUs.net
はい、そうですか。

647 :デフォルトの名無しさん:2017/11/18(土) 19:08:28.21 ID:8fhXEikQ.net
>>645 Java
2年前の問題と俺の回答
http://peace.2ch.net/test/read.cgi/tech/1429195275/451

を改造したもの (50-54行目と標準入力)
https://ideone.com/2chU62

648 :デフォルトの名無しさん:2017/11/18(土) 19:44:36.57 ID:oFg54zrO.net
>>645 Ruby
f = ->a, b, c, d, e, f, g{10000*a + 1000*b + 100*c + d - (1000*e + 10*f + g) == 78037}
[1, 2, 5, 6, 7, 8, 9].permutation{|a| puts "%d%d%d4%d - %d3%d%d == 77777" % a if f[*a]}
#=>87142 - 9365 == 77777

649 :デフォルトの名無しさん:2017/11/18(土) 20:40:01.77 ID:6foiYhRZ.net
>>647
何も、そこまで作り込まなくても良いだろw

色々な覆面算に対応するため、汎用的に書いたのか

650 :デフォルトの名無しさん:2017/11/19(日) 22:39:02.74 ID:oda4btU4.net
500, 100, 50, 10, 5, 1円のすべての種類の硬貨を、1枚以上使って、
合計15枚で750円にする時、10円硬貨は何枚になるか?


A〜E の5人のランナーが走った結果、
完走したのは、1着とべべの2人で、残りの3人は、途中で棄権した

ここで、完走した2人は、必ず真実を言い、
棄権した3人は、必ず嘘をつくものとする
(つまり、事実に対して、真偽値を取る)

A: D は棄権した
B: A は、べべだった
C: E は棄権した
D: C は、べべだった
E: B は完走した

A〜Eがこのように答えた時、1着は誰か?

651 :デフォルトの名無しさん:2017/11/20(月) 01:25:03.39 ID:Z32/GYkn.net
先に答えやそれに至る式がわかっててコードに書き直すだけになっちゃうから
数学的に道筋立てて答えが出せるものはあんまりおもしろくないんだよな

652 :デフォルトの名無しさん:2017/11/20(月) 03:34:46.27 ID:GkhyFhEh.net
アルゴリズムとは、数式の完全コピー

最初に、数式を考えて、その数式が間違っていれば、
撃墜モードでは、そこを突かれて撃墜される

結局、数式の証明が大事。
証明に、勘違いが無いかどうか

653 :デフォルトの名無しさん:2017/11/20(月) 11:24:27.08 ID:7i/OQPcC.net
べべってなんぞ?

654 :デフォルトの名無しさん:2017/11/20(月) 11:38:36.82 ID:OJcNabXy.net
>>653
たぶんこの場合は大阪の方言

655 :デフォルトの名無しさん:2017/11/20(月) 11:45:30.45 ID:7i/OQPcC.net
俺地方の人間だからわかんない。

656 :デフォルトの名無しさん:2017/11/20(月) 18:24:16.80 ID:Slkhafwt.net
べべって最下位のことじゃないか
どべ

657 :デフォルトの名無しさん:2017/11/21(火) 23:50:05.58 ID:zUV8sDjk.net
>>645
こんなのどうかな。Kotlin で作った。
https://paiza.io/projects/ObrsYN_Z7G44yy-pOlZh_Q

658 :デフォルトの名無しさん:2017/11/23(木) 10:05:44.85 ID:zWeuVerg.net
お題
1から99を表示する
お題:1から999を出力する

ただし0を含む数は除く

659 :デフォルトの名無しさん:2017/11/23(木) 10:11:40.40 ID:TrZHjzbP.net
>>658
1000.times{|i|p i unless i.to_s[?0]}

660 :デフォルトの名無しさん:2017/11/23(木) 12:15:31.91 ID:6AL/1aep.net
>>658 GNU Smalltalk

1 to: 999 do: [:n | (n asString includes: $0) ifFalse: [n displayNl]]

661 :デフォルトの名無しさん:2017/11/23(木) 12:38:42.26 ID:KUvGqrz8.net
>>658 F#

let () = seq { 1..999 } |> Seq.iter (printfn "%d")

662 :デフォルトの名無しさん:2017/11/23(木) 12:40:24.86 ID:KUvGqrz8.net
>>661
すまん、問題よく読んでなかった・・・

663 :デフォルトの名無しさん:2017/11/23(木) 13:30:23.43 ID:jBvfUrCY.net
>>658
https://ideone.com/unKY2z
C++。ほかの言語だと一行で書けるんだけどなぁ。
まぁ過去に比べれば大分短くなったけど。

664 :デフォルトの名無しさん:2017/11/23(木) 15:45:02.09 ID:ys+VuKpG.net
>>658
文字も数もその場に合わせて適当に解釈してくれる言語だと楽だね。
perl だとこれでできる。

for(1..999){print"$_\n"unless(/0/)}

665 :デフォルトの名無しさん:2017/11/23(木) 16:16:10.88 ID:JcpJJmmU.net
>>658
@Mathematica

nListWithoutZero[n_]:=n//
 Range[1,#]&//
 Map[ToString,#]&//
 StringCases[#,RegularExpression["^(?!.*0).*$"]]&//
 Flatten;

In[1] := nListWithoutZero[999]
Out[1] = (略)

666 :デフォルトの名無しさん:2017/11/23(木) 16:34:54.15 ID:2sNCCDGP.net
>>658
ダメだお題の意味がわからん
降参

667 :デフォルトの名無しさん:2017/11/23(木) 16:42:00.54 ID:QTAUjuBR.net
>>658 rust
https://ideone.com/NFrvi7
fn main() {
println!("{:?}", (1..1000).filter(|i| !i.to_string().contains("0")).collect::<Vec<_>>())
}

668 :デフォルトの名無しさん:2017/11/23(木) 16:46:25.38 ID:ys+VuKpG.net
>>658
Kotlin で文字列変換してやる場合

fun main(args: Array<String>) {
 for (i in 1..999)
  if (! i.toString().contains('0', false))
   println(i)
}

数値のままやる場合

fun main(args: Array<String>) {
 for (i in 1..999)
  if (i % 10 != 0
     && (i < 10 || i / 10 % 10 != 0)
     && (i < 100 || i / 100 % 10 != 0))
    println(i)
}

669 :デフォルトの名無しさん:2017/11/23(木) 17:05:13.53 ID:fGVRHt7J.net
>>658
>>668 同じく数値のままやる場合
https://ideone.com/xySZgM

670 :デフォルトの名無しさん:2017/11/23(木) 17:08:37.18 ID:fGVRHt7J.net
http://rio2016.2ch.net/test/read.cgi/math/1510671832/722
お題:
n^2-1 = m^5
を満たす自然数 n, m は存在するか?

存在するという人と存在しないという人の両方が存在します

671 :デフォルトの名無しさん:2017/11/23(木) 17:18:38.32 ID:TrZHjzbP.net
>>670
(n, m) = (1, 0)
揚げ足取りはおいておいて、プログラミングで説く問題じゃないよね

672 :デフォルトの名無しさん:2017/11/23(木) 18:35:09.23 ID:zveldNvP.net
>>658
python

今回は必要ないかもだけど桁数増えた場合を考え再帰で
https://ideone.com/WC8Ksm

673 :デフォルトの名無しさん:2017/11/23(木) 19:09:48.15 ID:fGVRHt7J.net
>>671
まあ自明な解はさておき、その他は見つからないのが不思議です

674 :デフォルトの名無しさん:2017/11/23(木) 20:21:54.46 ID:/mQ4CZGQ.net
>>673
カタラン予想ですでに存在しないことが証明されているのに何が不思議なのかね

675 :デフォルトの名無しさん:2017/11/23(木) 20:24:10.50 ID:hjkeK8jf.net
>>673
その問題は数学的に解くものではないかな?
まあ、コンピュータなら力業でかなりの値を n, m に入れて計算して確認できるけどさ。

676 :デフォルトの名無しさん:2017/11/23(木) 20:56:22.87 ID:fGVRHt7J.net
>>674
カタラン予想というのがあるんですね、ありがとうございます!
https://en.wikipedia.org/wiki/Catalan%27s_conjecture

677 :デフォルトの名無しさん:2017/11/23(木) 21:05:28.66 ID:uF7hi9HH.net
>>658
#!/bin/sh
seq 999|grep -v 0

678 :668:2017/11/24(金) 06:21:36.98 ID:8wyGH9pr.net
>>658
Kotlin数値判定版。こんな風にも書けるなと後で気づいた。

fun f(n: Int): Boolean {
 var m = n;
 while (m != 0) {
  if (m % 10 == 0)
   return false
  m = m / 10
 }
 return true
}

fun main(args: Array<String>) {
 (1..999).filter(::f).forEach(::println)
}

679 :デフォルトの名無しさん:2017/11/24(金) 07:42:25.55 ID:MEHEP0+e.net
存在するしないをプログラミングで証明するのはお題として良くない

680 :デフォルトの名無しさん:2017/11/24(金) 20:42:02.54 ID:G34PGfZh.net
log 2 を2進数表記した時の小数点第 n 位から n + 9 位までを求めよ. (1 ≦ n ≦ 10^10)
cf. log 2 = 0.10110001...

*Sample input*
1
11
10000
31415926
314159265

*Sample output*
1011000101
1100100001
0010110110
1001010110
0111101001

681 :デフォルトの名無しさん:2017/11/24(金) 23:31:00.22 ID:r53+zpq0.net
>>680
c++で書いたけど小数第100億位を計算するのに5時間くらいかかりそうorz

682 : :2017/11/25(土) 00:04:03.20 ID:ROI3Hzdd.net
>>681
もう初手に届くとは劇速ですね

683 :デフォルトの名無しさん:2017/11/25(土) 06:53:37.62 ID:Uo3oYb2P.net
無条件でlogって書いたら普通自然対数だろ

684 :デフォルトの名無しさん:2017/11/25(土) 07:01:13.88 ID:Uo3oYb2P.net
ライブラリを使えばほとんど何も書かなくて良いけど
どこから書くことを求められてるの?

685 :デフォルトの名無しさん:2017/11/25(土) 07:03:54.53 ID:Uo3oYb2P.net
>>679
「良くない」じゃなくて「出来ない」でしょ

686 :デフォルトの名無しさん:2017/11/25(土) 07:16:55.64 ID:Uo3oYb2P.net
>>684
と思ったけど、普通に全桁計算したら終わらないな

687 :デフォルトの名無しさん:2017/11/25(土) 07:34:12.57 ID:Uo3oYb2P.net
Σ { 1 / (2^i × i) }
を使って10^10項位までを42bitくらいだけ計算すれば出来るかな?
1/nの周期性を考えないと計算量的に無理?
10^10が微妙に32bitを越えてるのがイヤだねえ

688 :デフォルトの名無しさん:2017/11/25(土) 08:21:38.07 ID:Uo3oYb2P.net
>>687
ダメだ
ざっと計算量を見積もったらとても5時間じゃ終わらない

689 :デフォルトの名無しさん:2017/11/25(土) 13:10:34.24 ID:l6j6CjYT.net
>>375
xxx@xxx-VirtualBox:~/casl$ casl -s -e -i stdlib.casl -i bigint.casl fact.casl
1
1
2
6
24
120
720
5040
40320
362880
          途 中 省 略
1405006117752879898543142606244511569936384000000000
60415263063373835637355132068513997507264512000000000
2658271574788448768043625811014615890319638528000000000
119622220865480194561963161495657715064383733760000000000
5502622159812088949850305428800254892961651752960000000000
258623241511168180642964355153611979969197632389120000000000
12413915592536072670862289047373375038521486354677760000000000
608281864034267560872252163321295376887552831379210240000000000
30414093201713378043612608166064768844377641568960512000000000000

暇つぶしに書いてみたけど足算掛算割算しかできない
引算は難しすぎるんで諦めた

690 :デフォルトの名無しさん:2017/11/25(土) 16:35:37.76 ID:J1zvm3XW.net
バイナリ法で最適化した結果なんとか1時間あれば10^10位は計算できるようになったがまだ縮められるかな

691 : :2017/11/25(土) 16:41:37.19 ID:ROI3Hzdd.net
備忘メモ
http://rio2016.2ch.net/test/read.cgi/math/1510671832/890
ちゃんとお題にして出題しなおします

692 : :2017/11/25(土) 21:49:51.82 ID:ROI3Hzdd.net
>>680
指数関数のマクローリン展開で試してみたのですが、これは収束が遅すぎますね、それに収束半径を超えてるし…
なにか収束の早いよい方法はないものか…

693 : :2017/11/25(土) 21:54:37.24 ID:ROI3Hzdd.net
>>692
×指数関数
○対数関数

694 :デフォルトの名無しさん:2017/11/25(土) 22:58:19.91 ID:yyAYDlfh.net
対数関数のマクローリン展開?
そりゃ無理だ
log 0 が定義されてない

695 :デフォルトの名無しさん:2017/11/25(土) 23:12:41.85 ID:FRsJtlII.net
>>689
CASLで書いたの?
ソースコードは?

696 :デフォルトの名無しさん:2017/11/25(土) 23:38:41.06 ID:yyAYDlfh.net
>>680
log 2 = Σ_[i=1, 2, ...] { 1 / (2^i × i) }
冪剰余

でいける気がしてきた

しばらく暇がない
時間が空いたら
アセンブラ & C++ & OpenMP
でやってみる

697 :デフォルトの名無しさん:2017/11/26(日) 02:33:02.82 ID:T275kIwU.net
>>650
(setq aaa '(1 5 10 50 100 500))
(setq ddd 750)
(setq jjj 15)

(defun bbb (ccc iii)
(if (= iii 0)
ccc
(let (eee)
(dolist (fff ccc)
(dolist (ggg aaa)
(when (<= (+ (apply #'+ fff) ggg) ddd)
(push (cons ggg fff) eee))))
(bbb (remove-duplicates (mapcar (lambda (x) (sort (copy-seq x) #'<)) eee) :test 'equal) (1- iii)))))

(let* ((kkk (bbb '((0)) jjj))
(lll (mapcar (lambda (x) (remove 0 x)) kkk)))
(remove-if-not (lambda (x) (and
(= (apply #'+ x) ddd)
(= (length x) jjj)
(= (length (remove-duplicates x)) (length aaa))
)) lll))
((1 1 1 1 1 5 5 5 10 10 10 50 50 100 500))

698 :デフォルトの名無しさん:2017/11/26(日) 02:34:12.86 ID:T275kIwU.net
>>650
(setq aaa '(A B C D E))

(defun fff (ddd)
(if (null (cdar ddd))
ddd
(let (eee)
(dolist (jjj ddd)
(let ((bbb (car jjj))
(ccc (cdr jjj)))
(setq eee (append (mapcar (lambda (x) (cons (cons x bbb) (remove x ccc))) ccc) eee))))
(fff eee))))

(defun iii (kkk)
(if (< kkk 2) #'identity #'not))

(let* ((ggg (fff (list (cons nil aaa))))
(hhh (mapcar (lambda (x) (car x)) ggg)))
(remove-if-not (lambda (x) (and (funcall (iii (position 'A x)) (> (position 'D x) 1))
(funcall (iii (position 'B x)) (= (position 'A x) 1))
(funcall (iii (position 'C x)) (> (position 'E x) 1))
(funcall (iii (position 'D x)) (= (position 'C x) 1))
(funcall (iii (position 'E x)) (< (position 'B x) 2)))) hhh))
((D C B E A) (D C E B A) (D C A E B) (D C E A B) (D C A B E) (D C B A E))

699 : :2017/11/26(日) 11:18:11.00 ID:rNgJnhxq.net
>>694
いやいや
https://ja.wikipedia.org/wiki/%E5%AF%BE%E6%95%B0
log(1-x) = - Σ((1/n)x^n) に x = -1 を機械的に代入しました、収束半径外ですが、この値は正しいらしい。

700 :デフォルトの名無しさん:2017/11/26(日) 12:12:20.72 ID:ffy1o2uq.net
お題
ASCIIコード表が載っている本をあげよ

701 :デフォルトの名無しさん:2017/11/26(日) 12:35:31.87 ID:tJzac9f2.net
>>695
うんCASL
全部で1200行かあ
xxx@xxx-VirtualBox:~/casl$ wc -l stdlib.casl bigint.casl fact.casl
274 stdlib.casl
851 bigint.casl
76 fact.casl
1201 合計

ソースはこういうのが延々続いててずっと眺めてるとゲシュタルト崩壊起こして
何が何だか分からなくなるよ
ld gr5,0,gr1
ld gr6,1,gr1
lad gr4,4,gr1
addl gr4,gr0
st gr4,0,gr1
st gr6,1,gr1
ld gr4,=1
st gr4,2,gr1
st gr0,3,gr1
ld gr6,gr1
ld gr1,0,gr1
st gr5,0,gr1
st gr6,1,gr1
xor gr4,gr4
st gr4,2,gr1
lad gr2,-4,gr2
subl gr2,gr0
st gr2,3,gr1
ld gr0,gr3

702 :デフォルトの名無しさん:2017/11/26(日) 12:35:43.37 ID:WgExDItE.net
>>699
他だ単に対数って言えば
log x のこと
これをマクローリン展開は無理

log (1-x) のマクローリン展開ならそう書かないと通じない
収束半径の外じゃなくて、収束半径丁度でしょ

703 :デフォルトの名無しさん:2017/11/26(日) 12:49:31.90 ID:WgExDItE.net
-log (1-x) のマクローリン展開に、
x = 1/2 を入れると
>>696 になる

704 :デフォルトの名無しさん:2017/11/26(日) 13:21:30.43 ID:jiBTwXK4.net
理解はしてないが、出てきたので貼っとく。



指数対数関数等の超越関数の多倍精度計算

本論文では、 指数対数関数の高精度計算として Taylor 展開に BSA 法を使って高速化する方法提案する。
約 1000 桁以下の精度の計算では、 Taylor 展開を使った計算が Sasaki and Kanada[5] によって、様々な計算
法を比較して最も高速であることが示されているので、 計算時間が問題となるのは、 1000 桁以上の精度の
計算である。 ここで提案した Taylor 展開に BSA 法を適用して高速化した方法と Sasaki and Kanda によっ
て提案された方法を 1000 桁を超えた精度で比較し、 その高速性を示した。

211 階乗計算例
10000! の計算を行う。 この計算では、 BSA 法を使うだけでなく、 1600 桁以上の数値に対しては FFT を利用して乗算を行っている。
計算方法 計算時間(msec)
BSA 47
従来の方法 3578

このほか、 三角関数、逆三角関数、双曲線関数など簡単な規則で各項の係数が表現でき、 多くの関数がこの
行列の乗算形式に変形できます。Taylor 展開の係数が簡単な規則で表現できない $\tan x$ が例外的に表現できないだけである。

3 まとめ
指数関数や対数関数の Taylor 展開に BSA 法を適用することによって、 BSA を使わない従来の方法に比べ40 %程度の高速化ができた。
対数関数に対しては、 5000 桁程度の精度で最も高速な計算方法として知られた Sasaki and Kanada の方法を超えることを示した。

http://www.kurims.kyoto-u.ac.jp/~kyodo/kokyuroku/contents/pdf/1456-24.pdf

705 : :2017/11/26(日) 13:37:36.96 ID:rNgJnhxq.net
>>702
たしかに

収束半径は |x < 1| なのでは?

706 :デフォルトの名無しさん:2017/11/26(日) 13:44:48.59 ID:jiBTwXK4.net
とりあえず理解はできた計算方法として、logxの近似値などをaとおいたとき、
logx = a + log(x/e^a)という変形を用いる方法だ。
aが近似値だと、x≒e^aなので良いらしい。

707 :デフォルトの名無しさん:2017/11/26(日) 13:58:53.69 ID:WgExDItE.net
>>705
収束半径は1
収束半径は、収束するエリアと収束しないエリアの境目となる円の半径
収束半径丁度の時は収束する場合もあるししない場合もある

708 :デフォルトの名無しさん:2017/11/26(日) 14:20:52.91 ID:WgExDItE.net
>>706
計算する項が少なければ良いわけではなく、
各項の計算時間も重要

709 :デフォルトの名無しさん:2017/11/26(日) 15:04:05.33 ID:jiBTwXK4.net
exp(x)は、(exp(x/k))^k (kは2ベキ)、とするといいらしい。
k=2なら、括弧内を計算したやつ同士の掛け算。

710 :695:2017/11/26(日) 18:33:11.28 ID:XHzckn9a.net
>>701
なるほどありがとう
怖いもの見たさがあった

711 : :2017/11/26(日) 20:54:13.61 ID:rNgJnhxq.net
>>689
えへへ、調べさせてもらったよw
後半は 42! から 50! までの値だね
この範囲なら、多数桁×ワンレジスタの計算で済みますね

多数桁×多数桁を実装すれば思いっきり褒めてあげるよ、えへへ:−)

712 :デフォルトの名無しさん:2017/11/26(日) 21:11:54.45 ID:tJzac9f2.net
>>711
ほい
xxx@xxx-VirtualBox:~/casl$ casl -s -e -i stdlib.casl -i bigint.casl bimul.casl
350306543997676425792
153864088327713953064
53899597027434699691252340823058767026688

713 : :2017/11/26(日) 21:47:36.23 ID:rNgJnhxq.net
>>712
おお、すごい、gmp で確認したがあってるぞ
私はまだ 10進文字列から多数桁型への変換は実装できていない、また仕事ができてしまったなあ

714 :デフォルトの名無しさん:2017/11/26(日) 23:01:31.48 ID:tJzac9f2.net
>>713
gmpで確かめてるのかあ
俺は確認はclispかrubyでやってるよ
10進文字列からの変換は10倍しながら足しこんでいくだけだから
そんなに難しくないでしょ
掛算なしでも(n<<3)+(n<<1)でできるし
逆の10進文字列への変換は割算が必要だから実装するの大変だったなあ
これができあがるまではメモリを16進ダンプして計算が合ってるか確かめてた

715 :デフォルトの名無しさん:2017/11/27(月) 06:49:43.60 ID:qqP20rnw.net
>>696がよさげだな
こういうことだろ?

Σ(1/n) >> n
小数にもシフトを適用するとして。

和の誤差がかわかれば、どこまで計算したらいいかわかるがどうやるんだ?

716 :デフォルトの名無しさん:2017/11/27(月) 07:17:24.36 ID:qqP20rnw.net
誤差しらべたら、テイラー展開は平均値の定理一般化だったか


数値計算とテイラー展開
ある区間において,関数 f(x)がn 回微分可能であるとし,定数aはこの区間に含まれるものとする.x もこの区間内に含まれるとき,
http://math-lab.main.jp/taylor07a.jpg
をみたすa とx の間の実数c (a <c <x または x <c <a)が存在する
http://math-lab.main.jp/taylor7.html

717 :デフォルトの名無しさん:2017/11/27(月) 07:40:30.25 ID:qqP20rnw.net
log(1+x)の誤差項、剰余項は、(-1)^(n-1)/n * (x/(1+c))^n らしいので、
-log(1-x)では、1/n * (x/(1+c))^n か。
x=1/2で考えると、この項をなるべく大きくするならc=0で、誤差は(1/n) >> n以下か。ふたたびシフト使用。

718 :デフォルトの名無しさん:2017/11/27(月) 07:44:14.48 ID:qqP20rnw.net
まとめると、
log2 - (Σ(1/k) >> k) < (1/n) >> n  (級数はn-1までの和)

719 :デフォルトの名無しさん:2017/11/27(月) 08:54:49.81 ID:qqP20rnw.net
どこか間違えてる 次数か?

720 :デフォルトの名無しさん:2017/11/27(月) 08:59:59.63 ID:qqP20rnw.net
いやあってるか。
A(k) = (1/k)>>kと置くと、
log2 - ΣA(k) < A(n) (級数はn-1までの和) で
ΣA(k) (n+1以上の和) < A(n) が成立するのか。

721 :デフォルトの名無しさん:2017/11/27(月) 10:11:23.24 ID:qqP20rnw.net
やはり、どこか間違ってるな。
上のとおりだと、log2 - ΣA(k) (級数はn-1までの和)は、
A(n) を含むので、A(n)より小さいはずがない。

722 : :2017/11/27(月) 23:07:50.22 ID:b0u4s5jJ.net
>>701
>ソースはこういうのが延々続いててずっと眺めてるとゲシュタルト崩壊起こして何が何だか分からなくなるよ
対象のコード書いてるときの「感覚」をハードディスクかどこかに貯めておいて、
必要に応じてまた脳みそに搭載できるようにならないものか…

そのマシン語の一行一行にも、もともとはなんらかの意味的構造があったのに、それが消えてしまうなんて損失以外のなにものでもないよね

723 :デフォルトの名無しさん:2017/11/27(月) 23:36:53.65 ID:bwTh4Bk9.net
>>680

>>696の方法で作ってみました
n=10^10の時に48.3秒くらいです

724 :デフォルトの名無しさん:2017/11/28(火) 13:09:45.40 ID:IU/PYwM1.net
>>723 はHaswell (4770) 3.4GHz固定での結果で
Skylake (6700K) 定格だと38.4秒でした

ちゃんとCPUも進化してるんですね

725 :デフォルトの名無しさん:2017/11/28(火) 13:17:18.48 ID:IU/PYwM1.net
>>681
>>690
C++だとどうやって計算してるかが非常に気になります
32bitを越える値同士の乗算(結果が64bitを越える)部分

アセンブラだと
64bit x 64bit ===> 128bit
128bit / 64bit ===> 64bit
等があるのでそれを使っちゃってますが

726 :デフォルトの名無しさん:2017/11/28(火) 13:20:30.03 ID:IU/PYwM1.net
冪剰余を求めるのに
(a * b) % c
みたいなのがたくさん出てきませんか?
aもbもcも32bitの範囲を微妙に越えてて

727 :デフォルトの名無しさん:2017/11/28(火) 14:44:32.26 ID:jzUFRHpN.net
誤差部分の間違いが判った。これでよさげだ。
ただし誤差評価を荒くやってはダメそうだが。一番最後の行のところ。



誤差項ありのマクローリン展開は、0<=c<=xが存在して

f(x) = Σ x^k * f(k)(0)/k! (kは0からn-1まで) +  x^n * f(n)(c)/n!

f(x) = -log(1-x)のn次導関数は、(n-1)!/(1-x)^n。
このときマクローリン展開は誤差項は x^n / (n*(1-c)^n)

x=1/2ならば、c=1/2のとき最大で、1/n

728 :デフォルトの名無しさん:2017/11/28(火) 19:45:30.22 ID:jzUFRHpN.net
これが収束速いようだ。


log(2) = 3log(81/80) + 5log(25/24) + 7log(16/15)

log((x+1)/(x-1))
= log((1+1/x)/(1-1/x))
= 2 Σ 1/((2n+1)*x^(2n+1))

729 :デフォルトの名無しさん:2017/11/28(火) 22:18:22.38 ID:7WoPw74F.net
>>728
1/log(2) ≒ 3.32
1/2log(161)+1/2log(49)+1/2log(31) ≒ 0.85

なので、計算に必要な項数は1/4程度
でも、1つの項の計算には時間がかかる

log(1-x)のマクローリン展開に0.5を入れた物は
分母が i * 2^i だから速く計算できるのだ

730 :デフォルトの名無しさん:2017/11/28(火) 22:20:46.48 ID:7WoPw74F.net
>>727
残りの項を等比数列と見なせば
簡単に誤差の上限が出ます

731 :デフォルトの名無しさん:2017/11/28(火) 22:47:09.33 ID:7WoPw74F.net
>>724
Haswellで33.96秒に縮まりました
シングルスレッドだと182.54秒で5.3倍
HTTが効くということは、
まだ多少改善の余地がありそう

一番内側のループは
vmulpd
vmulpd
vroundpd
vfmsub213pd
vfmsub132pd
vsubpd
なんと浮動小数点で計算してます

732 :デフォルトの名無しさん:2017/11/28(火) 22:53:54.93 ID:7WoPw74F.net
n=10000000000の時は
0000010101 でした
出題者さま、合ってます?


また、たまたまですが

n=10000000004では
0101010101

n=10000000005では
1010101010

になります

733 :デフォルトの名無しさん:2017/11/28(火) 23:30:15.70 ID:9HoDrqB3.net
一番内側のループのコード
http://fast-uploader.com/file/7067434368942/

PORT5がガラ空きで、処理のほとんどがPORT0,PORT1
こんなんでもHTTが効く
やっぱり浮動小数点はレイテンシがデカい

AVX512になれば
レジスタの数が倍になるので
8パラにしてレイテンシを隠蔽出来るんだけど
もちろんレジスタ長が倍になる方が大きい

734 :デフォルトの名無しさん:2017/11/29(水) 13:17:33.09 ID:mHyZby47.net
>>728は後半部分が間違ってるか。log((x+1)/x) = log(1+1/x) の展開を用いるのが正解で。

log(・)の中身を1に近づけた方が収束が早くなるが、
こういった分解 log(2) = 3log(81/80) + 5log(25/24) + 7log(16/15)はどうみつけるのか。
これは数値が(x+1)/x の形だけど、(x+1)/(x-1)の分解もあるのか。こっちだと計算するベキ項が一つ飛ばしにできる。>>728のように。

735 :デフォルトの名無しさん:2017/11/29(水) 13:34:57.75 ID:8/kTvoZy.net
2 = (81/80)^3 * (25/24)^5 * (16/15)^7

3 と 5 の指数の合計が0になる組み合わせを検索すれば良い

736 :デフォルトの名無しさん:2017/11/29(水) 13:37:30.83 ID:8/kTvoZy.net
log(81/80) = log(162/160) = log((161+1)/(161-1))

わかってて書いてるんだと思ったが
>>729のlogの中身はこの値

737 :デフォルトの名無しさん:2017/11/29(水) 13:42:05.17 ID:mHyZby47.net
>>728はそういうことか。みつけたやつのコピペで、そのとき考慮はしてなかった。

738 :デフォルトの名無しさん:2017/11/29(水) 13:45:31.51 ID:mHyZby47.net
指数も固定でなくていいはずで、
16/15よりかはたとえば1001/1000のほうが1に近いからそういうのはいくらでも見つけられるのかとおもった。

739 :デフォルトの名無しさん:2017/11/29(水) 14:44:26.09 ID:8/kTvoZy.net
分母分子の素因数の数と同じ項数が必要

例えば素因数が 2, 3, 5, 7 の4種類の場合、
1個差もしくは2個差のペアを4個探す

例えば
126/125
225/224
2401/2400
4375/4374

これらを適当に掛け算して2^nになるようにすると
項が4個の式がみつかる

740 :デフォルトの名無しさん:2017/11/30(木) 00:31:43.08 ID:H4qIjcIH.net
分母、分子とも 2, 3, 5, 7, 11, 13, 17 のみしか素因数を持たない形の場合、
以下が一番計算する項の数が少ないようです

log(2) = 72*log(126/125)+27*log(225/224)-19*log(2401/2400)+31*log(4375/4374)

741 :デフォルトの名無しさん:2017/11/30(木) 05:07:54.11 ID:fMs2N0Mh.net
>>740
その数値を検索すると、音楽のコンマというのが出てくる。関係あったり理論があったりするのか。


A.D. Fokker: Unison Vectors and Periodicity Blocks
http://www.huygens-fokker.org/docs/fokkerpb.html

List of 7-prime limit accidentals - The?Sagittal?forum
http://forum.sagittal.org/viewtopic.php?f=6&t=252

742 :デフォルトの名無しさん:2017/11/30(木) 05:28:17.43 ID:fMs2N0Mh.net
log(2)とは無関係で、単に一個差のやつで適当な素因数分解できるやつに名前がついてるだけ?



An Investigation into the Extraction of Melodic and Harmonic Features from Digital Audio

unit interval name
4375/4374 Ragisma
2401/2400 Breedsma
225/224 Septimal Kleisma
145/144 Difference between 29:16 and 9:5
126/125 Small Septimal Semicomma
121/120 Undecimal Seconds Comma
81/80 Syntonic Comma

http://scholar.sun.ac.za/handle/10019.1/100826

743 :デフォルトの名無しさん:2017/11/30(木) 11:02:35.16 ID:fMs2N0Mh.net
log2のほうは、分子・分母の素因数分解が似通ってないと成立しないってことで、
音楽のほうは小さい素数に限定して一個差ペアを求めたと理解。
log2のほうは、共通の素数で大きいやつを最初に固定すれば考えれば、よさげかと。

744 :片山博文MZ :2017/11/30(木) 12:12:48.85 ID:NsMGt5if.net
お題。横x[cm]、縦y[cm]の長方形のステンレスの1枚の板がある。この板からm枚の複数の長方形の部材を切り出す。
部材のサイズは配列で与えられる。
部材のサイズ(縦×横)はそれぞれだいたい決まっているが、1cm程度変わってもよい。
ただし、部材の縦または横が変わるとそれぞれ一点減点となる。
すべての部材を切り出すことができれば、減点がなるべく少ない方法の切り出し方法を出力せよ。
すべての部材を切り出すことができなければ、面積が広い順になるべくたくさん部材を切り出せ。

745 :片山博文MZ :2017/11/30(木) 12:19:33.84 ID:NsMGt5if.net
テストデータ。

x=10, y=10,
{
{5, 10}, {2, 2}, {2, 2}, {4, 3}, {6, 5}
}

x=5, y=12
{
{2, 5}, {3, 3}, {2,9}, {3, 2}, {4,3}
}

746 :片山博文MZ :2017/11/30(木) 12:28:07.36 ID:NsMGt5if.net
部材の縦と横は入れ替わってもよい。
可能ならば、切り出し方法をSVG形式で出力せよ。

747 :片山博文MZ :2017/11/30(木) 12:51:59.04 ID:NsMGt5if.net
切り出し方法は、

切り出す部材のx座標、y座標、幅、高さ

のリストとして出力せよ。

748 :片山博文MZ :2017/11/30(木) 12:56:57.25 ID:NsMGt5if.net
切り出しに余裕があるときは、なるべくx座標の大きい方、y座標の大きい方を残すようにせよ。

749 :デフォルトの名無しさん:2017/11/30(木) 13:08:21.37 ID:tkxPMdZc.net
斜めもOK?

750 :デフォルトの名無しさん:2017/11/30(木) 13:08:55.87 ID:tkxPMdZc.net
人間用のパズルで、斜めにしないと解けないのとかありそう

751 :デフォルトの名無しさん:2017/11/30(木) 14:23:24.17 ID:SHLZLl2M.net
問題文の条件が“だいたい”“なるべく”なんて
あいまい表現だらけ

これでプログラミングの問題かよ

752 :片山博文MZ :2017/11/30(木) 16:18:01.24 ID:NsMGt5if.net
斜めは考えなくてもよい。

訂正。
すべての部材を切り出すことができれば、減点が最小である切り出し方法を出力せよ。
すべての部材を切り出すことができなければ、面積が広い順に切り出せる部材の面積が最大になるよう部材を切り出せ。
切り出しに余裕があるときは、x座標の大きい方、y座標の大きい方を残すようにせよ。

753 :680:2017/11/30(木) 17:10:37.24 ID:8ZVWPbH7.net
>>732
そこにある3つとも正解です
当初は L = Σ1/(k*2^k) として
2^n * L の小数部分を愚直に求める方法を想定していました

754 :デフォルトの名無しさん:2017/11/30(木) 17:26:52.63 ID:r8WkgLop.net
普通に多倍長で計算したら計算量的に終わらないですよね?
n=314159265を求めるのに
冪剰余は使ってますよね?

おそらく私も同じような方法と思います
FMA3命令とOpenMPで高速化してるだけで

755 :デフォルトの名無しさん:2017/11/30(木) 22:49:50.18 ID:TklDiPhy.net
愚直という言い方は良く割りませんでしたね
仰る通り冪剰余は用います

756 :片山博文MZ :2017/12/01(金) 14:26:32.50 ID:fw1UFg83.net
再出題。横cx[cm]、縦cy[cm]の長方形または正方形のステンレスの1枚の板がある。この板からm枚の複数の長方形または正方形の部材を切り出す。
m枚の部材のサイズは(縦, 横)の配列で与えられる。
すべての部材を切り出すことができれば、切り出し方法を出力せよ。切り出しが不可能ならば「impossible」と出力せよ。
切り出し方法は、
(部材インデックス、部材の一番左のx座標、部材の一番上のy座標、幅、高さ)
のリストとして出力せよ。斜めの方向の切り出しは考えなくてもよい。
切り出しに余裕があるときは、x座標の大きい方、y座標の大きい方を残すようにせよ。ただし、x軸は右の向き、y軸は下向きとする。

テストデータ。

cx=10, cy=10, m=5, {5, 10}, {2, 2}, {2, 2}, {4, 3}, {6, 5}

cx=5, cy=12, m=5, {2, 5}, {3, 3}, {2, 8}, {3, 2}, {4, 3}

757 :デフォルトの名無しさん:2017/12/01(金) 17:01:28.13 ID:BqJQPTjH.net
頭の悪そうな文章だな

正方形⊂長方形
ステンレスの板の座標上の位置指定が無い
余裕がある場合の条件の意味が曖昧

758 :片山博文MZ :2017/12/01(金) 18:41:36.20 ID:fw1UFg83.net
ステンレスの板の左上座標は原点にあるものとする。
切り出しは、可能な限り、座標の小さい方を優先する(「余裕」の意味)。

759 :デフォルトの名無しさん:2017/12/01(金) 20:32:10.86 ID:9YSaSAW0.net
>>758
相変わらず曖昧な表現

全ての長方形の上辺と左辺がどこかに接していれば良いのか?
そうじゃないのか?

このような条件のを1個だけ見つければ良いのか
すべて見つけるのか

760 :片山博文MZ :2017/12/01(金) 20:41:14.79 ID:fw1UFg83.net
全ての部材の上辺と左辺が別の部材の辺、もしくは、元の板の端に接していること。
このような条件のをすべて見つけること。

761 :片山博文MZ :2017/12/01(金) 20:43:57.24 ID:fw1UFg83.net
なんか、工学関係でこのような問題があるらしいが、まだ解決策があるかどうかわからん。これが解ければ、実用化待ったなし。

762 :デフォルトの名無しさん:2017/12/01(金) 21:15:23.73 ID:9YSaSAW0.net
お題じゃなくて依頼かよ

763 :デフォルトの名無しさん:2017/12/01(金) 21:19:54.38 ID:9YSaSAW0.net
実用性なら
切りやすさとか余り素材の形状とか
そういうのが重要だろうに

問題として中途半端過ぎる

764 :デフォルトの名無しさん:2017/12/01(金) 21:20:21.36 ID:qVeescqP.net
また片山博文MZ が乞食をやってるのか

765 :デフォルトの名無しさん:2017/12/01(金) 21:23:07.33 ID:9YSaSAW0.net
普通に考えて、NP問題だろう

766 :デフォルトの名無しさん:2017/12/01(金) 21:48:16.44 ID:8H4JUlF5.net
なにやら揉めてますね
そろそろうんざりなので次のお題どうぞ

767 :デフォルトの名無しさん:2017/12/01(金) 23:36:29.58 ID:N1IVcYDB.net
スポーツスケジューリング問題。

768 :デフォルトの名無しさん:2017/12/03(日) 02:47:12.16 ID:QazTjKaA.net
お題ってこういうのでもいいのかな
a[10] = { 0, 1, 2, 3, 4, 5, 6, 7, 8, 9 }
このように10個の整数を要素に持つ配列がある(整数の値は不定)
b = エントリーポイント
c = 移動する数量
d = 移動する距離
を与えることにより、配列の要素を移動させるプログラムを作りなさい
ただし配列の右端の要素を一つ移動させると、配列の左端に移動するものとする


a = 3, b = 1, c = 5, a = 0124567839
b = 1, c = 3, d = 1, a = 0412356789
b = 7, c = 1, d = 5, a = 1273456890
b = 0, c = 8, d = 1, a = 8012345679
b = 4, c = 5, d = 4, a = 6783901245
b = 9, c = 5, d = 4, a = 5679012384
b = 7, c = 3, d = 1, a = 9123456078

769 :デフォルトの名無しさん:2017/12/03(日) 02:49:08.83 ID:QazTjKaA.net
>>768
一番上の例は
b = 3, c = 1, d = 5, a = 0124567839
の間違いです

770 :デフォルトの名無しさん:2017/12/03(日) 09:00:27.20 ID:ucQfMVKf.net
>>768
https://ideone.com/zZdIlU
C++。なかなか素直な問題なのに邪悪な思念で見ていたため手間取った。
マニュアルを熟知していると簡単かもしれない。

771 :デフォルトの名無しさん:2017/12/03(日) 09:04:48.71 ID:b0AFqm0g.net
>>768 Ruby
https://ideone.com/0l4cEo

772 :デフォルトの名無しさん:2017/12/03(日) 10:14:55.69 ID:ZeeZWfzn.net
>>768
@Mathematica

https://ideone.com/z4q1Nk

773 :デフォルトの名無しさん:2017/12/03(日) 12:12:41.56 ID:jbLAlLAh.net
>>768
b + c が10を超えることはありますか?

774 :デフォルトの名無しさん:2017/12/03(日) 12:55:22.10 ID:sC6phLBS.net
>>773訂正
b,c,dが10を越えることはありますか?

775 :デフォルトの名無しさん:2017/12/03(日) 13:05:23.73 ID:b0AFqm0g.net
>>771はb,c,dが任意の自然数でも大丈夫なようにしておいた

776 :デフォルトの名無しさん:2017/12/03(日) 13:08:19.92 ID:QazTjKaA.net
みなさんプログラム作るの早いですね

>>774
一応a、b、cの条件は、このようになると思います。
0 <= a <= 9
0 <= b <= 8
0 <= c <= 10 - b - 1

bとcは0では意味がないので、こっちのほうがいいのかな
0 <= a <= 9
1 <= b <= 8
1 <= c <= 10 - b - 1

777 :デフォルトの名無しさん:2017/12/03(日) 13:12:58.80 ID:QazTjKaA.net
>>776
すみません、間違えました、abcじゃなくてbcdですね
0 <= b <= 9
1 <= c <= 8
1 <= d <= 10 - b - 1

778 :デフォルトの名無しさん:2017/12/03(日) 13:34:35.32 ID:QazTjKaA.net
>>777
すみません、訂正の訂正です。dは10-c-1ですorz
0 <= b <= 9
1 <= c <= 8
1 <= d <= 10 - c - 1

779 :デフォルトの名無しさん:2017/12/03(日) 17:47:25.64 ID:Gq7SJlPX.net
Linked list 使うと楽そうな感じするな

780 :デフォルトの名無しさん:2017/12/03(日) 18:26:21.27 ID:QazTjKaA.net
>>770,771
せっかくなのでこのテストデータを作って検証してみましたが、全部合っていました、さすがですね
それにしても他人のプログラムって動かすことはできても、理解するのは困難ですね
>>772
Mathematicaは残念ながら持っていないので検証できませんでした

b = 8 c = 6 d = 2 a = 8901236745
b = 4 c = 3 d = 2 a = 0123784569
b = 4 c = 5 d = 2 a = 8123904567
b = 2 c = 5 d = 2 a = 0178234569
b = 0 c = 5 d = 3 a = 5670123489
b = 9 c = 3 d = 4 a = 3459016782
b = 5 c = 1 d = 7 a = 1253467890
b = 4 c = 6 d = 2 a = 8923014567
b = 6 c = 7 d = 2 a = 8901253467
b = 7 c = 5 d = 4 a = 5789016234
b = 4 c = 2 d = 2 a = 0123674589
b = 6 c = 4 d = 5 a = 4678950123
b = 7 c = 4 d = 4 a = 4789056123
b = 4 c = 5 d = 4 a = 6783901245
b = 8 c = 1 d = 5 a = 1238456790
b = 0 c = 2 d = 6 a = 2345670189
b = 3 c = 7 d = 1 a = 9120345678
b = 9 c = 4 d = 2 a = 4901256783
b = 8 c = 3 d = 6 a = 3456890712
b = 2 c = 5 d = 3 a = 0178923456

781 :デフォルトの名無しさん:2017/12/03(日) 20:16:05.68 ID:hV+xPFYR.net
>>768
ruby ワンライナー
https://ideone.com/sXGhCG

782 :デフォルトの名無しさん:2017/12/03(日) 20:37:58.61 ID:ucQfMVKf.net
>>780
お、あんた偉いな。
出題者でちゃんと答え合わせやってくれる人が稀でレスポンス薄いことが多いんだよ。
ちゃんと動いてよかったよ。

783 :デフォルトの名無しさん:2017/12/03(日) 20:51:48.60 ID:ucQfMVKf.net
>>774
自分は10超えてもらったら困るなぁ。
10っていうか、配列の要素数だな。
修正はそんなに難しくないけど。

784 :デフォルトの名無しさん:2017/12/03(日) 23:00:23.32 ID:QazTjKaA.net
>>781
1行プログラムすごいですね
さすがスクリプト言語

>>782
そうなんですか、でもいろいろな言語で回答されるから、出題者も大変かもしれないですね
私は問題の意味すらよく分からないことが多いので、もっぱら見る専ですが

785 :デフォルトの名無しさん:2017/12/04(月) 02:36:11.06 ID:iGjrIGoV.net
>>768
これでいいのかなあ?
Perl で書いた。
https://paiza.io/projects/ncIY4LljeBahWZPJpd8ZPQ

786 :デフォルトの名無しさん:2017/12/04(月) 06:47:18.44 ID:Rc7ie/2s.net
>>785
b = 6、 c = 6、 d = 2
のときの動作がおかしいようです
8901452367
となるはずが
0167892345
となっており、移動する"678901"の並びが崩れてしまっています

787 :785:2017/12/05(火) 02:32:35.27 ID:LDxS5CId.net
>>786
問題勘違いしてました。素直にぐるぐる回すように修正しました。
https://paiza.io/projects/ncIY4LljeBahWZPJpd8ZPQ
下の「入力」タブの方にスペース区切りで b, c, d の値を1行づつ並べて入れてから実行させると「出力」に結果が出ます。
とりあえず >>768 に書いてある値を入力にセットしたところ出力は同じになりました。

788 :デフォルトの名無しさん:2017/12/05(火) 20:55:13.95 ID:vy+ohhoY.net
>>787
入出力の確認をしただけですが、今回は問題ないようです
paiza使うと簡単に確認できて便利ですね

789 :デフォルトの名無しさん:2017/12/05(火) 21:48:50.31 ID:32LsMTj+.net
>>768
僕の頭だとどうしても右端から左端に行くときの動きがイメージできないので、解答締め切ったあとでもいいのでちょろっと教えてもらえると嬉しいです
上3つまではわかるけどそこから先がそもそも答えにたどり着けない……

790 :デフォルトの名無しさん:2017/12/05(火) 23:47:14.24 ID:ynbcQBXQ.net
>>789
ヒント。
(インデックス++)%配列の長さ
を繰り返すとどうなりますか?%は余剰デス。

791 :デフォルトの名無しさん:2017/12/06(水) 00:25:49.49 ID:gvvJf1Ph.net
>>789
上の3つまでは分かるということなので、3つめのcの値を一つずつ増やしてみると、こんな感じになります
"7"、"78"、"789"、"7890"と並ぶ数値が増えていっているのが分かると思います
c+d の合計は9が最高なので、これ以上cを増やすにはdを減らさなくてはなりません
b = 7 c = 1 d = 5 a = 1273456890
b = 7 c = 2 d = 5 a = 2378456901
b = 7 c = 3 d = 5 a = 3478956012
b = 7 c = 4 d = 5 a = 4578906123

今度はcを2に固定して、dの値を一つずつ増やすと、こんな感じになります
"78"の並びが一つずつ右へずれていっているのが分かると思います
c+d の合計は9が最高なので、これ以上dを増やすにはcを減らさなくてはなりません
b = 7 c = 2 d = 1 a = 0123456978
b = 7 c = 2 d = 2 a = 8123456907
b = 7 c = 2 d = 3 a = 7823456901
b = 7 c = 2 d = 4 a = 2783456901
b = 7 c = 2 d = 5 a = 2378456901
b = 7 c = 2 d = 6 a = 2347856901
b = 7 c = 2 d = 7 a = 2345786901

こんな感じで分かるでしょうか?私も自分の頭で考えると混乱しますw

792 :デフォルトの名無しさん:2017/12/06(水) 00:31:57.65 ID:+0bHqE6f.net
自分の考え方は、配列の一部を切り取ってパッディングするんだけど、
まず配列を回転させて切り取る第0インデックスを配列の最初に持って来る。
すると、配列の後ろにはすでにパディングが終わった数列のができてる。
で切り取って削除して尻尾にくっつける。
で、さっき回した分を戻してやると完成。

という方法で、>>770を解いた。

793 :デフォルトの名無しさん:2017/12/06(水) 00:32:05.18 ID:+0bHqE6f.net
自分の考え方は、配列の一部を切り取ってパッディングするんだけど、
まず配列を回転させて切り取る第0インデックスを配列の最初に持って来る。
すると、配列の後ろにはすでにパディングが終わった数列のができてる。
で切り取って削除して尻尾にくっつける。
で、さっき回した分を戻してやると完成。

という方法で、>>770を解いた。

794 :デフォルトの名無しさん:2017/12/06(水) 00:32:32.42 ID:+0bHqE6f.net
あちゃー
二重投稿になったある。
そんなに大事でもないのだけど。

795 :デフォルトの名無しさん:2017/12/06(水) 00:38:22.54 ID:+0bHqE6f.net
C++は比較的不自由な言語なので頭使うのは鍛えられるよ。
パーツが少ないのでほとんど自作しないといけない。

796 :デフォルトの名無しさん:2017/12/06(水) 00:46:18.89 ID:DokUEpLm.net
コンパイルが必要とかでスクリプト系の言語より
使うことに不便があるかもしれないが、
できるアプリが自由で高速・軽量!

ライブラリもSTLやboost以外にも、探せば便利なのがいっぱい。

797 :デフォルトの名無しさん:2017/12/06(水) 01:07:01.76 ID:+0bHqE6f.net
よそのライブラリ使うとイデオンで動かないからなぁ。
まぁ、業務ではライセンスに合ったものを使えばいいよ。

798 :デフォルトの名無しさん:2017/12/06(水) 02:01:03.49 ID:QVT4XBLu.net
>>789
塊が移動すると考えれば良い。
例えば 7, 4, 2 だとすると、 789 と先頭の 0 が移動する塊だ。
で、ちょっと分かり易くするためにこの塊を伏せて * で書くとするとこうなる。

*123456***

それでこの塊を右に一つずらす。その時に1は食われて尻尾から吐き出される。

**234561**

移動量2なのでもう一回右にずらす。すると2が食われて尻尾から吐き出される。

***345612*

それでは * から 7890 に戻してみよう。

8903456127

できあがり。

799 :デフォルトの名無しさん:2017/12/06(水) 02:58:25.26 ID:+0bHqE6f.net
>>793
これ間違ってるな。
最後尻尾にくっつけるんじゃなくて、適所にインサートだった。
で戻す。
書いた日から時間たってるからすまん。

800 :デフォルトの名無しさん:2017/12/06(水) 08:41:18.93 ID:obBhCrma.net
>>790
>>791
>>798
ありがとうございます
やっと理解できました
がんばって解いてみます

801 :デフォルトの名無しさん:2017/12/06(水) 17:48:37.81 ID:T95E7suL.net
log2だけど、これ使うといいらしいよ。分割統治法のBSA法というやつらしい。
2個ずつの積を繰り返すことで計算回数が減らせる。


{{1, a0}, {0, r}}*{{1, a1}, {0, r}}を[ [○,P], [○,Q] ]とおくと
r*P/Qはa0 + a1/rであり、

{{1, a0}, {0, r}}*{{1, a1}, {0, r}}*{{1, a2}, {0, r}}を[ [○,P], [○,Q] ]とおくと
r*P/Qはa0 + a1/r + a2/r^2。

上の式を実際に計算してみる。
http://www.wolframalpha.com/input/?i=%7B%7B1,+a0%7D,+%7B0,+r%7D%7D*%7B%7B1,+a1%7D,+%7B0,+r%7D%7D*%7B%7B1,+a2%7D,+%7B0,+r%7D%7D

802 :デフォルトの名無しさん:2017/12/06(水) 21:09:44.45 ID:N+lgs3o9.net
>>801
普通のlog 2の計算には使えても
>>680 には使えないかと

803 :デフォルトの名無しさん:2017/12/07(木) 00:07:53.25 ID:YC8MWngg.net
>>731 から更に縮まりました
もう誰も興味が無いかも知れませんが

コードいる?

804 :デフォルトの名無しさん:2017/12/07(木) 09:24:10.80 ID:XIHsqoOR.net
>>801でできるはずだ。やってみる

805 :デフォルトの名無しさん:2017/12/07(木) 10:09:03.65 ID:DjN/Fbox.net
n=10^10で19.28秒にまで縮めた
n=10^13くらいまでなら夜寝てる間に終わる

>>804
期待してます!

806 :デフォルトの名無しさん:2017/12/07(木) 15:54:52.13 ID:XIHsqoOR.net
804だけど考えてみたら面倒なんだな。
有理数(整数)で完全に求めてから割り算するのは時間かかりそうだから、
展開も、割り算も、有限で打ち切って求める精度がでるようにするのが普通?

807 :デフォルトの名無しさん:2017/12/07(木) 18:15:20.36 ID:wGY0QmnN.net
お題
辺の長さが10,000以下の整数である直方体について
すべての面の対角線も整数となるものを全て求める

808 :デフォルトの名無しさん:2017/12/07(木) 19:52:07.65 ID:5gbe7aWB.net
対角線を出す式を忘れたのだ〜。
数学って難しい。

809 :デフォルトの名無しさん:2017/12/07(木) 20:10:49.23 ID:bwV7uU+3.net
>>807
答えは浮かんだけど、
手元にインタプリタがないので書けない。

810 :デフォルトの名無しさん:2017/12/07(木) 20:38:16.84 ID:fmQCcJGl.net
>>808
えーと、ほら、三角形の三角形のアレ

811 :デフォルトの名無しさん:2017/12/07(木) 21:23:21.04 ID:BdlZ1dXv.net
>>805 のソース一式とexeです。
>>680 の回答となります。
http://fast-uploader.com/file/7068204781334/

実行ファイルは、拡張子をexe_からexeに変えてください。
OSはWindows 64bit Vista以降
CPUはHaswell以降
で動作します。

812 :デフォルトの名無しさん:2017/12/07(木) 21:26:37.34 ID:5gbe7aWB.net
作ってみたけど、オッセ―。
なんか数学的にやらないとダメぽ。

>>810
検索したら出てきた。
ただのHYPODだった。

813 :デフォルトの名無しさん:2017/12/07(木) 21:34:41.35 ID:5gbe7aWB.net
あ〜ん。足し算10兆回かかるとかむりげーじゃ。
どうしようかこれ。

814 :デフォルトの名無しさん:2017/12/07(木) 21:40:02.60 ID:5gbe7aWB.net
>>807
https://ideone.com/5Tu6fx
C++。ギブアップ。遅すぎだ。
デバッグ不完全だからそこを忘れないで。
多分1日かけても終わらないと思う。

815 :デフォルトの名無しさん:2017/12/07(木) 21:40:16.73 ID:BdlZ1dXv.net
>>807
特に大きな工夫もなく普通に3重ループで出来ましたよ
151個かな

816 :デフォルトの名無しさん:2017/12/07(木) 21:42:04.13 ID:BdlZ1dXv.net
https://ideone.com/09jmll

817 :デフォルトの名無しさん:2017/12/07(木) 21:44:43.70 ID:8KQwIEWy.net
>>807 Java
https://ideone.com/iYsIgq
こうでいいのけ?

818 :デフォルトの名無しさん:2017/12/07(木) 21:48:20.51 ID:5gbe7aWB.net
ぶ。おれは・・・いったい・・・。
だめだなぁ。才能ないなぁ。

819 :デフォルトの名無しさん:2017/12/07(木) 22:07:39.64 ID:BdlZ1dXv.net
>>817の意味がわからん
解説よろしく!

820 :デフォルトの名無しさん:2017/12/07(木) 22:27:00.87 ID:XIHsqoOR.net
>>801のように積へ変換した場合、どれ位の精度・桁数が必要なのか簡単にわからないな。
>>801でやると、誤差ありの巨大な数同士の掛け算になって、その結果誤差が拡大する。
和のままやると、適当に2ベキをかければ、各項の整数部分だけ計算すればよさそうだけど。
>>801は桁を十分にとったとしても、2進10桁以上の部分も計算途中で無視はできないか。

821 :デフォルトの名無しさん:2017/12/07(木) 22:32:10.24 ID:5gbe7aWB.net
グア。題意勘違いしてた。
直方体の対角線はいらんのか。うおー。
ばかばかー。

822 :デフォルトの名無しさん:2017/12/07(木) 22:57:05.97 ID:5gbe7aWB.net
>>807
https://ideone.com/Uipgvw
C++。書き直したら、>>816のパクリ見たくなった。
そしてそれでも遅いとかあかんなー。
うーん。なんでなんだろう・・・。

823 :デフォルトの名無しさん:2017/12/07(木) 23:06:46.33 ID:BdlZ1dXv.net
>>820
普通に、求めたい精度+α の精度を保てば十分かと
演算回数はlog(n)のオーダーなので
普通にlog(2)を求める時には使える手法

824 :デフォルトの名無しさん:2017/12/07(木) 23:06:47.69 ID:8KQwIEWy.net
>>819
https://ideone.com/ZCINKa
適当にコメント付けた

825 :デフォルトの名無しさん:2017/12/07(木) 23:18:28.32 ID:8ACt5G91.net
物体は表面だけでなく無数の内面を有するという考えは絶対に存在する
一方で全てを表面でしかとらえない人の存在も否定できない

826 :デフォルトの名無しさん:2017/12/07(木) 23:24:59.96 ID:BdlZ1dXv.net
このポエムをどうプログラミングしろと?

827 :デフォルトの名無しさん:2017/12/08(金) 03:20:08.11 ID:JkPU7Xcj.net
>>807 Ruby
https://ideone.com/3BPK13
20秒程かかってしまう

828 :デフォルトの名無しさん:2017/12/08(金) 12:45:22.72 ID:lX5lg0SB.net
log2は>>801でもできそうだ。
積の先頭ほど精度が必要で、無視できる上限・下限を積の位置で可変にするか、
最初の積の計算で必要な精度をすべてに適用するかでいけそう。

829 :デフォルトの名無しさん:2017/12/08(金) 14:42:05.63 ID:3hyL4NRS.net
>>817
これパクってstd::bitset<10001>でやったら動かないかな
nextSetBitだけ作れば動きそう

830 :デフォルトの名無しさん:2017/12/08(金) 17:20:05.13 ID:qoqqt6pE.net
>>829
やってみた
いろいろ怪しいが速度は強烈に速いな

https://ideone.com/d4dU1Q

831 :デフォルトの名無しさん:2017/12/09(土) 00:08:31.54 ID:NdkcUgXa.net
>>828
時間的に無理だろって話

832 :デフォルトの名無しさん:2017/12/09(土) 02:59:25.21 ID:C21Kkp0z.net
>>807
これでいいかな?
Kotlin で書いた。しかしpaiza.ioの制限なのか、何故か import kotlin.math.* がエラーだったので java.lang.Math の関数を使っている。
https://paiza.io/projects/ImbYdf3NSWgW-zj8Ke6cDw

833 :デフォルトの名無しさん:2017/12/09(土) 09:05:25.04 ID:WAlxvB4a.net
お題
10,000以下の三角形数をもとめる

834 :デフォルトの名無しさん:2017/12/09(土) 09:42:44.72 ID:HfcuYzyJ.net
>>833
三角数だよね?
.js
https://ideone.com/WZBIFF

835 :デフォルトの名無しさん:2017/12/09(土) 10:35:24.71 ID:PLdtTEIG.net
>>833 ruby
i,j=0,-1;p i while(i+=j+=1)<10000

836 :デフォルトの名無しさん:2017/12/09(土) 21:40:06.26 ID:PLdtTEIG.net
>>833 Brainf**k ワンライナー(笑)
https://ideone.com/TyMeco
遅すぎて8bitまでしか計算できなかった

837 :デフォルトの名無しさん:2017/12/09(土) 23:46:47.87 ID:3TNSAl29.net
ideone.comはたいして最適化しないBrainf**k処理系だからな

838 :デフォルトの名無しさん:2017/12/10(日) 00:50:34.10 ID:J0bkBqjd.net
>>833
1+2+3+... だよね? じゃあこうだ。
perl -e '$i=0;$n=1;while(($i+=$n)<=10000){print"$i\n";++$n}'

839 :デフォルトの名無しさん:2017/12/10(日) 00:52:24.37 ID:J0bkBqjd.net
>>833
while じゃなくて for にするとこう。
perl -e 'for($i=0,$n=1;($i+=$n)<=10000;++$n){print"$i\n"}'

840 :デフォルトの名無しさん:2017/12/10(日) 08:00:18.20 ID:uc7Ht429.net
>833 R
cat(cumsum(0:140))

841 :デフォルトの名無しさん:2017/12/10(日) 09:05:44.08 ID:gZDGKqrG.net
>>801は精度管理の手間を考えたら、そのまま級数計算するのと大差ないと諦めたが。
こうすれば割り算の小数計算がほぼでないからいいのでは? 既に実装済?


an = 1/n、r = 2として、Σ an/r^n を定数C倍したやつの整数部分の取り出し。
C*anを再びanとおく。
Σ an/r^n
= a2/r^2 + a4/r^4 +・・・+ aN(2)/r^N(2) (添え字は2の倍数を動く)
+ a3/r^3 + a9/r^9 + a15/r^15 + ・・・+ aN(3)/r^N(3)
+ ap/r^p + ・・・+ aq/r^q + ・・・ (pは素数、qはp未満の数で割り切れないpの倍数)

= 1/N(2)r^N(2) * ( N(2)*a2*r^(N(2)-2) + ・・・ + N(2)*aN(2) ) + ・・

この分子部分は、各項、整数でそのまま計算しても>>801でも速くなるはず。

842 :デフォルトの名無しさん:2017/12/10(日) 21:52:17.18 ID:gZDGKqrG.net
>>841は添え字に被りが出ていて間違えてた。

843 :デフォルトの名無しさん:2017/12/10(日) 22:07:36.99 ID:XvO3Mos9.net
やってみて時間教えて

844 :デフォルトの名無しさん:2017/12/10(日) 22:08:42.09 ID:XvO3Mos9.net
>>805 >>811 を抜ける?

845 :デフォルトの名無しさん:2017/12/10(日) 22:22:45.57 ID:gZDGKqrG.net
秒数ではCPU依存するから正確に比較できない。
掛ける2だけのループを10^10回するだけでも19秒では終わらない。
無料のideone codepadなどの実行可能時間以内にできる範囲とか、
ベースとなる簡単なコード、関数の何倍時間がかかるかなどだと比較できるけど。

846 :デフォルトの名無しさん:2017/12/10(日) 22:36:08.95 ID:3sNoocWL.net
演算回数の見積りは?

847 :35歳:2017/12/11(月) 02:10:45.96 ID:OsSLt9Cy.net
Bronze取りました

848 :35歳:2017/12/11(月) 02:11:34.08 ID:OsSLt9Cy.net
Bronze取りました

849 :デフォルトの名無しさん:2017/12/11(月) 04:58:28.24 ID:zs4BBX0s.net
くだらん話かも知れんけど
竹内関数のメモ化

https://ideone.com/sVhke1

これはC++によるものだけど、C++や他の言語でもっと高速に書く方法はあるか?

850 :デフォルトの名無しさん:2017/12/11(月) 05:19:34.19 ID:zs4BBX0s.net
あ、ちなみにideoneの結果は間違っている
多分スタックオーバーフローしている
100, 50, 0の場合は答えは100

851 :デフォルトの名無しさん:2017/12/11(月) 05:38:23.74 ID:pBTqvDfH.net
>>849
constexprにすれば実行時はほぼ計算無しにできると思う。
コンパイル時間半端ないけど。

852 :デフォルトの名無しさん:2017/12/11(月) 05:45:19.55 ID:pBTqvDfH.net
ところで、いくらメモ化してても5回しか呼ばれないってことがあるのか?って気はする。

853 :デフォルトの名無しさん:2017/12/11(月) 07:04:34.34 ID:pBTqvDfH.net
https://ideone.com/Y9WhRz
竹内関数constexprにしてみた。
ウチの環境だとこの辺のパラメータが限界っぽいかな。
100,50,0だとメモリ使い切るらしい。

854 :デフォルトの名無しさん:2017/12/11(月) 07:09:30.00 ID:pBTqvDfH.net
あら、コンパイルエラーになっちゃった。
VCだと通ったんだけど。もちろんステップ数とか設定はいじってるが。

855 :デフォルトの名無しさん:2017/12/11(月) 08:32:22.37 ID:gAGFZ0s2.net
>>849
こんなのとか?

たらいを回すならHaskella
2006年04月07日 22:09
http://blog.livedoor.jp/dankogai/archives/50447103.html

856 :デフォルトの名無しさん:2017/12/11(月) 12:03:51.37 ID:uZdMj4Ux.net
お題
6つの辺の長さが 与えられた4面体の体積を求める

857 :デフォルトの名無しさん:2017/12/11(月) 13:07:53.63 ID:iSg/oyC4.net
お題
8つの辺の長さが与えられた超5面体の体積を求める

858 :デフォルトの名無しさん:2017/12/11(月) 14:29:31.82 ID:hKbhSguL.net
お題
与えられた自然数を高々四個の四角数(平方数)の和で表せ

859 :デフォルトの名無しさん:2017/12/11(月) 14:35:19.85 ID:k7Z6O4lr.net
>>856 Ruby
https://ideone.com/qbHExm
ただし四面体ABC-Dに対して入力の順序は
AB BC CA CD DA BD とする

860 :デフォルトの名無しさん:2017/12/11(月) 16:06:12.32 ID:k7Z6O4lr.net
>>857
8つだと多胞体が一意に定まらないと思うんだが

861 :デフォルトの名無しさん:2017/12/11(月) 16:48:32.53 ID:qWzXCzKk.net
>>853
64bit環境でやったらスラッシング起きて\(^o^)/

862 :デフォルトの名無しさん:2017/12/11(月) 16:53:24.07 ID:iSg/oyC4.net
>>860
じゃあ10個で

863 :デフォルトの名無しさん:2017/12/11(月) 17:03:18.77 ID:k7Z6O4lr.net
>>862
10でも足りないと思うんだけどひょっとして超五面体って五胞体のつもりで言ってる?

864 :デフォルトの名無しさん:2017/12/11(月) 17:31:58.31 ID:iSg/oyC4.net
4次元の5胞体

865 :デフォルトの名無しさん:2017/12/11(月) 17:32:53.55 ID:iSg/oyC4.net
5C2=10

866 :デフォルトの名無しさん:2017/12/11(月) 22:30:10.81 ID:pBTqvDfH.net
>>861
ウチはi6700メモリ8Gだな。20秒くらいボケーっとしてたらコンパイル完了する。
実実行より全然早い。

867 :デフォルトの名無しさん:2017/12/11(月) 23:43:41.01 ID:jF/PrBtV.net
>>866
んー32bitでコンパイルしてみるかな
64bitはマジ卍ヤバい

868 :デフォルトの名無しさん:2017/12/12(火) 00:19:40.23 ID:JpJzeAvs.net
>>866
悪い
VCで /constexpr:steps 1000000を付けてコンパイルしたら4秒ほどでコンパイルが終わった
多分/MPも付けてるからだと思う
実行結果は25になった

gcc 7.2.0 64bitだとどんどんメモリを食って行って最後にスラッシングが起きる
馬鹿正直な実装をしているからかも知れないね

VCの方がいろいろとメモリを食わないように工夫されてるのかも

Clangでも/constexpr:steps は -fconstexpr-steps という形でサポートされてるようだから
多分行けると思う
メモリ64G積んでるし

869 :デフォルトの名無しさん:2017/12/12(火) 01:27:56.58 ID:qpuoD4bc.net
>>868
いいマシーンだな。
まぁ、ウチはあれくらいで資源尽きちゃうけど、メモリ64Gもあったらもっと行けるな。
気が向いたらどうぞ。

870 :デフォルトの名無しさん:2017/12/12(火) 04:08:00.08 ID:SvlIPxM4.net
>>858 Java
https://ideone.com/ND4D2n

871 :デフォルトの名無しさん:2017/12/12(火) 22:52:21.86 ID:38dJ/vud.net
以下のURLのように、同じ色の点同士をつなぐゲームがある。
https://play.google.com/store/apps/details?id=com.bigduckgames.flow

N×Mの2次元配列が与えられる。配列の各要素は半角英字('a'-'z')または'*'である。
半角英字は色付きの点を表し、'*'は空のマスを表す。
'*'以外の文字は、配列中に必ず2個ずつ存在する。

このパズルの解を一つ出力せよ。
・'-'は左右のマスをつなぐ
・'|'は上下のマスをつなぐ
・'.'はマスをつながない

解がない場合は"No solution"と出力せよ。

[input]
***rg
**bg*
r****
ob*yo
****y

[output]
*-*-*-r.g
|.......|
*.*-b.g-*
|.|......
r.*.*-*-*
..|.|...|
o.b.*.y.o
|...|.|..
*-*-*.*-y

872 :デフォルトの名無しさん:2017/12/15(金) 09:30:54.91 ID:gDuLBiTf.net
>>841
やってはないけど、そもそもこれ間違ってるのと、同じような発想でやるとしても
全ての素数での分類ではなく、3分割くらいのほうが効率がいいのと、
Σ (2の倍数) + Σ (3の倍数かつ2の倍数でない) + Σ (2と3で割り切れない) 
分割する事もなく、N項の和だとしたらNの階乗か分数をなくせる最小公倍数かけてもいい。それだと掛け算もしくは割り算がいくつも出てくるが。

873 :デフォルトの名無しさん:2017/12/16(土) 14:04:43.43 ID:+Cq6iaDY.net
>>871
等幅フォントで表示しているエディタにコピペしてようやっと何を言わんとしているか分かった。
それってマスとマスの間に - または | を入れて繋ぐってことでいいんだよね? で、つながない所がピリオドだと。
(まあ等幅フォントのASCIIでやるならそれしか方法ないとは思うけど)。

874 :デフォルトの名無しさん:2017/12/16(土) 14:07:59.25 ID:+Cq6iaDY.net
しかしピリオドは

**
**

の時に

*.*
...
*.*

のようになって中央のピリオドが本来なら不要なものになるわけだが、それはスペースでなくても良いのかな?
まあただの幅合わせだからどうでもいいものではあるが。

875 :デフォルトの名無しさん:2017/12/17(日) 12:04:31.10 ID:V69L7+t+.net
>>801 の方法で出来ると書いてるけど
結局誰もやってないのか

個人的には(高速には)出来ないと思っている

876 :デフォルトの名無しさん:2017/12/17(日) 12:18:14.72 ID:V69L7+t+.net
>>871
課題じゃなくて
単にソルバーの作成依頼でしょ

877 :デフォルトの名無しさん:2017/12/17(日) 17:50:05.54 ID:3PMrWzl3.net
無理に線引かないでrだのgだので埋めた方がわかりやすいと思うがな
すべてのマスを埋めなければならないルールが抜け落ちてるみたいだから
実は別物のゲームで交差があるとかなったらそうはいかないが

878 :デフォルトの名無しさん:2017/12/17(日) 20:33:27.61 ID:0HU8GFa9.net
ブレゼンハム的なやつって、始点と傾き(と区間や境界等で決まる明示されない終点)が
与えられた際に、終点座標を求めてから始点に向かうのってアリなんだろうか?

879 :デフォルトの名無しさん:2017/12/18(月) 08:21:39.40 ID:T+ClDj4W.net
ここでやるには問題がでかすぎ
100分割してほしい

880 :デフォルトの名無しさん:2017/12/21(木) 14:37:42.61 ID:/SOyyWKP.net
>>871 Ruby
https://ideone.com/EOj9mz
ただし解は1つでありかつ線が通らないマスは無いことを前提とする
問題はここから引用:→http://www.nikoli.com/ja/puzzles/numberlink

881 :デフォルトの名無しさん:2017/12/21(木) 14:38:13.66 ID:/SOyyWKP.net
あ、あと出力方法は自分好みに適当にいじってる

882 :デフォルトの名無しさん:2017/12/21(木) 15:58:54.33 ID:jrnuCabF.net
訂正
ただし解は1つでありかつ線が通らないマスは無いことを前提とする
ではなく
解が存在すればすべての解は全てのマスを通ることを前提とする

883 :デフォルトの名無しさん:2017/12/22(金) 19:30:58.88 ID:PPoMR9m8.net
お題
22の分割(たとえば3+3+5+8)のうち
分割したそれぞれの数の逆数の和が1になるものを求める

884 :デフォルトの名無しさん:2017/12/22(金) 19:39:05.90 ID:PPoMR9m8.net
早速間違えましたすみません
3 +5 +6 +8
でした

885 :デフォルトの名無しさん:2017/12/22(金) 20:06:25.02 ID:FRcsVGN9.net
>>883 ruby
f=->n,k{n==1?[[k]]:(1..k/n).flat_map{|i|f[n-1,k-i].map{|j|[i,*j].sort}}.uniq}
(1..22).each{|i|f[i,22].each{|a|p a if a.map{|e|1r/e}.sum==1}}

#=>[2, 4, 8, 8]
[2, 5, 5, 10]
[3, 3, 4, 12]

886 :デフォルトの名無しさん:2017/12/22(金) 21:21:23.60 ID:Mb+deFNF.net
C++で書いたけど、オセー。
デバッグ大変だ。
うーん。困ったなぁ。

887 :デフォルトの名無しさん:2017/12/22(金) 22:04:05.10 ID:Mb+deFNF.net
https://ideone.com/3ZWKv1
C++。こんなコード書いてみたけど、無理ゲー。
ちょっと厳しいなぁ。最近解けてないなぁ。
ちなみにデバッグ不完全なのであしからず。

888 :デフォルトの名無しさん:2017/12/22(金) 22:16:42.15 ID:cfNB9eDL.net
こういう数学的な問題を解くにはやっぱりプログラミング以前に数学を勉強した方がいいのでしょうか?

889 :デフォルトの名無しさん:2017/12/22(金) 22:23:25.38 ID:Mb+deFNF.net
>>888
数学は大事だよー。
俺数学出来ないから、解けない問題がそれなりにある。
算数では限界だ〜〜。
まぁ、数学とプログラミングって習得時はオーバーラップするところが少ないから融合するまでちょっと大変かな。
でも数学は強力なツールです。

890 :デフォルトの名無しさん:2017/12/22(金) 22:40:51.06 ID:wxhiocJz.net
一般論でいえば必要だろうけど>>883なんて全部列挙したところで計算量はたかが知れてるし
目下必要なのは論理学的思考能力なのでは

891 :デフォルトの名無しさん:2017/12/22(金) 22:51:38.40 ID:rSDEoHGj.net
>>883 Java
https://ideone.com/WiMkNl

22くらいなら大丈夫だけど桁が増えると(´・ω・`)

892 :デフォルトの名無しさん:2017/12/23(土) 10:08:16.78 ID:t1pvAVGb.net
お題
52をいくつかの自然数に分解して
それらの最小公倍数を最大化せよ

893 :デフォルトの名無しさん:2017/12/23(土) 10:30:51.64 ID:NDYwz7Jw.net
分解って、積じゃなくて和で良いんだよね?
数学の知識を使うと一瞬だけど

894 :デフォルトの名無しさん:2017/12/23(土) 10:47:21.72 ID:xQ13BTQc.net
>>892
2*3*5*7*11*23

895 :デフォルトの名無しさん:2017/12/23(土) 10:51:04.98 ID:NDYwz7Jw.net
>>894
ダメだろ

896 :デフォルトの名無しさん:2017/12/23(土) 10:54:01.61 ID:NDYwz7Jw.net
数学の知識が無いなら素直にコンピューターの力を借りなさい

897 :デフォルトの名無しさん:2017/12/23(土) 11:07:49.77 ID:afY4COyy.net
そもそも数学で簡単にとけない問題を力わざでとくための計算機だろ

898 :デフォルトの名無しさん:2017/12/23(土) 11:20:55.96 ID:2Y/dvvuZ.net
>>894
よくわからんが52の場合は
180180 [3, 4, 5, 7, 9, 11, 13]
ということらしいぞ

899 :デフォルトの名無しさん:2017/12/23(土) 11:22:52.27 ID:TLP4YLw7.net
2+3+5+7+11+13+11=52

900 :デフォルトの名無しさん:2017/12/23(土) 11:23:44.11 ID:ZIVZRbx3.net
C++17発行されたから開発環境がさっさと対応してGCDくらい使えるようになりたい。

901 :デフォルトの名無しさん:2017/12/23(土) 20:45:26.14 ID:PT43Bq9S.net
>>768の問題で0<=b<10,0<c<10という制限がついた時
移動を何回か繰り返すと必ず元に戻るんだけど
その回数はbには無関係にc,10-c,10の最小公倍数で
okかな?

902 :デフォルトの名無しさん:2017/12/23(土) 23:54:31.73 ID:PT43Bq9S.net
一回の移動ではd=1ね。
ま、その制限を付けなくともd,c,10-c,10の最小公倍数になるんだろうけど

903 :デフォルトの名無しさん:2017/12/24(日) 06:15:12.28 ID:C5ELqEVz.net
>>901-902
10-c,10の最小公倍数じゃね?

904 :デフォルトの名無しさん:2017/12/24(日) 12:14:01.21 ID:PCWcyI8B.net
>>899
>>896

905 :デフォルトの名無しさん:2017/12/24(日) 13:55:05.53 ID:aCkD6VOe.net
数学云々言ってる奴って、何故かその成果見せないよな。
俺でも出来そうなFUD、いやマウントかな。

906 :デフォルトの名無しさん:2017/12/24(日) 16:16:06.49 ID:7ASFTRv4.net
ていうか、計算機はどちらかというと算数だよな。
数学は公式とか証明とか、そういう手順みたいなものを考えるわけで、プログラミングに近い。
コンピュータは作られたプログラムに従って計算結果を出すだけ。
もちろんプログラムそのものをコンピュータに作らせることも可能だけどね。これは次元が違う話だよね。

907 :デフォルトの名無しさん:2017/12/24(日) 20:21:20.17 ID:TJswah5E.net
プログラムには算数と三角関数とかがあればいい
あとN進法

908 :デフォルトの名無しさん:2017/12/24(日) 22:49:45.08 ID:ke4WkGne.net
行列演算とか諸々の配列操作関数がないと無理だな

909 :デフォルトの名無しさん:2017/12/24(日) 23:49:37.37 ID:HHMC0VFW.net
では簡単なお題を
bを底とする値vを、2〜36進数に変換し表示してください。
なお、bは2〜36の整数、vは0以上の整数とし、不正な入力はないものとしてよい。
また、底と値の区切り文字は入出力ともに特に問わない。
[入力例]
16 deadbabe
[出力例]
2#11011110101011011011101010111110
3#100122100210210001200
4#3132223123222332
5#30122344134421
6#1414413520330
7#161402600604
8#33653335276
9#10570723050
10#3735927486
11#1647919685
(略)
27#9h9ll1i
28#7l225hi
29#6842o9l
30#53m7kg6
31#46f9hir
32#3farelu
33#2tf7mor
34#2e7m366
35#214kbpb
36#1ps9w3i

910 :デフォルトの名無しさん:2017/12/25(月) 00:09:45.09 ID:3pQBp6tI.net
>>909 Java 手抜き実装二つ
https://ideone.com/jybq1N
https://ideone.com/Wm8OQ2

911 : :2017/12/25(月) 00:32:41.94 ID:LEWwY/wL.net
>>909
c++old http://mevius.2ch.net/test/read.cgi/tech/1434079972/29

912 :デフォルトの名無しさん:2017/12/25(月) 02:44:06.33 ID:FXcNW9u1.net
>>909 ruby
n=eval"%2$p.to_i %1$d"%"16 deadbabe".split
(2..36).each{|i|puts"%d#%s"%[i,n.to_s(i)]}

913 :デフォルトの名無しさん:2017/12/25(月) 04:27:01.50 ID:Cnt90MG5.net
>>909
https://ideone.com/dxqIrP
C++。まぁこれくらいなら算数でも解ける範囲やな。
ただしコードがバグってないとは言ってない。へへ。

914 :デフォルトの名無しさん:2017/12/25(月) 06:21:01.65 ID:P1JMpVx5.net
>>909 lisp
https://ideone.com/6D7LRp

915 :デフォルトの名無しさん:2017/12/25(月) 12:28:00.12 ID:Lg9qxqUa.net
>>909
Kotlinらしくしてみようとはしたが、あまりにも短く、更に俺がまだよくKotlinを知らないためにこんな風になった。
https://paiza.io/projects/BpAXUQuDCaOSD6Q6GN4O8A
肝心な所はJavaとほぼ同じ。

916 :デフォルトの名無しさん:2017/12/25(月) 19:56:50.93 ID:IEH/2als.net
>>909 F#
https://ideone.com/rRZ949

917 :デフォルトの名無しさん:2017/12/26(火) 10:23:38.85 ID:Hd2qVaf/.net
>>909 Squeak/Pharo Smalltalk

| n |
n := '16 deadbabe' replaceAll: Character space with: $r; asNumber.
2 to: 36 do: [:i | Transcript cr; show: i; space; show: (n radix: i) asLowercase]

918 :デフォルトの名無しさん:2017/12/28(木) 04:53:27.57 ID:N8L362th.net
お題を捏造してやるぜ。
アンサーが42になる式を捏造せよ。という数学パズル。
小難しい式をでっち上げた人が優勝。
算数から数学、物理まで式になってればすべての手法が使用可能。統計とかでもいいよ。
制約は答えが42になることのみ。
解けるものはいるか?

919 :デフォルトの名無しさん:2017/12/28(木) 04:55:12.31 ID:N8L362th.net
あー、忘れてた。
ちゃんと検算して答えを確認できること。
俺、算数しかできないから、各種サービスにかけて検算できるのが望ましい。

920 :デフォルトの名無しさん:2017/12/28(木) 04:57:55.88 ID:8O6aNcDe.net
ぷろぐらみんぐ・・・?

921 :デフォルトの名無しさん:2017/12/28(木) 05:01:29.85 ID:N8L362th.net
ベンチマーク的な感じだな。
たまには本気を出したいだろ?お前ら。

922 :デフォルトの名無しさん:2017/12/28(木) 05:04:57.45 ID:N8L362th.net
当たり前だが、必要な関数が標準ライブラリになかったら自作すること。

923 :デフォルトの名無しさん:2017/12/28(木) 07:10:57.10 ID:s+AqweGp.net
>>918 ruby
require 'open-uri'

expr = "the Answer to the Ultimate Question of Life, the Universe, and Everything"

uri = "https://www.google.com/complete/search?output=toolbar&q=%s"
puts open(format(uri, expr.gsub(' ', '%20'))).string[/data="\K[^"]*/]
#=> 42

924 :デフォルトの名無しさん:2017/12/28(木) 07:34:31.76 ID:N8L362th.net
>>923
元ネタはそれ。正解の一端。

925 :デフォルトの名無しさん:2017/12/28(木) 08:07:25.24 ID:i+4FV8XV.net
>>918
brainfuck
https://ideone.com/eJSjws

926 :デフォルトの名無しさん:2017/12/28(木) 09:35:15.49 ID:wX0EFIYP.net
>>918
難しさの判定を人間が気分でするしかないとなると死ぬまで気に入らないと
言い続けて終わらないようにもできてしまうわけで、少なくともお題の判定
方法としては適切ではないのではないか?

927 :デフォルトの名無しさん:2017/12/28(木) 11:19:23.23 ID:ZkyapKMq.net
式を捏造せよと言ってんのに、検算して答えがあってることを確かめろとか矛盾してて草

928 :デフォルトの名無しさん:2017/12/28(木) 11:41:23.15 ID:N8L362th.net
>>925
基本的かつ合理的。

>>926
投票制にする?

>>927
答えは42になることだけは決まってるんだから、検算できないのはどういう理由?
プログラミングやるんだから、イデオンとか使うんじゃだめなの?

929 :デフォルトの名無しさん:2017/12/28(木) 15:25:23.94 ID:0tvuK50P.net
片山に次ぐ逸材かもしれないが出題者が馬鹿だとやる気が出ないという良い見本

930 :デフォルトの名無しさん:2017/12/28(木) 15:31:12.30 ID:N8L362th.net
自由を泳げないって不便だね。
何やっても良いんだからなんかすればいいって話なんだけど。
定型の答えなんか求めてないのは出題見ればわかるだろ。
発想力が欠如してるんじゃないか?

基本的にベンチマークだと言ってるでしょ?
捏造っていう言葉が悪かったら謝るが。構成しろってことにすれば大体同じや。

931 :デフォルトの名無しさん:2017/12/28(木) 15:32:51.23 ID:N8L362th.net
口だけのやつはぶっぶーですわ。

932 :デフォルトの名無しさん:2017/12/28(木) 16:00:26.20 ID:4ng0NpPh.net
自由を泳げないって

933 :デフォルトの名無しさん:2017/12/28(木) 18:49:12.75 ID:Er3In3fn.net
Cコ:彡

934 :デフォルトの名無しさん:2017/12/29(金) 00:30:23.43 ID:+gfutoXL.net
>>909 rust
https://ideone.com/H6mdNM
・BigInt不使用
・n.to_s(b)の形にしたかったが素早く諦めた
・色んなところに迷いと妥協が見え隠れ

935 :デフォルトの名無しさん:2017/12/29(金) 02:28:40.84 ID:IV3yH5ho.net
お題:入力があったら6面のサイコロを振って出た目を出力してください
ただし数字を使ってはならない

936 :デフォルトの名無しさん:2017/12/29(金) 03:07:31.93 ID:GekNq94X.net
そういう数字を使ってはいけないって誰得なの?

937 :デフォルトの名無しさん:2017/12/29(金) 05:31:16.53 ID:5y9SQxLe.net
>>935
Unicode の U+2680 〜 U+2685 は?
??????

938 :デフォルトの名無しさん:2017/12/29(金) 09:00:41.20 ID:IV3yH5ho.net
>>936
さぁ。。。

>>937
確認してないけどいいのでは

939 :デフォルトの名無しさん:2017/12/29(金) 09:20:10.35 ID:GekNq94X.net
>>935
https://ideone.com/GrYSWB
C++。こんな感じ?

940 :デフォルトの名無しさん:2017/12/29(金) 09:29:20.12 ID:GekNq94X.net
https://ideone.com/ZRXCD5
こっちの方がそれっぽいか。

941 :デフォルトの名無しさん:2017/12/29(金) 20:57:46.71 ID:QkO9em45.net
数字を使ってはならないってのが謎
AAで出力しろってか?

942 :デフォルトの名無しさん:2017/12/29(金) 21:21:03.51 ID:aTe03Y1I.net
>>935
Rubyで。
p rand('abcdef'.length) + 'z'.length

943 :デフォルトの名無しさん:2017/12/29(金) 21:28:13.36 ID:FAMD2vO+.net
数字を使うなって表示なのかそれともソースなのか?

表示なら
●●●●●●とか

944 :デフォルトの名無しさん:2017/12/29(金) 21:29:41.90 ID:u/2CuQjm.net
両方だろ

945 :デフォルトの名無しさん:2017/12/29(金) 21:50:46.64 ID:VnRfvHlH.net
そもそも入力が何なのかすら意味不明。却下

946 :デフォルトの名無しさん:2017/12/29(金) 22:22:21.11 ID:1z8qBjEb.net
お題
自然数 n を入力とし, a と b を乗ずると n になるような自然数 a と b を出力する.
a と b の侯補が複数存在する場合は, a と b の和がもっとも小さなものを出力すること.

947 :デフォルトの名無しさん:2017/12/29(金) 22:32:07.85 ID:5y9SQxLe.net
>>938
じゃあUnicodeのU+2680からの文字を使った版。Kotlinで。
https://paiza.io/projects/GElJ8jIbi45jocyYirbO8w
入力があったらの部分は最初の readLine() だ。
下の「入力」タブの所で改行を一つ入れてあるので開いたら即出力がある。

948 :デフォルトの名無しさん:2017/12/29(金) 23:03:49.04 ID:5y9SQxLe.net
>>935
また Kotlin。
サイコロの目の一つは5x5ビットあれば表現できるので配列に詰め込んで後で変換して出すようにした。
https://paiza.io/projects/uGhQ6cuRqbnlGc2TTvT92w?language=kotlin
別に配列でなくてもとにかく 5*5*6 (=150) bit 詰め込めるなら何でも良い。

949 :デフォルトの名無しさん:2017/12/29(金) 23:13:30.16 ID:VnRfvHlH.net
>>946
15.times{|n|
sqrt_n = Integer.sqrt(n)
(2 * sqrt_n..n + 1).each { |s|
(sqrt_n..n).each { |i|
next unless i * (s - i) == n
puts '%d * %d = %d' % [i, s - i, n]
break
} || break
}
}

0 * 0 = 0
1 * 1 = 1
1 * 2 = 2
1 * 3 = 3
2 * 2 = 4
5 * 1 = 5
2 * 3 = 6
7 * 1 = 7
2 * 4 = 8
3 * 3 = 9
5 * 2 = 10
11 * 1 = 11
3 * 4 = 12
13 * 1 = 13
7 * 2 = 14

950 :デフォルトの名無しさん:2017/12/29(金) 23:14:32.74 ID:VnRfvHlH.net
>>949はRuby2.5.0ね

951 :デフォルトの名無しさん:2017/12/29(金) 23:25:01.60 ID:IY4nOP57.net
>>935
https://paiza.io/projects/dpm1gJDuxMLd6pvQau8rig?language=kotlin

952 :デフォルトの名無しさん:2017/12/30(土) 00:34:17.12 ID:64dx8gku.net
>>946
def r9_946(n)
Math.sqrt(n).to_i.downto(1) do |e|
return [e, n / e] if (n / e) * e == n
end
end

1.upto(100) do |n|
a, b = r9_946(n)
printf("%d = %d * %d¥n", n, a, b)
end

953 :デフォルトの名無しさん:2017/12/30(土) 00:45:39.83 ID:6/kbfUjB.net
>>949から100倍くらい早くなった
https://ideone.com/4qZ3Di

954 :デフォルトの名無しさん:2017/12/30(土) 01:37:16.58 ID:2QbO+yEX.net
>>951
出目が0〜6?

確率的には
0: 1.56%
1: 9.38%
2: 23.44%
3: 31.25%
4: 23.44%
5: 9.38%
6: 1.56%
くらいか?

955 :デフォルトの名無しさん:2017/12/30(土) 09:12:17.38 ID:6VD4P8Az.net
>>946
https://ideone.com/GSawYb
C++。条件足りてるかよくわかってないけど、適当に書いたらそれぽい感じになった。
あってる保証はない。

956 :デフォルトの名無しさん:2017/12/30(土) 09:19:03.41 ID:6VD4P8Az.net
なんか俺の劣化>>953みたいな感じだな。
うーん。名案だとは思ったのだけど。むむむ・・・。

957 :デフォルトの名無しさん:2017/12/30(土) 09:31:25.37 ID:6VD4P8Az.net
うほ、フィルターしてる条件にバグがあった。良く動いてたな。

958 :デフォルトの名無しさん:2017/12/30(土) 09:33:18.81 ID:6VD4P8Az.net
これ、片方1のやつって素数かな?
エラトステネスの篩とどっちが軽いかな。

959 :デフォルトの名無しさん:2017/12/30(土) 12:46:04.20 ID:ZPxTZMGf.net
お題
要素が素数, かつ要素の総和が2018になる集合のうち, 要素数がもっとも大きい集合を出力する.

960 :デフォルトの名無しさん:2017/12/30(土) 12:59:47.50 ID:64dx8gku.net
>>959
[2]*(2018/2)

961 :デフォルトの名無しさん:2017/12/30(土) 14:25:23.55 ID:6VD4P8Az.net
>>959
https://ideone.com/lwcNQR
C++。DPの練習。必要な数はわかったが過程の表示の仕方がわからない。
どうすれバインダー。Orz

962 :デフォルトの名無しさん:2017/12/30(土) 15:14:55.24 ID:6VD4P8Az.net
>>961
https://ideone.com/DdepmL
C++。あってるか知らんけど、力業でベタ作業した結果、それっぽい数字にたどり着いた。
と、思ったら全然違う数字を指していた。

963 :デフォルトの名無しさん:2017/12/30(土) 15:16:57.66 ID:6VD4P8Az.net
>>959
https://ideone.com/XzeJ1E
C++。でけたー。DP難しいなぁ。

964 :デフォルトの名無しさん:2017/12/30(土) 15:18:24.57 ID:ZOKm+QEU.net
>>959
それ1なのでは?

965 :デフォルトの名無しさん:2017/12/30(土) 15:25:04.70 ID:6VD4P8Az.net
要素数だから、コンテナカウントだと思って書いたんだけど。
え?題意勘違いしてる?

966 :デフォルトの名無しさん:2017/12/30(土) 15:25:53.45 ID:6VD4P8Az.net
element countだよね?

967 :デフォルトの名無しさん:2017/12/30(土) 15:30:31.85 ID:qiSXHyFx.net
2が1009個ある集合、>>960で答えが出てる

968 :デフォルトの名無しさん:2017/12/30(土) 15:32:23.03 ID:6VD4P8Az.net
>>967
あー、それそういう意味だったのか。
うわー俺、蛇足だった。

969 :デフォルトの名無しさん:2017/12/30(土) 15:36:57.18 ID:6VD4P8Az.net
>>967
それをさ、プログラムで解くのきつくない?
総当たりしないと俺は無理。重複許可すると途端に大変になる。

970 :デフォルトの名無しさん:2017/12/30(土) 16:05:45.05 ID:6VD4P8Az.net
https://ideone.com/RVtf4i
適当に拡張してみたが、搭載メモリ8Gを使い切ってしまいデバッグ不可。
これ、意外と難問かもしれん。

971 :デフォルトの名無しさん:2017/12/30(土) 16:20:15.12 ID:6VD4P8Az.net
ちょっとくどいけど、
これさ、量子アニーリングじゃないと解けないやつかなぁ??
なんかさっきからいじってるけど、ローカルポケットに落ちてる気がする。
グローバルポケットに落とす方法が皆目見当つかない。
解説頼む。

972 :デフォルトの名無しさん:2017/12/30(土) 16:48:17.65 ID:ZPxTZMGf.net
すみません, お題での 集合 は Ruby では Setクラス のような, 要素に重複や順序性のないものを考えていました.
想定していた回答例は以下です.
answer
(2 3 5 7 11 13 17 19 23 29 31 37 41 43 47 53 59 61 67 71 73 79 83 89 97 101 103 107 113 127 131 137 139)
(apply #'+ answer)
2018
(length answer)
33

973 :デフォルトの名無しさん:2017/12/30(土) 17:07:30.49 ID:YKsh4iwJ.net
9個まではすぐ見つかるんだけど10個になった途端重くなる
10個の場合は存在しない?
逆に21個や33個の場合はすぐ見つかるんだ、どういう分布なんだろな

974 :デフォルトの名無しさん:2017/12/30(土) 17:11:07.91 ID:6VD4P8Az.net
>>972
おー、よかった。どこまで深淵があるのか怖かったよ。
多分、>>963であってると思う。たぶん。

975 :デフォルトの名無しさん:2017/12/30(土) 18:48:52.49 ID:YKsh4iwJ.net
>>959 C#
https://ideone.com/J9MmxS
33個と決め打ちした場合は4通りがすぐ出る
そのあと延々ループしてるがTime limit exceededで打ち切ってくれる

976 :デフォルトの名無しさん:2017/12/30(土) 21:03:54.80 ID:30TR5CU8.net
>>959
1も素数なんだが1が2018個ある集合はありなのか?
それだとお題としてほとんど意味のないひっかけ問題みたいになるわけだが、
そうではないなら問題を修正しろ。

977 :デフォルトの名無しさん:2017/12/30(土) 21:15:48.77 ID:KgXg1sy3.net
>1も素数なんだが
>1も素数なんだが
>1も素数なんだが

978 :デフォルトの名無しさん:2017/12/30(土) 21:23:36.74 ID:bLWDJrON.net
>>977
あ、2か。
でも問題がこれだと同じことだよなあ。

979 :デフォルトの名無しさん:2017/12/30(土) 21:26:45.29 ID:bA88XQgg.net
>>976
4つ上のレスも確認できないくせに何言ってんの

980 :デフォルトの名無しさん:2017/12/30(土) 21:42:01.90 ID:ZPxTZMGf.net
一般に, 素数は 1およびその数自身のほかに約数を有しない正の整数 と定義されますので, ここではその定義に従います.

また一般に, 重複や順序性のない もののあつまり を 集合(set) と呼ぶことが多いので, ここではその用法に従います.
集合(set)に対して, ものをならべたものは列(sequence)と呼ぶことが多いです.
ここでは『AABBCC』は文字列ですが, 文字集合ではないとします.

% irb
irb(main):001:0> require 'prime'
=> true
irb(main):002:0> 1.prime?
=> false
irb(main):003:0> 2.prime?
=> true
irb(main):004:0> require 'set'
=> true
irb(main):005:0> Set.new([1]*2018).size
=> 1

981 : :2017/12/31(日) 00:33:09.15 ID:UjqOw9qv.net
お題:指定した複数の wav フォーマットを連結して一つ wav ファイルを作成するプログラムを書け
・ファイルの指定方法はコマンドライン引数指定でかまわない
・wav ファイルフォーマットの仕様上の上限である 4GiB まで正常に結合できることを必須の最低条件とする
・PCM フォーマット・ステレオ2ch・サンプリング周波数 44.1kHz に対応しておればよい
・GUI に対応しておればなおよい

背景:いや、いろいろダウンロードして試しているのだけれども、4GiB まで正常に結合できるソフトウェアが見つからないのです‥

982 :デフォルトの名無しさん:2017/12/31(日) 01:09:52.02 ID:iFZSMKfw.net
それでこのスレに辿り着くのは面白い

983 :デフォルトの名無しさん:2017/12/31(日) 01:21:45.50 ID:QH0un2fa.net
前からこのスレにいる人でしょ。
お題としてはまったくこのスレに向いてないと思うが。

984 :デフォルトの名無しさん:2017/12/31(日) 01:55:26.47 ID:mjAZsjOp.net
2000から3000位まで試してみたが、大体33前後になるみたい
(微妙に増加していくが緩慢)

985 :デフォルトの名無しさん:2017/12/31(日) 03:02:32.19 ID:rf+Z6LCT.net
>>981
これ使えないか?
http://hakobe932.hatenablog.com/entry/20060613/1150208682

986 :デフォルトの名無しさん:2017/12/31(日) 05:23:36.13 ID:Q5J3BQB7.net
>>981
waveチャンクって2gbまでだっけ?sizeフィールドが32bitsignedだったような気がするんだけど。どうだっけ?

987 :デフォルトの名無しさん:2017/12/31(日) 05:28:08.43 ID:Q5J3BQB7.net
書き出すのはそんなに難しくないんだけど、読み込むのが面倒なんだよなぁ。
それに、適当にくっつけるとくっつけたところにブツ!っていうのノイズが入ることがあったはず。

988 :デフォルトの名無しさん:2017/12/31(日) 05:33:17.47 ID:Q5J3BQB7.net
https://ideone.com/AExWE6
これで、ちっちゃいやつは書き出した実績がある。ローカルの話だけどな。
読み込みはRiffの仕様よく知らないからわからない。

989 :デフォルトの名無しさん:2017/12/31(日) 09:49:37.67 ID:Jha/n6sD.net
自分で書くよりfoobar2000でMerge all tracks into one output fileしちゃうよな
むしろ6GBとかいける、wave64になってんのかな

990 :デフォルトの名無しさん:2017/12/31(日) 10:12:35.10 ID:vp+PvkVL.net
完全にスレチ

991 : :2017/12/31(日) 13:06:54.78 ID:UjqOw9qv.net
>>989
foobar2000 に merge する項目はありますか?
最新バージョンをインストールしましたが見当たりません‥

992 :デフォルトの名無しさん:2017/12/31(日) 14:14:14.10 ID:Jha/n6sD.net
>>991
foo_converter.dllが標準で入ってるからそのまま使えるよ
スレチというかこの場合はサイト違いだな、Hydrogenaudioで検索した方が沢山みつかる

993 :デフォルトの名無しさん:2017/12/31(日) 19:53:16.48 ID:R6E+DNla.net
"2018と素数" 類似問題
[お題]
前問よりどうやら、ユニークな素数の和で2018を作ると、
構成(要素)数 33個が最大で 4種類あるらしい。
最小は2個で27種類あるみたいだ。
3個だと 73種類、 4個だと 85014種類あるみたいだ。

ユニークな素数の和で2018を作る時、
最大の種類が作れるのは、構成数何個のときで、何種類か。

(注) 8個を超えると10億超えがしばらく続くらしい。

994 :デフォルトの名無しさん:2017/12/31(日) 19:58:31.90 ID:Q/CIq2T0.net
>>981
ちゃんと理想の仕様を書けば作るけど

995 :デフォルトの名無しさん:2017/12/31(日) 20:02:31.92 ID:Q/CIq2T0.net
>>986
32bit unsignedで(4Gi-1)Bまでだね
ファイルサイズも32bit unsigned

996 :デフォルトの名無しさん:2017/12/31(日) 22:29:40.39 ID:q2wUTltf.net
>>993 Java
https://ideone.com/DHe5Ij

997 : :2018/01/01(月) 00:07:03.13 ID:JOZ5/YyG.net
>>981
http://mevius.2ch.net/test/read.cgi/tech/1434079972/30
‥‥書初めになりました

998 :デフォルトの名無しさん:2018/01/01(月) 06:04:16.51 ID:4wMbPbHX.net
どうして2048ではなく2018などという中途半端な数にしたんだろうとずっと不思議に思っていたのだが(お題だから敢えて変な数にしたのかとか思ったんだが)、ようやっとわかったよ。今年の西暦年だったんだね。

999 :デフォルトの名無しさん:2018/01/01(月) 06:13:54.33 ID:+ZNxt5nC.net
>>995
勘違いしてたか。訂正ありがとう。

1000 :デフォルトの名無しさん:2018/01/01(月) 08:09:01.71 ID:OeEKMk/d.net
>>997
そっちのスレに書くと本当にBTC貰えるの?
てか、出題者と交渉したって貰える保障が全くなくて無意味なスレのような気がするんだけど。

1001 :2ch.net投稿限界:Over 1000 Thread
2ch.netからのレス数が1000に到達しました。

総レス数 1001
281 KB
掲示板に戻る 全部 前100 次100 最新50
read.cgi ver 2014.07.20.01.SC 2014/07/20 D ★